Case History and Data Interpretation in Medical Practice ABM Abdullah
INDEX
×
Chapter Notes

Save Clear


Case History and Data InterpretationAnswers Chapter I

“To all students of medicine who listen, look, touch and reflect: may they hear, see, feel and comprehend.”
— John B
318
319
 
Case No. 001
  1. Sideroblastic anemia with hemosiderosis.
  2. Bone marrow study to see ring sideroblast.
  3. Avoid iron.
Note: Blood picture is dimorphic, with severe anemia, high iron, ferritin and TIBC, which is highly suggestive of sideroblastic anemia.
Sideroblastic anemias are a group of disorders, in which dimorphic blood picture is associated with marked dyserythropoiesis and abnormal iron granules in the cytoplasm of erythroblast, called ring sideroblast (seen by Prussian blue staining).
Types of sideroblastic anemia:
  1. Hereditary.
  2. Acquired, which may be (i) Primary, (ii) Secondary.
Primary sideroblastic anemia is one of the types of myelodysplastic syndrome, a refractory anemia with ring sideroblast.
Secondary sideroblastic anemia may occur in (i) Drugs and chemicals (INH, cycloserine, alcohol, chloramphenicol, lead), (ii) Hematological disease (myelofibrosis, polycythemia rubra vera, myeloma, Hodgkin's lymphoma, hemolytic anemia, leukemia), (iii) Inflammatory disease (rheumatoid arthritis, SLE), (iv) Others (carcinoma, myxedema, malabsorption).
Treatment: Primary cause should be treated. In some cases, high dose pyridoxine, 200 mg daily may be helpful. Folic acid should be given.
 
Case No. 002
  1. HELLP syndrome (HELP syndrome).
  2. Reticulocyte count, viral screen (hepatitis A, B, E).
  3. Termination of pregnancy (or delivery of the baby).
Note: HELLP syndrome stands—H for hemolysis, EL for elevated liver enzyme, LP for low platelet in a patient with preeclampsia. HELLP syndrome is a variant of preeclampsia, affects 1 per 1000 pregnancies, common in multiparous women. It is common in multipara, perinatal mortality is 10 to 60% and maternal mortality is 1.5 to 5%. In 15% cases, BP may be normal and proteinuria may be absent. HELLP syndrome usually occurs in last trimester of pregnancy or within first week of delivery. Liver disease is associated with hypertension, proteinuria and fluid retention. Serum transaminases are high and the condition can be complicated by hepatic infarction and rupture. Differential diagnoses are HUS (hemolytic uremic syndrome), 320TTP (thrombotic thrombocytopenic purpura) and fatty liver. However, in TTP and HUS, no hypertension or no proteinuria. In fatty liver, transaminase are very high and clotting screen is usually abnormal.
Treatment of HELLP syndrome: (i) Deliver the fetus immediately, if >34 weeks gestation, (ii) In <34 weeks, intravenous steroid may be given, (iii) Control of hypertension, (iv) Platelet and blood transfusion may be necessary, (v) If convulsion, magnesium sulphate intravenously, (vi) Sometimes in severe renal failure, dialysis may be necessary.
 
Case No. 003
  1. Insulinoma.
  2. Factitious intake of insulin.
  3. Retroperitoneal fibrosarcoma, mesothelioma.
  4. Serum glucose with simultaneous insulin and C-peptide, USG or CT scan of pancreas, 72 hours fasting with measurement of glucose, insulin and C-peptide.
Note: Insulinoma is the commonest cause of nondiabetic fasting hypoglycemia. It is the insulin-secreting tumor of beta cell of pancreas, small <5 mm and occurs in any part of pancreas. 90% are benign and slow growing, 10% may be malignant. Common in middle age. Symptoms of hypoglycemia occur during fasting and relieve by taking food. The patient frequently takes food and gains weight.
There are more synthesis of pro-insulin, insulin and high C-peptide. It may be confused with self-induced insulin intake. In such a case, there is high serum insulin, but C-peptide is absent or low.
Insulinoma is detected by CT, MRI or endoscopic or laparoscopic USG.
Retroperitoneal fibrosarcoma and mesothelioma can cause hypoglycemia due to secretion of insulin-like substance (insulin-like growth factor-2).
Treatment: Resection, if possible. Diazoxide may be used. It reduces insulin secretion from the tumor. Symptoms may remit with octreotide or lanreotide (somatostatin analogue).
 
Case No. 004
  1. Bartter's syndrome.
  2. 24 hours urine potassium, serum renin and aldosterone (others—24 hours urinary calcium).
  3. Diuretic abuse, laxative abuse.
    321
Note: This young patient with the history has hypokalemia, metabolic alkalosis and normal blood pressure. Differential diagnoses are diuretic abuse, laxative abuse, self-induced vomiting, etc. All are absent in the history, only remaining is Bartter's syndrome. This disease is characterized by hypokalemia, metabolic alkalosis, normal blood pressure, high renin and high aldosterone, hypercalciuria. Primary defect is impairment of sodium and chloride reabsorption in the ascending limb of Henle's loop. Urinary calcium is >40 mmol/L. Prostaglandin levels are also high. It may be associated with short stature and mental retardation.
Diagnosis: Low serum potassium (<3 mmol) and high 24 hours urine potassium (>20 mmol) plus high renin and high aldosterone. Biopsy of the kidney shows hyperplasia of juxtaglomerular cells (prominent interstitial cells and interstitial fibrosis may be present).
Treatment: Potassium, spironolactone, ACE inhibitor in some cases. NSAID (indomethacin), interfere with tubular prostaglandin production may be helpful. Magnesium supplement may be necessary, if hypomagnesemia.
(Two other syndromes—(i) Liddle's syndrome—characterized by hypokalemia, metabolic alkalosis, high blood pressure, low renin and low aldosterone, (ii) Gitelman's syndrome—variant of Bartter's syndrome characterized by hypokalemia, metabolic alkalosis, normal blood pressure, high renin and high aldosterone, hypocalciuria, hypomagnesemia).
 
Case No. 005
  1. Acute intermittent porphyria.
  2. Urine for Ehrlich's aldehyde test, urine for δ-aminolaevulinic acid (ALA-increased), PBG (increase). Others—urine for porphobilinogen. Assay of red cells for the enzyme PBG deaminase (very low).
Note: AIP, inherited as autosomal dominant, is characterized by recurrent abdominal pain, neurological and psychiatric problem. There may be peripheral neuropathy (usually motor), respiratory failure, ophthalmoplegia, optic atrophy, constipation, psychiatric problem such as anxiety, depression and frank psychosis. Epileptic convulsion, hypertension and tachycardia may occur. Respiratory muscle paralysis may be life-threatening, may require assisted ventilation.
Common in female, usually after 30 years. Primary defect is deficiency of porphobilinogen deaminase in the heme biosynthetic 322pathway. Precipitating factors—barbiturate, oral pills, griseofulvin, sulphonamide, alcohol, even fasting.
When urine is kept for long time, it turns red brown or dark red color (Portwine). Bedside test is Ehrlich's aldehyde test in which reagent is added to urine which shows pink color due to presence of porphobilinogen (which persists after addition of chloroform). Pink color may be due to urobilinogen, which disappears after addition of chloroform. Na may be low due to SIADH.
Treatment: No specific treatment. Acute attack is treated by administration of large quantities of glucose 500 gm/day (it reduces ALA synthetase activity). If no response in 48 hours, intravenous infusion of heme (hematin or hemarginate) is helpful. Hypertension and tachycardia are treated with β-blocker. Avoid precipitating factors. High carbohydrate intake may be helpful. Cyclical attack in woman may respond to suppression of menstrual cycle by GnRH analogue.
 
Case No. 006
  1. Fitz-Hugh-Curtis syndrome (chlamydia or gonococcal perihepatitis).
  2. Acute cholecystitis, liver abscess.
  3. USG of HBS, X-ray chest, endocervical swab for microscopy and culture, urine for routine and culture.
Note: Fitz-Hugh-Curtis syndrome is caused by chlamydia or gonococcus infection, which tracks up the right paracolic gutter to cause perihepatitis, secondarily from endocervical or urethral infection. It is characterized by fever, pain in the right hypochondrium with radiation to right shoulder, tender hepatomegaly, hepatic rub, small right pleural effusion, etc. (chlamydia infection may be asymptomatic in 80% cases).
Investigation: Endocervical swab for microscopy and special culture, direct fluorescent antibody for chlamydia, ELISA, PCR may be done.
Treatment: Tetracycline or doxycycline or erythromycin or azithromycin are used for chlamydia infection.
 
Case No. 007
  1. Sarcoidosis, SLE, lymphoma, disseminated tuberculosis.
  2. Sarcoidosis.
  3. Full blood count and ESR, X-ray chest, MT, ANA and anti-ds DNA, FNAC or biopsy from lymph node.
    323
Note: Sarcoidosis is a multisystem granulomatous disorder of unknown cause characterized by presence of non-caseating granuloma in different organs. More in female than male, also common in black people. Usual features are fever, polyarthritis or arthralgia, erythema nodosum. X-ray shows bilateral hilar lymphadenopathy.
Other features: Dry cough, breathlessness, lupus pernio, plaque, skin rash, uveitis, bilateral parotid involvement, cardiomyopathy, arrhythmia, hepatosplenomegaly, etc. Neurological features are cranial nerve palsy, asceptic meningitis, seizure, psychosis, multiple sclerosis type syndrome (neurosarcoid).
Investigation: CBC (low lymphocyte), high calcium, high ACE (related to activity), chest X-ray (BHL), MT (usually negative), biopsy of involved tissue (lymph node, lung). Lung function test—restrictive with impaired gas exchange.
Bronchoscopy: Cobble stone appearance of mucosa, bronchial lavage shows increased CD4 : CD8 T cell ratio. CT scan may be done.
Biopsy shows non-caseating granuloma consisting of epithelioid cells, lymphocytes around it and multinucleated giant cells.
Two syndromes in sarcoidosis are (i) Heerfordt's or Waldenstrom's syndrome—fever, uveitis, parotid enlargement and seventh nerve palsy, (ii) Lofgren's syndrome—erythema nodosum, iritis and bilateral hilar lymphadenopathy.
Patient with sarcoidosis may have lacrimal and parotid gland enlargement, called Mikulicz syndrome, characterized by xerostomia and gritty eyes.
 
Case No. 008
  1. Berger's IgA nephropathy.
  2. Serum IgA, immune complex estimation, kidney biopsy.
  3. Henoch Schonlein purpura.
Note: There is history of sore throat followed by hematuria with urinary abnormality, indicates Berger's nephropathy. It is a type of immune-complex mediated focal and segmental proliferative glomerulonephritis with mesangial deposition of IgA. In some cases, IgG, IgM, C3 may be deposited in mesangium. Common in children and young males, 20 to 35 years of age. Most patients are asymptomatic, presents with recurrent microscopic or even gross hematuria. It may follow a viral respiratory or GIT infection. Hematuria is universal, proteinuria is usual and hypertension is common, 5% may develop nephrotic syndrome. 324In some cases, progressive loss of renal function, leading to end stage renal failure (20%) in 20 years.
Kidney biopsy shows focal proliferative glomerulonephritis. Immune deposits are present diffusely in mesangium of all glomeruli and contain usually IgA.
Treatment: Episodic attack resolves spontaneously. Patient with proteinuria over 1 to 3 gm/day, mild glomerular change and good renal function should be treated with steroid. In progressive renal disease, prednisolone plus cyclophosphamide for 3 months, then prednisolone plus azathioprine. Combination of ACE inhibitor and ARB should be given to all cases. Tonsillectomy may be helpful, if recurrent tonsillitis.
 
Case No. 009
  1. Thyrotoxic crisis.
  2. FT3, FT4 and TSH.
Note: Clue for the diagnosis is goiter with loss of weight, loose motion, palpitation which are due to thyrotoxicosis. Consolidation has precipitated thyrotoxic crisis. It is characterized by life-threatening increase in severity of the signs and symptoms of thyrotoxicosis (also called thyroid storm). It is characterized by high fever, restlessness, agitation, irritability, nausea, vomiting, diarrhea, abdominal pain, confusion, delirium and coma. Precipitated by infection, stress, surgery in unprepared patient and following radio-iodine therapy (due to radiation thyroiditis).
Treatment: (i) Better in ICU, (ii) carbimazole 15 mg 8 hourly or propylthiouracil 150 mg 6 hourly, (iii) propranolol 80 mg 6 hourly, (iv) IV-normal saline and glucose, (v) antibiotic, (vi) steroid-dexamethasone (2 mg 6 hourly), (vii) Na-iopodate (radiographic contrast)—500 mg daily orally rapidly effective, reduces release of thyroid hormone and also reduces conversion of T4 to T3. Mortality rate in thyrotoxic crisis is 10%.
 
Case No. 010
  1. Hyperosmolar non-ketotic diabetic coma with UTI.
  2. RBS, urine for ketone bodies, serum osmolality, urine for C/S.
  3. IV 1/2 strength saline (0.45%), soluble insulin (preferably with insulin pump, 2 to 6 units hourly). When osmolality is normal, 0.9% normal saline should be given.
    325
Note: Hyperosmolar non-ketotic diabetic coma (HNDC) may be the first presentation in diabetes mellitus. Common in elderly, NIDDM, characterized by very high blood glucose (>50 mmol/L) and high plasma osmolality. No ketosis, because insulin deficiency is partial and low insulin is present, which is sufficient to prevent ketone body formation, but insufficient to control hyperglycemia. Hyperchloremic mild acidosis may be present due to starvation, increased lactic acid and retention of inorganic acid. Precipitating factors are large amount of sweet drink, infection, steroid, thiazide, myocardial infarction, etc. Plazma sodium is usually high (may be false low due to pseudohyponatremia). There may be high BUN, urea, creatinine and serum osmolality may be very high. Mortality rate may be up to 40%. Osmolality is calculated by = 2 × Na + 2 × K + plasma glucose + plasma urea (all in mmol/L).
Normal 280 to 300 mosm/kg. Consciousness is depressed, if it is > 340 mosm.
Other treatment: NG tube feeding, catheter if needed, antibiotic if infection, correction of electrolytes, low dose heparin (as thrombosis is common).
 
Difference between diabetic ketoacidosis and HNDC
Points
DKA
HNDC
1. Age
Young, may be any age
Elderly, >40 yrs
2. Precipitating factor
Insulin deficiency
Partial insulin deficiency
3. Breath
Acetone present
Absent
4. Kussmaul's breathing
Present
Absent
5. Sodium
Low
High
6. Bicarbonate
Low
Normal
7. Ketonuria
Present
Absent
8. Osmolality
Normal
High
9. Mortality
5 to 10%
30 to 40%
 
Difference between diabetic ketoacidosis and lactic acidosis
Points
DKA
Lactic acidosis
1. Precipitating factor
Insulin deficiency, infection, no biguanide
Biguanide
2. Dehydration
Present
Absent
3. Breath
Acetone
Absent
4. Ketonuria
Present
Absent or mild
5. Serum lactate
Normal
High >5 mmol/L
6. Mortality
5 to 10%
>50%
326
 
Case No. 011
  1. Bilateral occipital cortex (cortical blindness).
  2. Basilar artery occlusion by mural thrombus from left ventricle.
  3. MRI of brain.
  4. Poor.
  5. Murmur is due to mitral regurgitation or VSD.
Note: Thromboembolism may occur following myocardial infarction. Widespread bilateral cortical damage causes cortical blindness (Anton's syndrome). Causes are infarction, trauma, tumor. The patient cannot see and lacks insight into the degree of blindness and may even deny it. Pupillary response is normal. In myocardial infarction, rupture of the papillary muscles causes mitral regurgitation and rupture of interventricular septum causes VSD.
 
Case No. 012
  1. Any skin rash with the onset of fever, which disappears with fall of temperature (Salmon rash).
  2. Adult Still's disease.
  3. Lymphoma, SLE (others—septicemia, disseminated tuberculosis).
  4. Serum ferritin (very high), ANA, anti-ds DNA, FNAC of lymph node.
Note: High fever, polyarthritis, pleurisy, hepatosplenomegaly, lymphadenopathy are consistent with adult Still's disease. It may be confused with SLE, lymphoma, disseminated tuberculosis. ANA is negative and also high CRP is against SLE. Lymphoma is another possibility, but very high fever, seronegative arthritis, leucocytosis, etc. are against lymphoma.
Adult Still's disease is a clinical condition of unknown cause, characterized by high fever, seronegative arthritis, skin rash and polyserositis. It is usually diagnosed by exclusion of other diseases. Common in young adult, 16 to 35 years of age, rare after 60 years.
Diagnostic criteria of adult Still's disease are:
  1. Each of the four (i) fever, >39°C, (ii) arthralgia/arthritis, (iii) RA- negative, (iv) ANA-negative.
  2. Plus two of (i) Leucocytosis >15,000/cmm, (ii) evanescent macular/maculopapular rash, Salmon colored, nonpruritic, (iii) serositis (pleurisy, pericarditis), (iv) hepatomegaly, (v) splenomegaly, (vi) lymphadenopathy.
    327
Lymph node biopsy shows reactive hyperplasia.
Treatment: NSAID, high dose steroid, disease-modifying drugs.
 
Case No. 013
  1. Late onset congenital adrenal hyperplasia (CAH).
  2. Polycystic ovarian syndrome (PCOS).
  3. Dexamethasone suppression test.
  4. Steroid (prednisolone or dexamethasone).
  5. Serum 17-hydroxyprogesterone (high), urine for pregnanetriol (high).
Note: Differential diagnosis of hirsutism are congenital adrenal hyperplasia, polycystic ovarian syndrome (PCOS), androzen-secreting adrenal tumor, Cushing's syndrome, arrhenoblastoma. Serum 17-hydroxyprogesterone is high in 21 β-hydroxylase deficiency. In late onset CAH, this may be normal and if it is high after synacthen test, is highly suggestive of late onset CAH (which is due to mild enzyme defect).
After dexamethasone suppression test, there is fall of 17-hydroxyprogesterone and other androgens in CAH, but in PCOS and other androgen-secreting tumor of ovary or adrenal cortex, these do not fall after dexamethasone suppression test.
Hirsutism is present in late onset congenital adrenal hyperplasia. Also, the patient is tall, there is precocious sexual development. In children with CAH, there may be salt loss, adrenal crisis, increased pigmentation and ambiguous genitalia.
In 95% cases, CAH is due to 21 β-hydroxylase deficiency.
Rarely, 11 β-hydroxylase deficiency or 17 α-hydroxylase deficiency may be present. Both are associated with hypertension. There is less synthesis of cortisol, aldosterone and increased synthesis of 17-hydroxyprogesterone, testosterone, other androgen such as dihydro-epiandrosterone, androstenedione, increased ACTH and 24 hours urinary pregnanetriol.
Treatment: Steroid in high dose at night and low dose in morning. Late onset CAH may not require steroid. Hirsutism should be treated with anti-androgen.
 
Case No. 014
  1. Meningococcal septicemia with DIC.
  2. Dengue hemorrhagic fever.
    328
  3. Prothrombin time, APTT, serum FDP.
  4. Blood culture and sensitivity.
  5. LP and CSF study.
Note: High fever, multiple purpuric rash with DIC are highly suggestive of meningococcal septicemia, caused by Neisseria meningitidis. Septicemia due to other cause may be also responsible. Full blood count, blood culture, LP and CSF study are necessary. Meningococcal meningitis and septicemia are caused by Neisseria meningitidis, a gram-negative diplococcus. Septicemia may be associated with multiple petechial hemorrhage, also conjunctival hemorrhage. In CSF, pressure is high, color is purulent, high neutrophil, high protein and low glucose. Gram staining, culture and sensitivity are also necessary. Complications—DIC, peripheral gangrene following vascular occlusion, Waterhouse-Friderichsen syndrome, renal failure, meningitis, shock, arthritis (septic or reactive), pericarditis may occur. Skin rash in 70% (by N. meningitidis).
After sending blood for C/S, treatment should be started immediately. Usually, benzyl penicillin in high dose plus cephalosporine or aminoglycoside should be given intravenously.
LP is mandatory, provided there is no contraindication.
Leucocytosis is not a feature of dengue hemorrhagic fever.
 
Case No. 015
  1. Spontaneous bacterial peritonitis (SBP).
  2. Aspiration of ascitic fluid for cytology, biochemistry and C/S.
  3. Liver transplantation.
Note: In any patient with CLD with ascites, if fever develops, SBP is the most likely diagnosis. Spontaneous bacterial peritonitis means infection of the ascitic fluid in a patient with cirrhosis of liver, in the absence of primary source of infection. SBP may develop in 8% cases of ascites with cirrhosis (may be as high as 20 to 30%), usually by single organism (monomicrobial).
It is due to migration of enteric bacteria through gut wall by hematogenous route or mesenteric lymphatics, commonly caused by E coli. Other organisms are Klebsiella, Hemophilus, Enterococcus, other enteric gram-negative organism, rarely Pneumococcus, Streptococcus. Anaerobic bacteria are not usually associated with SBP.329
Features of spontaneous bacterial peritonitis:
  1. Fever, adominal pain, rebound tenderness, increasing ascites not responding to diuretic, absent bowel sound. Features of hepatic encephalopathy and fever may occur (in 1/3rd cases).
  2. Abdominal symptoms may be mild or absent (in 1/3rd cases) and in this patient, PSE and fever are main features.
  3. Ascitic fluid shows (i) cloudy (exudative—high protein and low sugar), (ii) neutrophil in fluid >250 or WBC >500/cmm.
Treatment: Broad spectrum antibiotic (cefotaxime or ceftazidime) plus metronidazole IV. Recurrence is common (70%) within one year, may be prevented by norfloxacin 400 mg daily or ciprofloxacin 250 mg daily. Mortality is 10 to 15%. SBP is an indication for referral to liver transplant center.
 
Case No. 016
  1. Avascular necrosis of head of femur with lupus gut (with UTI with lupus nephritis).
  2. Avascular necrosis of head of femur.
  3. MRI of hip joint.
Note: Any patient who is getting steroid for long time, if complains of severe backache with difficulty in walking, avascular necrosis should be excluded. In SLE, avascular necrosis may be due to prolonged use of steroid and also due to vasculitis. MRI is the most sensitive test to diagnose early. X-ray may be normal in early case.
Limping gait is a recognized complication of SLE. Severe acute abdominal pain, nausea, vomiting, diarrhea, etc. may occur in SLE due to vasculitis (called lupus gut).
 
Case No. 017
  1. Fat embolism in brain.
  2. CT scan/MRI of brain, urine for fat cells.
Note: Fat embolism may occur after bony fracture, more common with closed than open fracture. It is usually caused by trauma to the long bone or pelvis, orthopedic procedures, sometimes parenteral lipid infusion, recent corticosteroid therapy, burns, liposuction, sickle cell crisis. Fat embolism is characterized by hypoxemia, patechial rash over the upper part of the body, neurological abnormality, within 24 to 33072 hours of injury. Retinal hemorrhage with intraarterial fat globules may be seen with fundoscope. Fat cells can be detected in urine or sputum. Treatment is supportive, such as maintenance of oxygenation, hydration prophylaxis against thrombosis, nutrition, etc.
 
Case No. 018
  1. Postpartum cardiomyopathy.
  2. Echocardiography, serum electrolytes.
Note: If any patient after delivery develops pulmonary edema or features of heart failure, postpartum cardiomyopathy is the likely diagnosis. Differential diagnosis is pulmonary embolism. Postpartum cardiomyopathy is characterized by dilatation of the heart with biventricular failure, usually in the last trimester or within 6 months of delivery. It is a type of dilated cardiomyopathy. Cause is unknown. Immune and viral causes are postulated. Other factors are advanced age, multiple pregnancy, multiparity and hypertension in pregnancy. Commonly occurs immediately after or in the month before delivery (peripartum).
It occurs usually in multipara, age above 30 years. The patient usually presents with heart failure, dyspnea, orthopnea, cough with frothy sputum, weakness, pain in abdomen, swelling of leg, etc. Atrial fibrillation or other arrhythmia may occur.
Diagnostic criteria are (four criteria):
  1. Presentation in last month of pregnancy or within five months of delivery.
  2. Absence of an obvious cause for heart failure.
  3. Previously normal cardiac status.
  4. Echocardiographic evidence of systolic left ventricular dysfunction.
Treatment: Symptomatic for heart failure (diuretics, ACE inhibitor, digoxin). β-blocker may be helpful in some cases. Inotrophic agent may be given. More than half cases have a complete or near complete recovery over several months. Immunosuppressive therapy has doubtful value. Mortality rate is 10 to 20%.
The patient should avoid subsequent pregnancy, due to risk of relapse. However, if the heart size is normal in the first episode following heart failure, subsequent pregnancy is tolerated in some cases. If heart size remains enlarged, further pregnancy causes refractory chronic heart failure.331
 
Case No. 019
  1. Primary hyperparathyroidism.
  2. Serum Ca, PO4 and alkaline phosphatase, parathormone assay, hydrocortisone suppression test, X-ray of hand (to see subperiosteal erosion in the medial side of phalanges) and X-ray of skull (to see pepper pot), USG or CT scan of neck (other test—thallium/ technetium substraction scan of thyroid and parathyroid).
  3. Plenty of fluid (infusion of normal saline 4 to 6 liters daily) for hypercalcemia.
Note: This patient presents with the features of hypercalcemia and nephrocalcinosis, suggestive of hyperparathyroidism. It may be primary (due to adenoma, hyperplasia or carcinoma of parathyroid), secondary (chronic renal failure, malabsorption, ricket or osteomalacia) or tertiary (autonomous from secondary). Most symptoms are due to hypercalcemia. There may be band keratopathy (corneal calcification), nephrocalcinosis, pancreatitis, peptic ulcer, pseudogout, spontaneous fracture, osteoporosis. X-ray of skull shows pepper pot and loss of lamina dura. X-ray of hand shows resorption of terminal phalanges.
Treatment: For primary hyperparathyroidism, total parathyroidectomy followed by transplantation of a small amount of parathyroid in the forearm muscles. After surgery, calcium may fall rapidly and tetany may occur. Hypocalcemia may persist for several months. So, it is necessary to continue vitamin D supplement. In mild and asymptomatic case, follow up. Surgery is also required for tertiary hyperparathyroidism. Treatment of secondary causes should be done.
 
Case No. 020
  1. Cerebral lupus erythematosus with autoimmune hemolytic anemia with renal lupus.
  2. ANA, anti-ds DNA, antiphospholipid antibody, MRI of brain.
Note: In SLE, cerebral involvement may occur in up to 15% of cases. The patient may present only with psychiatric manifestations, such as behavior abnormality, irritability, confusion, hallucination, obsessional or paranoid or frank organic psychosis. Epilepsy or convulsion, stroke, peripheral neuropathy, chorea, transverse myelitis may also occur. CSF will show features of asceptic lymphocytic meningitis. Antiphospholipid or anticardiolipin antibody is present in 30 to 40%. ANA is positive 332in 90%, anti-dsDNA is positive in up to 60%. CRP is usually normal, high if secondary infection. Complement, mainly C3 is low.
Treatment: High dose prednisolone or methylprednisolone plus pulse cyclophosphamide intravenously.
 
Case No. 021
  1. Cerebral abscess.
  2. Full blood count (leucocytosis), CT scan or MRI of brain, blood for culture and sensitivity.
Note: Brain abscess may occur as a complication from ear infection. Other causes include infection in nose, paranasal sinus and tooth, head injury, penetrating trauma, septicemia, HIV infection, immunocompromised case, Fallot's tetralogy, etc. Organisms are streptococcus angiosus, bacteroids, staphylococcus, fungus. Mixed infections are common. Multiple abscesses are common in HIV. CT scan will show ring-like shadow in the brain.
Treatment: Broad spectrum antibiotic, metronidazole (in anaerobic). Surgery may be necessary, if drug fails. Mortality—25%. Epilepsy may occur in survivor.
 
Case No. 022
  1. Neuroleptic malignant syndrome.
  2. ANA and anti-ds DNA (to exclude SLE), serum ferritin (to exclude adult Still's disease).
  3. Serum CPK (raised).
Note: Neuroleptic malignant syndrome is a rare, but serious complication of any neuroleptic drug therapy such as phenothiazine, butyrophenons (commonly haloperidol), irrespective of dose. It occurs in 0.2% of cases, usually after days or weeks of neuroleptic drug therapy. May be precipitated after abrupt withdrawal of antiparkinsonian drug. It is characterized by high fever, stiffness of body, fluctuating consciousness, autonomic dysfunction (tachycardia, labile BP, pallor). There may be leukocytosis and abnormal liver function tests. High CPK (due to myonecrosis) is highly suggestive of the diagnosis. Sometimes, metabolic acidosis, respiratory failure, cardiac failure, rhabdomyolysis and even renal failure may occur.
Mortality is 20% in untreated cases and 5% in treated cases.
Treatment: (i) Stop the offending drug (ii) dopamine receptor agonist—bromocriptine (iii) antispastic agent—dantroline IV may be helpful (iv) supportive therapy (hydration, reduction of temperature).333
 
Case No. 023
  1. Obstructive airway disease.
  2. Pulmonary emphysema.
  3. X-ray chest, HRCT scan of chest.
Note: Ratio of FEV1: FVC is low, which indicates obstructive airway disease. Low ratio, reduced transfer factor of CO and high residual volume is highly suggestive of emphysema. HRCT scan is the most sensitive noninvasive investigation for diagnosis of emphysema. It is diagnosed by histological examination (not done usually).
Other causes of low transfer factor are pulmonary fibrosis, severe ventilation/perfusion mismatch, pulmonary edema, lobectomy, severe anemia.
 
Case No. 024
  1. Henoch Schonlein purpura.
  2. Serum IgA, skin biopsy.
  3. Renal biopsy.
Note: Combination of abdominal pain, bloody diarrhea, arthritis and purpura in a young patient is highly suggestive of Henoch Schonlein purpura. This is a small vessel vasculitis, common in boys of 5 to 15 years, but may occur in any age. It is characterized by purpura or petechial rash (in buttock and ankles), polyarthritis (in big joints), glomerulonephritis and gastrointestinal symptoms (pain and bleeding). If occurs in older children and adults, GN is more prominent. Acute abdominal pain may be due to intussusception. It usually follows upper respiratory infection. There is vasculitis, bowel is edematous and inflamed causing bleeding and obstruction. 30 to 70% have renal involvement with hematuria and proteinuria. Renal disease is usually mild, but nephrotic syndrome and ARF may occur. Only 1% develops end stage renal failure.
Diagnosis is usually clinical. Serum IgA is high in 50% cases. Skin biopsy may show leukocytoclastic vasculitis with IgA deposition. Kidney biopsy shows IgA deposition within and around the blood vessels. There may be focal and segmental proliferative GN.
Treatment: Usually self-limiting. Steroid is indicated, if there is GIT and joint symptoms. If renal involvement, pulse IV steroid (methylprednisolone) and immunosuppressive therapy. Prognosis is good in children, relatively worse in adults.334
Adverse factors in adult, if presents with hypertension, abnormal renal function and proteinuria >1.5 gm/day.
 
Case No. 025
  1. Gilbert's syndrome.
  2. 48 hours 400 kcal restriction test, IV 50 mg nicotine (there is rise of bilirubin).
  3. Reassurance, therapeutic trial with phenobarbitone 60 mg TDS. Avoid prolong fasting.
Note: In any young patient with recurrent jaundice, Gilbert's syndrome is the commonest cause. This syndrome is a type of unconjugated nonhemolytic hyperbilirubinemia. It occurs in 2 to 7% of normal individual, some cases may be inherited as autosomal dominant.
Most patients remain asymptomatic, diagnosis may be done as an incidental finding during routine examination. Jaundice is usually mild, occurs intermittently during infection or prolonged fasting. All other liver biochemistry are normal and no signs of liver disease. It is due to defect in the uptake of bilirubin by the liver and also there is deficiency of UDP glucuronyl transferase activity, the enzyme that conjugates bilirubin with glucuronic acid. Usually no hemolysis, bilirubin in urine is absent, urobilinogen in urine is present.
Liver biopsy—normal (may show increased centrilobular lipofuscin).
Treatment: Reassurance, avoid fasting.
Other causes of nonhemolytic hyperbilirubinemia are Criggler Najjar syndrome (type 1 and 2), Dubin Johnson syndrome (liver is black due to increased deposition of lipofuscin and melanin), Rotor syndrome.
 
Case No. 026
  1. Rhabdomyolysis with acute renal failure.
  2. Urine for ammonium sulfate test, spectroscopic examination of urine to detect myoglobin.
  3. Hemodialysis.
Note: In this case, clue to the diagnosis of rhabdomyolysis are history of multiple injury, followed by severe renal failure. Acute muscle destruction is called rhabdomyolysis, associated with high myoglobinemia and myoglobinuria. Myoglobin is highly toxic to the renal tubules and precipitates renal failure. Urine is red, but no RBC. Myoglobinuria gives a false positive dipstick result for blood 335(hemoglobin), which can be distinguished by the ammonium sulphate test. This test gives a colored precipitate with hemoglobinuria and colored supernatant with myoglobinuria.
Rhabdomyolysis is associated with high AST, CPK, creatinine, potassium, phosphate and uric acid. There is low calcium, because free calcium becomes bound by myoglobin.
Treatment: Supportive, such as adequate hydration, alkalinization of urine to reduce precipitation of myoglobin in the renal tubules. Dialysis, if renal failure. Nonsymptomatic hypocalcemia dose not require treatment. Loop diuretics should be avoided, as they result in an acidic urinary pH.
Causes of rhabdomyolysis- trauma (crush injury), severe exercise, convulsion or epilepsy, electrocution, hypothermia, heat stroke, alcoholism, polymyositis, neuroleptic malignant syndrome, burn, septicemia, infection (influenzae, Legionnaire's disease), ecstasy or amphetamine abuse. Rhabdomyolysis is one of the important causes of acute renal failure.
 
Case No. 027
  1. Giant cell arteritis (GCA).
  2. Temporal artery biopsy.
  3. High dose prednisolone to prevent blindness.
Note: In any elderly patient with unilateral headache associated with unexplained fever, arthritis, etc. with very high ESR, always think of giant cell arteritis. GCA is an inflammatory granulomatous arteritis of unknown cause involving the large arteries, predominantly affecting temporal and ophthalmic artery. Common in female (F:M = 4:1), 60 to 75 years of age, rare < 50 years.
Headache is invariable, mostly on temporal and occipital region. There is local tenderness, temporal artery is thick, hard, tortuous and tender. Jaw claudication, worse on eating, pain in the face, jaw and mouth (due to involvement of facial, maxillary and trigeminal branch of external carotid artery), TIA, visual disturbance, even blindness may occur. It is one of the important causes of PUO in elderly. Simple test is very high ESR and high CRP. ANA and ANCA are negative.
Temporal artery biopsy shows (i) Intimal hypertrophy, (ii) Inflammation of intima and subintima, (iii) Breaking up or fragmentation of internal elastic lamina, (iv) Infiltration of lymphocyte, plasma cells and giant cells in internal elastic lamina with necrosis of arterial media.336
Biopsy from the affected site should be taken before starting or within seven days of starting steroid. Whole length of the artery (>1 cm) should be taken, because the lesion is patchy or skip. Biopsy is negative in 30% cases, due to typical patchy nature of inflammation. Negative does not exclude the diagnosis.
Treatment: Prednisolone 60 to 100 mg/day for 1 to 2 months, then taper slowly (with the guide of ESR and CRP). To be continued for long time, 75% settle in 12 to 36 months. 25% may require low dose steroid for years. In some cases, steroid may be needed for lifelong. ESR and CRP are done which are the markers of disease activity. Relapse may occur in 30% cases. May be difficult to taper the dose, then methotrexate or azathioprine may be added.
 
Case No. 028
  1. Intestinal malabsorption.
  2. Celiac disease.
  3. Jejunal biopsy, serum antiendomysial antibody detection.
  4. Dermatitis herpitiformis.
  5. Howel-Jolly body, target cells.
  6. Avoid gluten-containing diet.
Note: Blood picture is dimorphic (both macrocytic and microcytic), due to deficiency of both iron, folic acid and rarely B12. Diarrhea, weight loss, abdominal discomfort, all are suggestive of intestinal malabsorption, associated with itchy vascular rash which is due to dermatitis herpitiformis. All the features are suggestive of celiac disease.
Celiac disease is characterized by mucosal destruction of proximal small bowel due to hypersensitivity to gliadin fraction of gluten protein. There is atrophy of villi, crypt hypertrophy, infiltration of plasma cells and lymphocytes in lamina propria.
Antiendomysial antibody (present in 50%) and tissue transglutaminase antibody may be present in the serum, which are highly sensitive and specific. Antiendomysial antibodies are IgA antibodies, which may not be detected in patient with low IgA antibody level. Since celiac disease may be associated with IgA deficiency, it is important to ward off serum IgA level, before interpretating antiendomysial antibodies. Antireticulin antibody may be present which is very sensitive, but less specific. Antigliadin antibody is also less sensitive (not used).337
Complications: Hyposplenism, gastric or small bowel lymphoma, esophageal carcinoma may occur. Dermatitis herpitiformis may be associated with celiac disease. Target cells, Howel-Jolly body are due to hyposplenism.
Treatment: Avoid gluten-free diet (wheat, rye, barley, oat). Iron, folic acid, calcium should be given.
 
Case No. 029
  1. Toxic shock syndrome.
  2. Blood for C/S, high endocervical swab for C/S.
  3. History of using tampons.
  4. DIC.
Note: In a menstruating female, present with burning micturition, multiple skin rash and high fever, likely diagnosis is toxic shock syndrome. It is characterized by local infection in genital tract by Staph. aureus producing exotoxin which causes high fever, shock, diffuse macular rash, desquamation of palms and soles, widespread damage of multiple organs. TSS toxin 1 is responsible in 75% cases. Local source of infection usually from tampons used during menstruation. Blood culture is usually negative. DIC may occur.
Diagnosis is done by 5 criteria (i) Temperature >39°C, (ii) Widespread erythematous macular rash, (iii) BP systolic <90 mmHg or postural diastolic drop, (iv) Source of localized TSST producing infection, (v) Evidence of toxic action on 3 systems—diarrhea or vomiting, myalgia or high CPK, drowsiness or confusion, high urea or creatinine, low platelet (<1,00,000).
Treatment: Management of shock, inotropic support plus IV flucloxacillin. Mortality—10%.
 
Case No. 030
  1. Tricyclic antidepressant poisoning (TCA).
  2. ECG, arterial blood gas analysis.
  3. Gastric lavage, cardiac monitoring.
Note: Clue of the diagnosis is history of taking drugs, unconsciousness, dilated pupil, tachycardia are all suggestive of anticholinergic side effects of tricyclic antidepressant drug. TCA poisoning is characterized by drowsiness, confusion, delirium, hallucination, agitation, myoclonic fit, convulsion and coma. Pupil dilated, loss of accommodation. Reflex- exaggerated, hypertonia or spasticity.338
Plantar–extensor, divergent strabismus, retention of urine may occur.
ECG–sinus tachycardia, QRS is prolonged, P is small, arrhythmia (SVT, VT).
There may be metabolic acidosis, respiratory failure. Most patients recover in 48 hours. However, in some cases, there may be persistent agitation, confusion, hallucination, rapid jerky movement which may last for several days.
Treatment: (i) Gastric lavage, if >250 mg tablet is taken. TCA cause delay gastric emptying, so lavage can be given up to 12 hours of poisoning, and activated charcol may be given. (ii) Protection of airways and oxygen is given via mask. (iii) Intravenous fluid. (iv) Cardiac monitor—arrhythmia may develop (does not require specific therapy). (v) If epileptic seizure, IV lorazepam or diazepam should be given. (vi) For acidosis, sodibicarb to produce alkalemia is helpful (it increases binding of TCA with protein). No role of forced diuresis or hemodialysis.
 
Case No. 031
  1. Lactic acidosis due to metformin.
  2. Serum lactate level, urine for ketone body, blood for pH.
  3. IV isotonic (1.26%) sodium bicarbonate (high dose), IV soluble insulin and glucose.
  4. Diabetic ketoacidosis, renal failure, renal tubular acidosis, salicylate poisoning.
Note: Here, there is severe metabolic acidosis with high anion gap. In lactic acidosis, there is high anion gap without hyperglycemia or ketosis. Mortality is >50%. Na-dichloroacetate may be given to lower lactate.
Lactic acidosis may be of 2 types:
  • Type A—due to tissue hypoxia. It may occur in shock, severe anemia, cyanide or carbon monoxide poisoning, respiratory failure.
  • Type B—due to impaired hepatic metabolism. It may occur in diabetes mellitus, hepatic failure, drug (biguanide, salicylate), toxin (methanol, ethanol), hematological malignancy, severe infection. Anion gap is calculated by = (Na + K) – (Cl + HCO3) in mmol/L.
    Normal value = 8 to 14 mmol/L.
Metabolic acidosis with high anion gap may occur in (i) Diabetic ketoacidosis, (ii) Renal failure, (iii) Lactic acidosis, (iv) Salicylate poisoning.339
Metabolic acidosis with normal anion gap may occur in (i) renal tubular acidosis, (ii) diarrhea, (iii) ureterosigmoidostomy, (iv) acetazolamide therapy, (v) ammonium chloride ingestion.
  • Normal cation in blood—Na+, K, Ca, Mg.
  • Normal anions in blood—Cl, HCO3, albumin, sulfate, lactate, phosphate.
 
Case No. 032
  1. Primary sclerosing cholangitis.
  2. Ultrasonography of hepatobiliary system, ERCP or MRCP.
  3. Liver transplantation.
Note: In long-standing inflammatory bowel disease, when there is biochemical evidence of cholestasis, one should exclude primary sclerosing cholangitis. 75% of primary sclerosing cholangitis is due to inflammatory bowel disease, commonly ulcerative colitis. The frequency of primary sclerosing cholangitis is inversely proportional to the severity of ulcerative colitis. 3 to 10% cases of ulcerative colitis may cause primary sclerosing cholangitis, less in Crohn's disease. Other liver disorders in inflammatory bowel disease are fatty liver, pericholangitis, sclerosing cholangitis, cholangiocarcinoma, cirrhosis of liver, granuloma, abscess, gallstone, etc. P-ANCA is positive in 60 to 80%.
Patient with primary sclerosing cholangitis may be asymptomatic, but may present with advance liver disease. Fatigue, pain in right hypochondrium and pruritus are common complaints. Diagnosis is confirmed by ERCP that shows multiple stricture and dilatations in the intrahepatic biliary ducts. MRCP is another investigation of choice. Antibiotic prophylaxis before instrumentation of biliary tree is mandatory to prevent bacterial cholangitis.
Treatment: Supportive. Cholestyramine for prutirus. Fat-soluble vitamins supplementation, biliary stenting may improve biochemistry and symptoms. However, liver transplantation is the definitive treatment.
 
Case No. 033
  1. Tuberculous peritonitis.
  2. MT, chest X-ray, ascitic fluid analysis (cytology, biochemistry, AFB, PCR), laparoscopy and peritoneal biopsy.
  3. SLE, lymphoma.
    340
Note: Unexplained ascites, weight loss, diffuse abdominal pain, fever, bowel abnormality may be associated with intestinal TB with peritoneal involvement. It may be secondary to pulmonary TB. Ascitic fluid is straw-colored, exudative (high protein and low glucose) and there is high lymphocyte. AFB and PCR may be positive. Laparoscopy may show tubercle which is taken for biopsy.
Treatment: Anti-Koch's therapy, continued for one year. Steroid may be added.
 
Case No. 034
  1. Pancytopenia.
  2. Aplastic anemia, hypersplenism, megaloblastic anemia, aleukemic leukemia.
  3. Pernicious anemia.
  4. Bone marrow study, serum B12, Schilling test, endoscopy and biopsy.
  5. Due to immature red cells.
Note: Anemia with high MCV and pancytopenia is the clue for the diagnosis of pernicious anemia. It is due to absence of intrinsic factor secondary to gastric atrophy resulting in deficiency of vitamin B12. Parietal cell antibodies are present in 90% cases, intrinsic factor antibody present in 60%. Common in elderly above 60 years, more in females and blood group A.
There may be peripheral neuropathy, subacute combined degeneration, optic atrophy, dementia. Carcinoma of stomach in 1 to 3%.
In PBF, macrocytosis with hypersegmented neutrophil may be present. Bone marrow shows megaloblastic changes, reduction of precursor of granulocyte, megakaryocyte. There may be increased iron, bilirubin and LDH.
Pernicious anemia may be associated with vitiligo, Hashimoto's thyroiditis, Graves' disease, Addison's disease, alopecia areata, PBC, chronic active hepatitis, primary ovarian failure.
Treatment: Injection vitamin B12 1000 μgm IM, 5 doses, 2 to 3 days apart, then every 3 months, to be continued for lifelong. Following therapy, hypokalemia and also iron deficiency may occur.
Oral B12, usually 2 mg/day may be given. 1 to 2% is absorbed by diffusion without intrinsic factor. Sublingual B12 may be effective.341
 
Case No. 035
  1. Wernicke's encephalopathy.
  2. Inj. vitamin B1 250 mg IM/IV BD or TDS for 48 to 72 hours. Correction of dehydration and electrolyte imbalance. Add other B-complex.
  3. Prothrombin time, serum vitamin B1, CT scan of brain, endoscopy.
Note: Wernicke's encephalopathy is the acute cerebral manifestation of vitamin B1 deficiency, commonly in long-standing heavy drinking and inadequate diet. Occurs after repeated vomiting, alcoholism, prolonged starvation or diarrhea. Common features are acute confusion, disorientation, drowsiness or altered consciousness. There is bilateral symmetrical ophthalmoplegia, bilateral or unilateral paralysis of lateral conjugate gaze, horizontal or vertical nystagmus, abnormal pupillary reflex, ataxia. Rarely, ptosis, meiosis and unreactive pupil. When associated with memory disturbance and confabulation, it is called Korsakoff's psychosis. CT scan is normal and CSF also normal, but slight rise of protein may occur.
Lesion may be in (i) Brainstem causing ophthalmoplegia, nystagmus and ataxia (ii) Superior vermis of cerebellum causing ataxia (iii) Dorsomedial nucleus in thalamus and adjacent area of grey matter, causing amnesia.
Korsakoff's psychosis is characterized by impairment of memory and confabulation. Loss of recent memory is common, but past memory may be normal.
Confabulation means falsification of memory with clear consciousness. The patient makes new stories unrelated to truth.
Treatment: Injection B1, followed by oral B1 100 mg TDS and other B complex vitamins. If promptly treated, it is reversible. If not treated promptly, lesion may be irreversible.
 
Case No. 036
  1. Paraneoplastic syndrome.
  2. X-ray chest, CPK, EMG, CT or MRI of brain, muscle biopsy.
  3. Treatment of primary cause.
Note: Bizzare neurological presentation, especially in elderly, is highly suggestive of paraneoplastic syndrome. This is characterized by multiple signs and symptoms associated with malignancy unrelated to metastasis. Actual mechanism unknown, causes vary according to the 342type of malignancy. Probable causes are (i) secretion of tumor product usually polypeptide, (ii) autoimmunity—cross reaction between tumor antigen and normal tissue antigen, (iii) release of cytokines (TNF α), (iv) myelitis—commonly in cervical cord.
Features: Varies with primary cause. Some are (i) neurological-neuropathy, cerebellar degeneration, motor neuron disease, myasthenic myopathic syndrome (Lambert-Eaton syndrome), GBS, (ii) musculoskeletal—polymyositis or dermatomyositis, clubbing, hypertrophic osteoarthropathy, (iii) Endocrine SIADH, ectopic ACTH syndrome, hypercalcemia. (iv) cachexia. Most are associated with carcinoma of lung (small cell), ovarian tumor, lymphoma, carcinoma of breast. Paraneoplastic syndrome may precede the clinical presentation of primary carcinoma in 50% cases.
Investigation: X-ray chest or other organ, USG of abdomen, CT/MRI, EMG, CPK, biopsy of muscles. Other investigation according to suspicion of cause.
 
Case No. 037
  1. Polycythemia rubra vera (PRV).
  2. Measurement of red cell mass (increased).
  3. Venesection.
Note: This patient has high hemoglobin and RBC, also high PCV, which suggests polycythemia. There is nothing in the history to suggest secondary causes. High WBC and platelat count associated with splenomegaly are all in favor of polycythemia rubra vera. Differential diagnosis of high hemoglobin and high PCV are polycythemia rubra vera, pseudopolycythemia and also secondary polycythemia. In case of pseudopolycythemia, hemoglobin and PCV are high, but red cell volume is normal (which is high in PRV).
PRV is characterized by increased hemoglobin, RBC, hematocrit, WBC and platelet. Neutrophil leucocytosis in 70%, basophil and platelet in 50%. Also increased LAP (leukocyte alkaline phosphatase), vitamin B12 and uric acid (may cause gout). Bone marrow shows hypercellular with increased megakaryocyte. Abnormal karyotype may be found in bone marrow.
In vitro, culture of marrow demonstrate autonomous growth in the absence of other growth factor.
In secondary polycythemia, only RBC is increased but WBC, platelet and plasma volume are normal. Also, red cell mass is normal.343
Complications: AML, thromboembolism (cerebral, coronary), hyper-tension, gout, peptic ulcer, myelofibrosis.
Polycythemia may be (i) relative due to reduced plasma volume (e.g. dehydration, diuretic) or (ii) true.
True polycythemia may be primary (PRV) or secondary, which may be due to (i) high altitude, cyanotic heart disease, COPD, smoking, (ii) inappropriate and excess erythropoietin secretion (renal cyst, renal cell carcinoma, cerebellar hemangioblastoma, hepatoma, uterine fibroma).
 
Case No. 038
  1. Psoriatic arthritis.
  2. Skin biopsy from lesion, HLA B27.
  3. Methotrexate.
Note: In any patient with skin lesion and ankylosing spondylitis, one should suspect psoriatic arthritis. 7% cases of psoriasis develop arthritis. Types of arthritis in psoriasis. 5 types—(i) Asymmetrical inflammatory oligoarthritis (of hands and foot)—40%. (ii) Symmetrical seronegative polyarthritis (like rheumatoid)—25% (no rheumatoid nodule and involvement of PIP, DIP, MCP joints, nail changes help to diagnose. 50% develop arthritis mutilans). (iii) Sacroilitis or spondylitis—15%. More in male, psoriatic lesion before arthritis and nail changes are usually present. (iv) Predominant DIP joint arthritis—15% (typical), nail dystrophy is invariable. (v) Arthritis mutilans—5%. Skin lesions with nail changes are usually present. Methotrexate, sulfasalazine and azathioprine are helpful both in skin lesion and arthritis. In persistent peripheral arthritis, sulfasalazine, methotrexate or azathioprine may be helpful, but not in axial disease. Retinoid acitretin may be helpful in arthritis and skin lesion.
Other causes of arthritis and skin lesion are Reiter's syndrome, inflammatory bowel disease (there is erythema nodosum, pyoderma gangrenosum).
 
Case No. 039
  1. Blastic crisis, AML.
  2. Splenic infarction.
  3. Bone marrow study.
Note: Blastic crisis may occur in CGL, commonly it causes AML in 70% and ALL in 30%. It is very severe, may be refractory to 344treatment with bad prognosis. Treatment response is relatively better in lymphoblastic than in myeloblastic. Blastic crisis can be suspected when there is rapid deterioration, increasing splenomegaly and blood picture shows increase of blast cells, basophils.
 
Case No. 040
  1. Pernicious anemia, folate deficiency, carcinoma of stomach.
  2. Endoscopy and biopsy, bone marrow study.
  3. Hypothyroidism, chronic liver disease and alcoholism.
Note: Features of anemia with high MCV are suggestive of macrocytic anemia, due to deficiency of B12 or folic acid. Macrocytic anemia may also occur in hypothyroidism, chronic liver disease, alcoholism etc. Bone marrow study is normoblastic in these cases. Thyroid function and liver function tests should be done. Oral B12 may not be effective in deficiency of intrinsic factor in the stomach. Associated folate deficiency should be considered also.
 
Case No. 041
  1. Iron deficiency anemia, β thalassemia minor, sideroblastic anemia.
  2. Hb-electrophoresis and serum ferritin.
Note: Microcytic hypochromic anemia may occur in iron deficiency anemia, β thalassemia, sideroblastic anemia and anemia of chronic disorders. Before giving iron therapy, other conditions should be excluded. In β thalassemia, there is defect in globin synthesis. In other cases, there is defect in heme synthesis.
 
Case No. 042
  1. Euthyroid state.
  2. FT3 and FT4.
  3. Pregnancy, oral contraceptive pill, ostrogen therapy, acute intermittent porphyria.
Note: In this case, patient's symptoms are due to pregnancy. Thyroid gland may be nomally enlarged in pregnancy, so her total T3 and T4 are high, but TSH is normal.
Any causes of high TBG will also cause high T3 and T4, but FT3 and FT4 will be normal. Alternately, any cause of low TBG will also cause low T3 and T4.345
Causes of high TBG: Congenital or hereditary, pregnancy, drug (estrogen, clofibrate, phenothiazine), oral contraceptive pills, acute intermittent porphyria, acute viral hepatitis. Also in hypothyroidism.
Causes of low TBG: Hereditary, malnutrition, nephrotic syndrome, liver failure, drugs (sulfonylurea, salicylate, phenytoin, phenylbutazone, anabolic steroid, androgen, corticosteroid), active acromegaly. Also in hyperthyroidism.
 
Case No. 043
  1. Pseudohyponatremia.
  2. Repeat the test after ether extraction of lipid.
Note: In this case, there is high cholesterol and triglyceride, but the patient has no symptoms, which is the clue for the diagnosis. Pseudohyponatremia may occur in hyperlipidemia or paraproteinemia. Normally, sodium ions are distributed in aqueous phase of plasma, but in these conditions, volume of distribution of sodium ions is lower than the volume of the sample. Ether extract of lipid would give the real result. Also, in true hyponatremia, the patient would have more symptoms and also would be more ill.
 
Case No. 044
  1. Acute pancreatitis, dissecting aneurysm, rupture of esophagus.
  2. X-ray chest, ultrasonogram of hepatobiliary system with pancreas, serum amylase, endoscopy.
Note: Esophageal rupture or perforation may occur following severe violent vomiting or retching, sometimes following alcohol ingestion. It may be associated with subcutaneous emphysema, left-sided pleural effusion or hydropneumothorax. This is called Boerhaave's syndrome. The patient presents with severe chest pain and shock as esophagogastric contents enter into the mediastinum and thoracic cavity. Diagnosed radiologically by barium swallow with water-soluble contrast. Careful endoscopy or CT scan may be done.
Treatment: Surgery. Mortality is very high, if treatment is delayed.
In dissecting aneurysm, there may be unequal pulse, aortic regurgitation, widening of mediastinal shadow in plain X-ray chest. Transesophageal echocardiogram is confirmatory.346
 
Case No. 045
  1. Addison's disease with renal tuberculosis.
  2. Serum morning and midnight cortisol, synacthen test, urine for AFB and mycobacterial C/S.
  3. Pigmentation, blood pressure in standing and lying.
Note: In any patient with weakness, hypotension, associated with low sodium and high potassium, is highly suggestive of Addison's disease.
Addison's disease is the primary adrenocortical insufficiency, due to destruction of adrenal gland, resulting in deficiency in glucocorticoid and mineralocorticoid. There is esthenia or weakness, hypotension and pigmentation. Supine BP may be normal, but marked drop of systolic pressure on standing may occur. Low serum cortisol and high ACTH is highly suggestive of Addison's disease. Causes are autoimmune (80%), tuberculosis. Rarely, hemochromatosis, sarcoidosis, secondary deposit, amyloidosis. Short synacthen test is helpful for diagnosis.
Serum cortisol may be normal or inappropriately low. To find out causes, adrenal autoantibody, plain X-ray abdomen (to see adrenal calcification in TB), USG or CT scan of adrenal should be done.
Treatment: Hydrocortisone 15 mg in the morning and 5 mg in afternoon (1800 hour). Mineralocorticoid may be necessary (9α fluro-hydrocortisone, 0.05 to 0.1 mg daily). Steroid card should be maintained.
 
Case No. 046
  1. Noncirrhotic portal hypertension.
  2. Doppler study of portal vein, venography of portal vein, liver biopsy to exclude cirrhosis.
  3. Splenectomy.
Note: Huge spleen, ascites, normal liver but esophageal varices are highly suggestive of noncirrhotic portal hypertension. In this case, the patient presents with features of portal hypertension and variceal bleeding without cirrhosis or hepatocellular damage, no stigmata of CLD and liver function tests are normal.
Causes: Unknown, but arsenic, vinyl chloride and other toxic agents have been implicated. The liver disease usually does not progress and the prognosis is relatively good.347
 
Case No. 047
  1. Pregnancy.
  2. Reduction of renal threshold.
Note: Patient's symptoms are due to hypoglycemia, following insulin therapy. Antidiabetic therapy should not be started on the basis of urine examination, as there may be renal glycosuria. Reduced renal threshold may be associated with glycosuria, which is common in pregnancy. Also, increased sensitivity to insulin occurs in pregnancy, especially in the first trimester.
 
Case No. 048
  1. Pickwickian syndrome.
  2. Hypothyroidism, sleep apnea syndrome.
  3. Secondary polycythemia.
  4. Type II respiratory failure due to alveolar hypoventilation.
Note: In any patient with obesity, sleep disturbance or sleep apnea with polycythemia, the likely diagnosis is Pickwickian syndrome. It is also called obesity-hypoventilation syndrome. Gross obesity is associated with respiratory failure. In most cases, PaCO2 is normal, but may be high. Airway obstruction may be present, more in smokers. There is reduced respiratory drive. Serum leptin is high. Complications such as pulmonary hypertension and cor pulmonale may occur in Pickwickian syndrome.
Treatment: Weight reduction, smoking should be stopped. Progesterone may be given (which increases respiratory drive).
 
Case No. 049
  1. Sick euthyroid syndrome.
  2. Repeat FT3, FT4 and TSH after cure of pneumonia.
  3. Obesity.
Note: This type of thyroid function may be confused with secondary hypothyroidism. He was always in good health and MRI is normal that indicates normal pituitary. So, the abnormal thyroid function is likely due to sick euthyroid syndrome. In any extrathyroidal illness (acute myocardial infarction, pneumonia, CVD), there may be abnormal thyroid function tests, even the patient is euthyroid, called sick euthyroid syndrome.348
Thyroid functions are affected in several ways (i) Reduced concentration of binding protein and their affinity for thyroid hormones, (ii) Decreased peripheral conversion of T4 to T3, occasionally more rT3 (inactive reverse T3), (iii) Reduced hypothalamic-pituitary TSH production. So, the patient with any systemic illness may have low both total and free T3 and T4. Also, TSH is low or normal. Level usually mildly below normal and thought to be mediated by interleukins (IL-1 and IL-6). The tests should be repeated after recovery of the systemic disease, usually after 6 weeks.
So, biochemical thyroid function should not be done in patient with acute nonthyroidal illness, unless there is good evidence of thyroid disease (such as goiter, exophthalmos) or previous history of thyroid disease.
 
Case No. 050
  1. Factitious hyperthyroidism.
  2. Serum thyroglobulin (low or absent), T4:T3 = 70:1.
Note: Factitious hyperthyroidism due to self-intake of thyroxine may occur in anxious or psychologically upset patient. Clues for the diagnosis are high thyroid hormones, but low radio-iodine uptake. Thyroglobulin level is zero or low and high ratio of T4:T3 = 70:1 (typically, in conventional thyrotoxicosis, the ratio is 30:1). Combination of negligible radio-iodine uptake, high T4:T3 ratio and low or undetectable thyroglobulin is diagnostic.
Causes of high T3 and T4, but low radio-iodine uptake are found in subacute thyroiditis, postpartum thyroiditis, iodine-induced hyperthyroidism and factitious hyperthyroidism.
 
Case No. 051
  1. Malignant hypertension.
  2. Secondary aldosteronism.
  3. Chronic renal failure.
  4. Fundoscopy to see papilledema.
  5. Burr cell.
Note: Malignant hypertension (also called accelerated hypertension) is characterized by high blood pressure, mainly the diastolic >120 mm Hg, with rapidly progressive end organ damage such as retinopathy (grades 3 and 4), renal dysfunction (proteinuria, renal failure) and/or 349hypertensive encephalopathy. Acute LVF may occur. There is fibrinoid necrosis in retinal arteries and arterioles. Unless treated, death may occur due to LVF, renal failure, stroke, aortic dissection. There is high risk of cerebral edema, hemorrhage and encephalopathy. Change in renal artery circulation leads to progressive renal failure, proteinuria and hematuria.
Treatment: Complete bedrest, sublingual nifedipine and oral antihypertensive (atenolol or amlodipine). Blood pressure should be reduced slowly over 24 to 48 hours. If BP is rapidly reduced, there may be cerebral, renal, retinal or myocardial infarction. IV frusemide may be used. In emergency, IV labetolol or glycerine nitrate or Na-nitroprusside or IM hydralazine may be used. Without treatment, 1 year survival is less than 20%. Renal function may be permanently lost in 20%.
 
Case No. 052
  1. Hypothyroidism with pernicious anemia.
  2. TSH, serum vitamin B12 assay, bone marrow study.
  3. Pericardial effusion, anemic heart failure.
Note: In any patient with typical features, cardiomegaly and low voltage tracing, hypothyroidism should be considered. This may be associated with any autoimmune disease. In pernicious anemia, there may be pancytopenia. Bone marrow study will show megaloblastic changes. Upper GIT endoscopy should be done to see gastric atrophy. Schilling test may be done.
 
Case No. 053
  1. Taking the drug irregularly, insufficient dose, took thyroxine just before the test (occasionally, if antithyroid antibody is present, may interfere with the assay of thyroid hormones).
  2. Hypothyroidism still present.
  3. Thyroxine dose should be increased.
Note: In this case, the patient is still hypothyroid. High T4 with high TSH indicates that the patient took thyroxine before the test and still hypothyroid. Presence of antithyroid antibody may also give abnormal result. For follow up, in a case with hypothyroidism, only TSH estimation is sufficient.350
 
Case No. 054
  1. Polymyositis.
  2. Heliotrope rash, proximal myopathy.
  3. CPK, EMG, muscle biopsy.
Note: The patient has muscular weakness with pain and difficulty in deglutition, which are suggestive of polymyositis. Differential diagnosis is polymyalgia rheumatica. Both may present with muscular weakness, polyarthritis and tender muscles. However, high muscle enzyme and positive RA test, ANA and anti Jo-1 are characteristics of polymyositis.
Polymyositis is a connective tissue disease of unknown cause, characterized by nonsuppurative inflammation and necrosis of skeletal muscles. When it is associated with skin rash, it is called dermatomyositis. Cause—unknown, HLA B8/DR3, virus (coxsackie, rubella, influenza) are implicated. Common in female (3:1), 4th or 5th decade, may be associated with malignancy (10%), commonly oat cell carcinoma of bronchus. Heliotrope rash is pathognomonic. May cause renal failure due to myoglobinuria. Pharyngeal, laryngeal and respiratory muscles involvement may lead to dysphagia, dysphonia and respiratory failure. ILD in 30% cases, strongly associated with antisynthetase (e.g. Jo-1) antibody.
RA test, ANA may be positive. Anti Jo-1 may be positive in 30% cases.
CPK is typically high. EMG shows (i) Spontaneous fibrillation (at rest), (ii) Small amplitude, short duration, polyphasic action potential (after voluntary activity), (iii) Spontaneous bizarre, high potency salvos of repetitive discharge (on mechanical stimulation of nerve).
Muscle biopsy shows focal necrosis, regeneration and inflammatory cells infiltration. MRI may be used to detect abnormal muscles.
Diagnostic criteria: (1) Typical clinical history, (2) Increased muscle enzyme (CPK), (3) EMG findings (4) Muscle biopsy.
 
Case No. 055
  1. Chronic granulocytic leukemia with chronic liver disease with portal hypertension.
  2. USG of HBS, endoscopy, bone marrow study.
  3. Blood transfusion, endoscopy with banding or sclerotherapy.
Note: This patient has huge splenomegaly with high WBC count, consistent with the diagnosis of chronic granulocytic leukemia. In 351CGL, patient may be asymptomatic in 25% cases. There may be huge splenomegaly, anemia, repeated infection, bleeding manifestations, etc. Bone marrow is hypercellular with increased myeloid precursors. Philadelphia chromosome is positive in 95% cases. Philadelphia chromosome is negative in older male, which is associated with poor response to therapy, with median survival of <1 year. Blastic crisis may occur, which may be myeloid (70%), or lymphoblastic (30%). Myelofibrosis may occur. CGL may be associated with low LAP score, high uric acid, high vitamin B12 and high LDH.
Treatment: Imatinib is the drug of choice, response in 95% cases.
 
Case No. 056
  1. Nephrogenic diabetes insipidus with hypothyroidism due to lithium.
  2. Water depletion.
  3. Plasma and urine osmolality, serum ADH, serum lithium.
Note: Patient with polyuria, but normal blood glucose, diabetes mellitus is excluded. Other differential diagnoses are cranial diabetes insipidus, nephrogenic diabetes insipidus and compulsive water drinking. The patient was on lithium. Likely diagnosis is nephrogenic diabetes insipidus, due to lithium. In this condition, plasma osmolality is high, but urine osmolality is low which is not affected by desmopressin. In cranial diabetes insipidus, where plasma osmolality is high and urine osmolality is low, that increases following desmopressin therapy, Toxicity of lithium may occur early, as therapeutic window is narrow. 95% is excreted in glomerulus, followed by tubular reabsorption in competition with sodium. During sodium depletion, more lithium is absorbed and in sodium excess, more lithium is lost. Toxicity is increased during diarrhea, vomiting, sweating, etc. Lithium level >2.0 mmol/L is associated with toxicity. Lithium may cause NDI, hypothyroidism, hyperglycemia, hyperparathyroidism, neurological complication (dysarthria, nystagmus, coarse tremor, seizure, hyperreflexia), renal failure and even collapse.
Treatment: The drug should be stopped, replacement of water and salt. Hemodialysis, if lithium level >3.5 mmol/L.
 
Case No. 057
  1. Hypokalemic hyperchloremic metabolic acidosis.
  2. Distal renal tubular acidosis.
    352
  3. pH of overnight urine, blood pH, ammonium chloride load test, plain X-ray abdomen (to see nephrocalcinosis).
  4. Osteomalacia.
Note: Biochemical abnormality is hypokalemic, hyperchloremic acidosis, also low calcium and phosphate. These are consistent with renal tubular acidosis, which is characterized by severe metabolic acidosis associated with failure to acidify urine either due to defect in excretion of hydrogen ion by distal tubule or due to failure of absorption of bicarbonate by proximal tubule. There is failure of acidification of urine despite severe metabolic acidosis.
RTA is of 4 types:
Type-1: Distal tubular acidosis—common. There is acidosis, hypokalemia and inability to lower urine pH<5.3, despite severe acidosis. Causes are congenital (AD, AR or sex linked), autoimmune disease (Sjogren's syndrome, chronic active hepatitis, primary biliary cirrhosis, SLE), drugs (amphotericin B, lithium, NSAID, lead), amyloidosis, cryglobulinemia, obstructive uropathy, renal transplant rejection.
Treatment: Sodium bicarbonate, potassium supplement and treatment of underlying cause.
Type-2: Proximal tubular acidosis—common in children. Causes are congenital (AD), cystinosis, Wilson's disease, tyrosinemia, glycogen storage disease type 1, multiple myeloma, hyperparathyroidism, drugs (degraded tetracycline, carbonic anhydrase inhibitor).
Treatment: Sodium bicarbonate in high dose, potassium supplement and treatment of underlying cause.
In PTA, there is proximal tubular defect resulting in aminoaciduria, glycosuria, phosphaturia, called Fanconi syndrome.
Type-3: Combined proximal and distal tubular acidosis (rare).
Type-4: Also called hyporeninemic hypoaldosteronism.
Patient with distal RTA may present at any age. In children, the patient presents with failure to thrive. Adult patient presents with renal colic, muscular weakness due to hypokalemia, osteomalacia, etc. Patient with proximal RTA presents with features of acidosis, polydipsia, polyuria, hypokalemic myopathy and ricket or osteomalacia. Diagnosis is suspected in any patient with hyperchloremic acidosis and can be corfirmed by early morning urinary pH >5.5. In patient in whom the diagnosis is suspected, but no acidosis is present, an acid load test using ammonium chloride is done.353
 
Case No. 058
  1. Parasagittal meningioma.
  2. Normal pressure hydrocephalus.
  3. CT scan of brain, MRI.
Note: The patient has spastic paraplegia, more likely cause here is cerebral, which may be due to parasagittal meningioma (Falx meningioma), thrombosis of superior longitudinal sinus, thrombosis of unimpaired anterior cerebral artery, trauma. In childhood, commonest cause is cerebral palsy. No sensory loss, which indicates no spinal cord compression.
 
Case No. 059
  1. Nephrotic syndrome due to rheumatoid arthritis.
  2. Amyloidosis.
  3. Renal biopsy or rectal biopsy.
Note: Amyloidosis is a complication of long-standing rheumatoid arthritis, which may present as nephrotic syndrome, chronic renal failure. The protein comprising amyloid is protein AA, which occurs in rheumatoid arthritis and other inflammatory conditions like inflammatory bowel disease, bronchiectesis, familial mediterranean fever, etc. This type of amyloid has a predilection for kidney, causing nephrotic syndrome, hematuria and renal failure. Usually normotensive, kidney may be enlarged and palpable. Splenomegaly, hypersplenism may occur. Rectal biopsy or involved organ biopsy is necessary for diagnosis. Amyloid stains red with Congo red and shows apple green birefringence under polarized light. Immunohistochemical staining can identify the type of amyloid fibril.
 
Case No. 060
  1. Paroxysmal nocturnal hemoglobinuria (PNH).
  2. Ham's acid serum test, flow cytometry to demonstrate CD 59 or RBC.
  3. Deep venous thrombosis.
  4. Due to reticulocyte.
Note: PNH is an acquired clonal abnormality of red cells, which are destroyed by activated complement resulting in intravascular hemolysis. Also, platelet and granulocyte are involved, resulting in thromhocytopenia and leukopenia. Commonly, the urine voided at 354night and in the morning on waking is dark-colored. Hemolysis may be precipitated by infection, surgery or iron therapy. Ham's acid serum test may be positive. There is tendency to thrombosis involving the mesenteric, portal (Budd-Chiari syndrome) or cerebral veins, also calf muscles. Cause of thrombosis is unknown, probably complement mediated activation of platelet deficient in CD55 and CD59, resulting in hypercoagulability. Aplastic anemia may precede PNH in 25% cases and acute myeloid leukemia may occur. Other complications in PNH are increased susceptibility to infection, iron deficiency, pigment gallstone, etc.
Investigation: Flow cytometric analysis of red cells with CD55 and CD59 has replaced Ham's test. Bone marrow may be hypoplastic or aplastic.
Treatment: Supportive (blood transfusion, iron), prednisolone (in some cases). In marrow failure, antithymocyte globulin, cyclosporin or bone marrow transplantation. Recently, a recombinant humanized monoclonal antibody (eculizumab) may be helpful. Gene therapy. Survival—10 to 15 years.
 
Case No. 061
  1. Microscopic polyangitis, Wegener's granulomatosis, SLE (other Churg-Strauss syndrome).
  2. P-ANCA, C-ANCA, ANA, anti-ds DNA, renal biopsy.
Note: In any disorder involving multiple systems of the body, always think of the above diagnosis. In Wegener's granulomatosis, the patient may consult with ENT or eye specialist. In chest, there may be migrating shadows, which appear or disappear that is seen in serial chest X-ray film.
Renal involvement may occur specially in microscopic polyarteritis for which renal biopsy may be required.
 
Case No. 062 (see also Case No. 45)
  1. Addison's disease, salt loosing nephropathy.
  2. Morning and midnight cortisol, synacthen test, serum ACTH, 24 hours urine for sodium.
  3. Postural hypotension.
Note: Hyponatremia and hyperkalemia are typical of Addison's disease. Only hyponatremia is more important. Glucose may be in lower limit 355of normal. Low cortisol and high ACTH is highly suggestive. For investigation, first of all, test should be done to confirm Addison's disease, then other test to find out the causes. In some cases, high TSH and low T4 may occur without hypothyroidism. With steroid, both TSH and T4 revert to normal. Addisonian crisis may occur following surgery, trauma, infection, any major illness. Acute abdomen may occur. Other autoimmune diseases may be associated such as pernicious anemia, IDDM, vitiligo, primary ovarian failure, PBC, etc.
 
Case No. 063
  1. Cushing's syndrome, primary aldosteronism, ectopic ACTH syndrome.
  2. Cushing's syndrome.
  3. Obesity, striae, plethoric face, skin is thin with bruise, proximal myopathy.
  4. Serum morning and midnight cortisol, dexamethasone suppression test, ultrasonography of abdomen, CT scan or MRI of suprarenal glands.
Note: Hypertension with hypokalemia and diabetes mellitus is highly suggestive of Cushing's syndrome. In primary aldosteronism, diabetes mellitus is not common and hypokalemia is more severe.
To diagnose Cushing's syndrome, serum morning and midnight cortisol, 24 hours urinary-free cortisol, short overnight dexamethasone suppression test should be done, first to see whether Cushing's syndrome is present or not. Then further test should be done to find out causes (e.g. ACTH, X-ray chest, USG of suprarenal gland, CT or MRI of suprarenal glands and pituitary gland).
Pseudo-Cushing's may occur in depression, obesity and alcoholism. In these cases, there is increased urinary excretion of steroid, absent diurnal variation of cortisol and failure of suppression by dexamethasone. To differentiate from Cushing's syndrome, insulin-induced hypoglycemia is helpful. In Cushing's syndrome, almost no response, but in pseudo-Cushing's syndrome, there is excess cortisol secretion.
 
Case No. 064
  1. Aplastic anemia, aleukemic leukemia.
  2. Bone marrow study.
  3. Blood transfusion.
    356
Note: Blood picture indicates pancytopenia. Causes are aplastic anemia, aleukemic leukemia, megaloblastic anemia, hypersplenism, PNH. Cause may be found by bone marrow study.
Other investigation should be done according to the findings in bone marrow.
 
Case No. 065
  1. Primary biliary cirrhosis.
  2. Pruritus.
  3. Antimitochondrial antibody.
  4. Liver biopsy.
  5. Orthotopic liver transplantation.
Note: Any patient presenting with pigmentation, cholestatic jaundice and hepatosplenomegaly, primary biliary cirrhosis is the likely diagnosis. Pruritus may precede the onset of jaundice by many years. PBC is an autoimmune disease, characterized by granulomatous destruction of interlobular bile ducts, inflammatory damage with fibrosis, cholestasis and eventual cirrhosis. Common in females, 40 to 60 years. Cause unknown, immunological mechanism plays a part. Environmental factor acts in genetically predisposed person. E. coli and other enterobacteria, retrovirus are probable triggering factors. Hypercholesterolemia is due to reduced biliary excretion, because of biliary obstruction. Xanthelasma and palmar xanthoma may be present, Osteomalacia is due to vitamin D deficiency resulting from fat malabsorption.
Antimitochondrial antibody is positive in 95% cases, antismooth muscle antibody is positive in 50%, antinuclear antibody is positive in 20%. There are 4 types of antimitochondrial antibody—M2 (specific for PBC), M4, M8 and M9.
Liver biopsy—infiltration of plasma cells and lymphocytes in portal tracts, granuloma (40%), small bile duct destruction with ductal proliferation, piecemeal necrosis and cirrhosis.
Hepatic granulomas are found in PBC, sarcoidosis, TB, schistosomiasis, drug reaction, brucellosis, parasitic (strongyloidosis).
 
Case No. 066
  1. Pheochromocytoma, drugs, food, clonidine withdrawal.
  2. Urine for metanephrine, USG or CT scan of adrenal glands.
Note: VMA is a breakdown product of catecholamine. It is usually high in pheochromocytoma. Other causes are drugs (methyldopa, MAO 357inhibitor, phenothiazine, tetracycline), foods (tea, coffee, chocolate, banana, ice-cream, vanilla), clonidine withdrawal. Before performing VMA measurement, above drugs and food must be stopped. Also, antihypertensive drug should be stopped. 24 hours urine is collected for VMA measurement.
 
Case No. 067
  1. Guillain Barre syndrorme, Froin's syndrome, carcinomatous neuropathy, bacterial meningitis.
  2. EMG, X-ray of vertebra, MRI of vertebral column.
Note: In this case, CSF shows high protein, but normal cells and also glucose. Very high protein in CSF occurs in Guillain Barre syndrome, Froin's syndrome, carcinomatous neuropathy, acoustic neuroma, meningitis (tuberculous, acute pyogenic, fungal).
 
Case No. 068
  1. Infectious mononucleosis with hepatitis.
  2. Paul-Bunnel test, antibody to EB virus, monospot test.
Note: Features of infections, hepatosplenomegaly, jaundice, high reticulocyte, low platelet are consistent with the diagnosis of infectious mononucleosis. This disease is caused by Epstein Barr virus. This virus infects and replicates in B lymphocytes. Transmission through oral contact. There is lymphadenopathy, splenomegaly, petechial rash in palate. Skin rash specially after ampicillin or amoxycillin occurs in 90% cases. In the peripheral blood, atypical lymphocyte is commonly found. False positive Paul-Bunnel test may be found in viral hepatitis, hydatid cyst and acute leukemia.
Causes of atypical lymphocyte are cytomegalovirus infection, toxoplasmosis, acute HIV or any viral infection can cause.
Complications: Commonly chronic fatigue syndrome. Rarely, mild hepatitis, thrombocytopenia, meningoencephalitis, myocarditis, hemolytic anemia, rupture of the spleen, etc.
Treatment: Symptomatic. Steroid may be indicated in encephalitis, meningitis, GBS, thrombocytopenia, hemolysis.
 
Case No. 069
  1. Macrocytic anemia.
  2. Bone marrow study.
    358
  3. Hypothyroidism, chronic liver disease, chronic alcoholism, (others are hemorrhage, hemolysis, azathioprine therapy).
  4. Thyroid functions and liver function tests.
Note: High MCV with macrocytosis indicates macrocytic anemia. Commonly, it is due to B12 or folic acid deficiency. MCV >110 is usually associated with megaloblastic bone marrow. Macrocyte is found in the peripheral blood and megaloblast in the bone marrow. Macrocytic anemia may be found with either megaloblastic or normoblastic bone marrow.
Causes of macrocytosis with megaloblastic marrow—vitamin B12 or folic acid deficiency.
Causes of macrocytosis with normoblastic marrow—chronic liver disease, hypothyroidism, chronic alcoholism, hemolysis, azathioprine therapy, etc. In such cases, liver function tests and thyroid function tests should be done.
Hypothyroidism itself is associated with high MCV. But in 10% cases, autoimmune hypothyroidism may be associated with pernicious anemia. Drugs causing high MCV or macrocytosis are methotrexate, azathioprine, hydroxyurea, zidovudine.
 
Case No. 070
  1. Hemolytic anemia.
  2. Coomb's test, lymph node biopsy, bone marrow study.
  3. Hodgkin's disease.
Note: High reticulocyte with high bilirubin, but normal liver enzymes indicates hemolytic anemia. In Hodgkin's disease, anemia may be due to autoimmune hemolytic anemia or bone marrow infiltration which is associated with leukoerythroblastic blood picture. Hallmark in lymph node biopsy is presence of Reed Sternberg giant cell. Common age is adolescence and also after 50 years (bimodal). Pruritus may occur in 10% cases and Pel-Ebstein fever occurs in 10% cases. Before starting treatment, staging should be done, which will guide to therapy and prognosis.
 
Case No. 071
  1. Gonococcal arthritis, reactive arthritis (or septic arthritis).
  2. Aspirated fluid for Gram stain and special culture for gonococcus, urine C/S, blood C/S, high vaginal swab for C/S.
    359
Note: In septic arthritis, there is high fever, leukocytosis, aspirated fluid shows high leukocyte, may be >50,000/cmm, glucose low and protein high.
Gonococcal arthritis usually involves large joints. Effusion may be present, associated with high fever and tenosynovitis involving the wrist, hand, feet and Achilles tendon. Skin rash is maculopapular at first, then pustular, then hemorrhagic and finally necrotic. History of urethritis, high fever, skin rash, arthritis of 1 or 2 joints, or tenosynovitis in a young person is highly suggestive of gonococcal arthritis. Routine C/S is negative.
Gonococcus is gram-negative intracellular diplococcus. Organism is isolated by culture from urethral swab in man, and endocervical swab in woman, also from synovial fluid, blood, throat or skin lesion. In female, there is less symptoms and may act as a carrier. Gonococcal organism is difficult to isolate by culture. In disseminated gonococcal infection, organism is isolated from blood culture in <20%, joint aspirate 50% and from skin lesion 5 to 10%.
Causes of monoarthritis in young patient are trauma, septic, tuberculous, reactive arthritis, hemophilia, other seronegative arthritis.
 
Case No. 072
  1. Restless leg syndrome (RLS) due to CRF.
  2. Correction of iron deficiency, erythropoietin, dialysis must be proper, clonazepam.
Note: Restless leg syndrome (also called Ekbom's syndrome) is a common complication in chronic renal failure. It may occur in iron deficiency anemia. The patient has irresistible movement of limb.
RLS is a neuromuscular abnormality, characterized by discomfort or abnormal sensation in the calf or feet requiring irresistible and frequent movement of the affected limb. It is worse in the evening or at the onset of sleep at night, interfering with sleep. Commonly involves the lower limbs, may also involve upper limbs. Exaggerated by pregnancy, inactivity, caffeine, sleep disturbance, etc. It is common in normal population, 1 to 5% (average 2%) cases, in middle age. But frequency increases up to 20 to 30%, if occurs after 60 years. May be familial, in 1/3rd cases, multiple members in the family may be affected, occasionally inherited as autosomal dominant. When RLS is associated with iron deficiency anemia and CRF, it is called secondary RLS.360
Treatment: Clonazepam (0.5 to 2.0 mg), levodopa (100 to 200 mg), dopamine agonist (pramipexole or ropinirole). Narcotics, benzodiazepine and anticonvulsant may be helpful. Treatment of primary cause should be done.
In CRF, correction of iron deficiency, proper dialysis, erythropoietin for correction of anemia. Kidney transplantation may cure the condition. Clonazepam and codeine phosphate may be helpful.
 
Case No. 073
  1. Renal cell carcinoma with metastasis in liver and cervical lymph nodes.
  2. Renal tuberculosis (or disseminated tuberculosis).
  3. Hematuria and pain in the flank.
  4. Renal mass.
  5. Urine for malignant cells, urine for AFB, MRI of renal system.
Note: Renal cell carcinoma or hypernephroma commonly presents with triad of painless hematuria, loin pain or heaviness and palpable mass in loin. Hematuria is present in 50% cases. In 20% cases, PUO may be the only manifestation, due to secretion of pyrogen by tumor. Usually, there is normocytic and normochromic anemia. Neoplastic cells may produce peptide hormones such as erythropoietin, renin, ADH, PTH-related peptide. There may be polycythemia due to excess erythropoietin, hypercalcemia due to bony metastasis or secretion of parathormone-like substance. Hypertension, hypokalemia may occur. The tumor is adenocarcinoma arising from proximal tubular epithelial cells, common in elderly, males affected twice more than females. In 10% cases, hypernephroma may be bilateral. Diagnosis is done by ultrasonography, IVP, CT scan or MRI.
Treatment: Radical nephrectomy that includes perirenal fascial envelope and ipsilateral para-aortic lymph nodes is done, if possible. Chemotherapy or radiotherapy is not helpful. Some benefit may be found with immunotherapy using interferon and interleukin-2.
 
Case No. 074
  1. Thrombotic thrombocytopenic purpura, hemolytic uremic syndrome.
  2. Renal biopsy.
  3. Plasma exchange with transfusion of fresh-frozen plasma.
Note: Any patient presenting with fever, thrombocytopenia, microangiopathic hemolytic anemia and renal failure, differential 361diagnoses are thrombotic thrombocytopenic purpura and hemolytic uremic syndrome. The two conditions are probably the part of the same disorder. TTP is a disorder of unknown etiology characterized by fever, microangiopathic hemolytic anemia, thrombocytopenia, neurological signs and renal failure. Common in young females. Neurological features are common in TTP, which includes headache, seizure and coma. Neurological feature is absent in HUS.
Diagnostic pentad in TTP—(i) fever, (ii) thrombocytopenia, (iii) microangiopathic hemolytic anemia, (iv) renal failure, (v) neuro-logical feature.
Pathophysiology of TTP—initially endothelial damage, cell swelling, platelet adherence and thrombosis. There is severe microangiopathic hemolytic anemia with thrombocytopenia.
Renal disease is due to hyaline occlusion of capillaries and arterioles and proliferative change in glomeruli. Microscopically, there is microvascular hyaline thrombi. Thrombocytopenia is invariable. Also, there is increased reticulocytes, presence of fragmented RBC (schistocyte), prolonged bleeding time, raised bilirubin and LDH, reduced haptoglobin. Usually no DIC.
Treatment – Plasma exchange plus fresh-frozen plasma infusion. Steroid may be used in some cases. Platelet transfusion is avoided, as it may aggravate the disease. Mortality 90% in untreated, 10 to 30% in treated cases.
 
Case No. 075
  1. Acute interstitial nephritis with ARF due to cefixime.
  2. USG of kidney, renal biopsy.
  3. Dialysis, prednisolone.
  4. Renal biopsy.
  5. Offending drug should be stopped.
Note: This patient presents with fever, skin rash, eosinophilia and renal impairment shortly after antibiotic therapy. These findings are consistent with acute interstitial nephritis. It is an acute inflammation of tubular interstitium, probably due to hypersensitivity reaction. It is characterized by fever, arthralgia, skin rash, bodyache and renal failure. There is also oliguria, salt and potassium loss, proteinuria, high eosinophil in the blood.
In urine, leukocyturia is common and eosinophils are high in 70%. Oliguria may be absent, despite moderately severe ARF.362
Renal biopsy shows infiltration of eosinophil, polymorph and lymphocyte surrounding the tubules and blood vessels. Tubular necrosis may be seen. In some patients, linear deposition of IgG and C3 along the tubular basement membrane may be seen.
Causes are (i) drugs (70%)- penicillin and NSAID commonest. Also, sulfonamide, allopurinol, cephalosporin, rifampicin, phenytoin, phenindione, diuretic (frusemide, thiazide), (ii) others—autoimmune, infection 15% (acute bacterial pyelonephritis, tuberculosis, CMV and hantavirus, leptospirosis), multiple myeloma, idiopathic 8%, tubulointerstitial nephritis with uveitis (TINU) 5%.
Treatment: (i) Prednisolone 60 mg/day, (ii) Offending drug should be stopped, (iii) Correction of electrolytes, (iv) Dialysis may be required.
 
Case No. 076
  1. Bone marrow study, serum protein electrophoresis.
  2. Waldenstrom's macroglobulinemia.
  3. Plasmapheresis for hyperviscocity, chlorambucil or fludarabine.
Note: Waldenstrom's macroglobulinemia is a low grade lympho-plasmocytoid lymphoma, producing large quantities of IgM paraprotein. This leads to hyperviscosity syndrome (70%) as well as suppression of normal bone marrow activity. Plasma viscocity is high and protein electrophoresis shows IgM band.
Bone marrow shows lymphoid cells and prominent mast cells.
No lytic lesion in bone or no renal involvement (which are common in myeloma). Bence Jones protein in urine occurs in 70% cases. Common in elderly, male slightly more than female. Median survival—5 years.
Causes of Bence Jones proteinuria—multiple myeloma, Waldenstroms macroglobulinemia, benign monoclonal gammopathy, heavy chain disease, non-Hodgkin's lymphoma, CLL, amyloidosis.
 
Case No. 077
  1. Hemosiderosis with CCF.
  2. Anemia and cardiomyopathy due to hemosiderosis.
  3. Serum ferritin.
  4. Desferrioxamine.
Note: In hereditary hemolytic anemia, hemosiderosis may occur following repeated blood transfusion. Also, hemolysis may be a contributing factor. Iron is deposited in different parts of the body, 363especially in the heart causing cardiomyopathy, arrhythmia, heart failure, etc. Chelating agent such as desferrioxamine should be used to prevent this complication.
 
Case No. 078
  1. Relapsing polychondritis.
  2. ANA, biopsy of nasal or ear cartilage.
  3. Prednisolone 40 to 60 mg/day.
Note: Relapsing polychondritis is an inflammatory disorder of unknown cause involving the cartilage, characterized by recurrent chondritis involving the cartilage of ear, nose, larynx and trachea. There is pain and redness of ear with progressive deafness, recurrent nasal involvement leads to saddle nose. Eye involvement causing conjunctivitis, episcleritis, scleritis, iritis. In heart, may be aortic regurgitation due to dilatation of aortic ring or destruction of valve cusp. Oral or genital ulcer may occur. Trachea and larynx involvement leads to hoarseness of voice, even stenosis may occur with difficulty in breathing. Acute focal glomerulonephritis may be found. This disease involves equally both male and female, common in elderly.
Investigations: Mild leukocytosis, very high ESR, RA factor and ANF may be positive. X-ray may show calcification of cartilage of nose, ear and trachea.
Diagnosis: Mainly by the history. Cartilage biopsy will show reduction of chondrocyte and infiltration of inflammatory cells.
Treatment: Prednisolone 40 to 60 mg/day. Dose is reduced with improvement. If no response, cyclophosphamide or azathioprine or ciclosporin may be helpful. Reconstructive surgery for deformity of ear, nose or trachea.
 
Case No. 079
  1. Leukoerythroblastic anemia.
  2. Myelofibrosis.
  3. Tear drop poikilocyte.
  4. Bone marrow study and LAP score.
Note: In any elderly patient with huge splenomegaly and high WBC, differential diagnoses are myelofibrosis and CML. Myelofibrosis is a disorder of unknown cause, characterized by bone marrow fibrosis, extramedullary hemopoiesis and leukoerythroblastic blood picture, due to neoplastic proliferation of primitive stem cells. It is common above 36450 years. There may be peptic ulcer, pruritus after hot bath, gout, etc. May transform to AML in 10 to 20% cases.
LAP score and uric acid are high.
Bone marrow study may be dry tap. Trephine biopsy is needed which shows increased megakaryocyte, increased reticulin and fibrous tissue. Initially, bone marrow is hypercellular, later fibrosis. Fibrosis is due to excess release of fibroblast stimulating factors from abnormal megakaryocyte, such as platelet-derived growth factor.
Treatment: Blood transfusion, folic acid. If leukocytosis, hydroxy-carbamide (hydroxyurea) may be given. In young patient, bone marrow transplantation.
Radiotherapy for huge spleen. If evidence of hypersplenism and huge spleen with pressure symptoms, splenectomy may be necessary.
Median survival—4 years (1 to 20 years).
Causes of death: Cardiovascular disease, infection or GIT bleeding.
Huge splenomegaly and very high WBC may be found in both myelofibrosis and CGL, may be difficult to differentiate clinically. Differentiating points are:
Points
DKA
Lactic acidosis
1. Blood film
High granulocytes of different stages
Leukoerythroblastic, tear drop poikilocytes
2. LAP score
Low
High
3. Philadelphia chromosome
Present in 90%
Absent
4. Bone marrow
Hypercellular with myeloid precursors
Fibrosis, stains black with reticulin
 
Case No. 080
  1. Amiodarone therapy.
  2. FT3, FT4.
Note: The patient has biochemical abnormality of thyroid function and he was on anti-arrhythmic drug, likely to be amiodarone toxicity. Long-term use of amiodarone may cause hyperthyroidism, hypothyroidism, goiter, etc. It inhibits the conversion of T4 to T3 in the periphery by blocking deiodinase activity. This results in low T3, but high T4.
Half-life of amiodarone is 28 days (may be 25 to 110 days). It does not impair ventricular function, so can be given in heart failure. This 365drug has delayed onset of action of 3 to 7 days, may take 50 days for maximum effects. Toxicity of amiodarone are corneal microdeposit, skin (slaty-grey pigmentation, hypersensitivity), thyroid (hypo- and hyperthyroidism), lung (allergic alveolitis, pulmonary fibrosis), GIT (nausea, vomiting, metallic taste), hepatitis, heart (prolonged QT, prominent U wave, torsades de pointes tachycardia). Others are neuropathy, myopathy, tremor. It potentiates the action of digoxin and warfarin. May promote digoxin toxicity by competing for digoxin binding sites, so increases plasma digoxin level, dose of digoxin should be halved when amiodarone is given.
Amiodarone may cause prolongation of QT, which may be associated with ventricular tachycardia (or Torsades de pointes). In such cases, amiodarone should be stopped and IV magnesium should be given.
Patient on long-term amiodarone therapy should have an annual thyroid test, LFT and Kco estimation.
 
Case No. 081
  1. Dimorphic anemia.
  2. Combined iron and B12 or folic acid deficiency, sideroblastic anemia, partial correction of anemia.
  3. Serum iron and TIBC, serum B12, folic acid, bone marrow to see ring sideroblast.
Note: Combination of macrocytic and microcytic anemia is called dimorphic anemia. Commonly, it is due to deficiency of iron, B12 or folic acid. When anemia is treated, dimorphic picture may be found in the early stage. Sideroblastic anemia may also show dimorphic picture.
 
Case No. 082
  1. Fundoscopy to see papilledema.
  2. Benign (idiopathic) intracranial hypertension.
  3. Iatrogenic Cushing's syndrome.
Note: BIH, also called idiopathic intracranial hypertension (IIH), is defined as symptoms of raised intracranial pressure without space occupying lesion or ventricular dilatation or focal neurological sign. It is common in young females, 18 to 40 years, obese, rarely familial. BIH may be associated with pregnancy, obesity, oral contraceptive pill, hypo- or hyperthyroidism, adrenal insufficiency, steroid use or 366withdrawal, drugs (sulfur, nitrofurantoin, nalidixic acid, tetracycline), hypervitaminosis A. The patient usually presents with frequent headache and visual disturbance. 6th nerve palsy may be seen (false localizing sign). Usually no epileptic attack. Visual loss may occur due to optic atrophy. Lumbar puncture shows high CSF pressure, no other abnormality.
CT scan of brain is normal with no ventricular dilatation. MR angiography or cerebral venography may be done to exclude cerebral venous sinus thrombosis.
Actual cause of BIH is unknown, likely mechanism is reduction in the reabsorption of CSF by the arachnoid villi.
Treatment: (i) Weight reduction, (ii) avoid offending drugs, (iii) loop diuretics or acetazolamide may be given, (iv) repeated lumbar puncture, (v) occasionally, steroid may be used (it reduces intracranial pressure).
Surgical treatment: Ventriculoperitoneal shunt, specially if there is progressive visual loss. Optic nerve fenestration may be done.
 
Case No. 083
  1. Acute cold hemagglutinin disease.
  2. Hemoglobinuria.
  3. Coomb's test and measurement of cold antibody (IgM).
Note: Cold hemagglutinin disease (CHAD) is characterized by hemolytic anemia due to auto-antibody that acts as red cell agglutinin at low temperature. Hemolysis is due to cold auto-antibody that reacts with red cells at temperature below 37°C (optimum 0 to 4°). Antibody is IgM type, polyclonal.
CHAD may be primary or secondary. Primary type occurs in adults, specially above 50 years and is rare in children. It occurs equally in both sexes. Secondary type occurs transiently in infectious mononucleosis, cytomegalo virus infection, mycoplasma pneumoniae. Also, rarely in lymphoma, chronic lymphatic leukemia, SLE, Waldenstrom's macroglobulinemia.
Postinfectious CHAD is usually self-limiting, resolves in 2 to 3 weeks.
 
Case No. 084
  1. Plummer-Vinson syndrome.
  2. Koilonychia.
  3. PBF to see microcytic hypochromic blood picture.
    367
  4. Serum iron, TIBC, ferritin.
Note: Combination of iron deficiency anemia, dysphagia and glossitis is called Plummer-Vinson syndrome (also called Paterson-Brown-Kelly syndrome). It is common in women, 4th to 6th decade, cause is unknown. There is constriction in upper esophageal sphincter in the postcricoid region and appears radiologically as a web. This web may be asymptomatic or may produce dysphagia. It may be difficult to see endoscopically. It is a premalignant condition, can predispose to carcinoma of esophagus, which needs to be excluded by endoscopy and biopsy. Blood picture will show features of iron deficiency anemia.
Treatment: Iron therapy. Rarely, dilatation may be required. If severe anemia, blood transfusion. Rarely, there may be squamous cell carcinoma.
 
Case No. 085
  1. Dissociative (conversion) disorder.
  2. Familial periodic paralysis.
  3. Serum electrolytes, mainly to see potassium.
Note: In young female patient, functional disorder is a very important differential diagnosis. Here, there is no neurological abnormality, which is the clue for the diagnosis. In periodic paralysis, muscles are extremely weak with hypotonia and loss of reflexes. Sometimes, in dissociative disorder, there is rigidity, which increases more and more during more maneuver.
The term dissociative (conversion) disorder has replaced ‘hysteria’. It is a disorder characterized by profound loss of awareness or cognitive ability without organic disease. Conversion means unresolved conflict is converted into symbolic physical symptoms as a defense against it, e.g. paralysis, sensory loss, abnormal movements, aphonia, gait disturbance. Dissociate means disintegration of different mental activity, e.g. amnesia, fugue, pseudoseizure.
 
Case No. 086
  1. Hereditary fructose intolerance.
  2. Measurement of glucose, after oral or I/V fructose (will show hypoglycemia) and urine for fructose (fructosuria).
  3. To avoid fructose-containing diet.
    368
Note: Hereditary fructose intolerance is an autosomal recessive disease. Symptoms develop after ingestion of fructose-containing diet. In infant, there is vomiting, retardation of growth, hypoglycemia, jaundice, hepatosplenomegaly, ascites, aminoaciduria, hyperuricemia, acidosis, etc.
Treatment: Avoid fructose-containing diets, e.g. fruits.
 
Case No. 087
  1. Euthyroid Graves' disease.
  2. TSH receptor antibody, orbital ultrasonography, CT scan of orbit.
  3. Graves' ophthalmopathy.
Note: This patient has been suffering from Graves' disease, who is clinically and biochemically euthyroid. One should remember that the natural history of Graves' disease is hyperthyroidism, followed by euthyroid and later, hypothyroidism. Commonest cause of bilateral exophthalmos is Graves' disease, which may or may not be associated with other features such as diffuse goiter, dermopathy. Antimicrosomal (antiperoxidase) and antithyroglobulin antibody may be present in low titer. TSH receptor antibody (TR Ab) is high.
Causes of bilateral exophthalmos are Graves' disease, cavernous sinus thrombosis, caroticocavernous fistula, craniostenosis, hypertelorism, severe myopia, Hand-Schuller-Christian disease.
 
Case No. 088
  1. Acute calculus cholecystitis, acute pancreatitis, splenic infarction.
  2. Ultrasonography of abdomen, serum amylase.
Note: In hereditary hemolytic anemia, there may be pigment gallstone formation. There is splenomegaly and splenic infarction may occur, especially in sickle cell anemia in which repeated infarction may cause autosplenectomy by the age of 16 years. Gallstone may be associated with acute pancreatitis.
 
Case No. 089
  1. Hyperchloremic hypokalemic metabolic acidosis with normal anion gap.
  2. Distal renal tubular acidosis, ureterosigmoidostomy, acetazolamide therapy.
  3. Distal renal tubular acidosis.
  4. Hypokalemia.
369Note: Usually, hypokalemia is associated with alkalosis. In some cases, hypokalemic acidosis may occur. Hyperchloremic hypokalemic metabolic acidosis with normal anion gap is common in distal RTA.
Anion gap is measured by (Na + K) – (Cl + HCO3). Here it is (136 + 3.1) – (110 + 14) = 15.1. Normal anion gap is 8 to 15.9. Other cause is bicarbonate loss in severe persistent diarrhea.
 
Case No. 090
  1. Goodpasture's syndrome, microscopic polyarteritis, Wegener's granulomatosis, SLE.
  2. P-ANCA and C-ANCA, anti-GBM antibody, kidney biopsy.
  3. Goodpasture's syndrome.
Note: In young patient with renal and lung involvement, commonest cause is Goodpasture's syndrome. Other causes are microscopic polyarteritis, Wegener's granulomatosis, SLE. In elderly, bronchial carcinoma with metastasis in the kidney or membranous glomerulonephritis may occur.
Goodpasture's syndrome is a clinical syndrome of glomerulonephritis and pulmonary hemorrhage mediated by anti-GBM antibody. It starts with upper respiratory tract infection, followed by cough, hemoptysis and glomerulonephritis or renal failure. The patient usually presents with recurrent hemoptysis and progressive, proliferative glomerulonephritis. In one-third cases, no lung injury, only glomerulonephritis is present. Lung hemorrhage is more in smokers.
Goodpasture's syndrome is more in males, age 20 to 40 years. Females are affected more, if it occurs after the age of 60. Systemic features like fever, malaise, arthritis, headache, weight loss are not common, but may occur. Hypertension is usually not a feature. Chest pain and pleurisy are also rare.
P-ANCA is positive in 30%, ANA is negative and complements are normal. Associated influenza A2 virus may be found. Lung function test—Increased Co transfer due to pulmonary hemorrhage and restrictive lung disease may occur in advanced stage. In sputum, hemosiderin-laden macrophage may be present.
Anti-GBM antibody is positive (usually IgG. Occasionally, may be IgA or IgM).
Kidney biopsy shows proliferative or crescentic glomerulonephritis.
Treatment: Plasmapheresis and methylprednisolone (1 to 2 gm/day for three days). Cyclophosphamide 2 to 3 mg/kg/day may be given. 370Occasionally, kidney transplantation may be considered. Recurrence may occur in transplantated kidney.
 
Case No. 091
  1. Laboratory error.
  2. The test should be repeated.
Note: When discrepancy between the clinical and laboratory findings is present, laboratory error should be suspected. If any doubt, the test must be repeated immediately before any therapy.
 
Case No. 092
  1. Wilson's disease.
  2. Serum ceruloplasmin, serum and urinary copper.
  3. Kayser-Fleischer ring.
Note: In a young patient, if presents with prolonged jaundice or recurrent hepatitis or CLD or neurological features like involuntary movement or abnormal speech, etc. or psychiatric manifestations, Wilson's disease should be excluded. Wilson's disease is an inborn error of copper metabolism, inherited as autosomal recessive, characterized by failure of biliary excretion of copper and its deposition in and damage of different organs, such as liver, basal ganglia of brain, cornea, kidney and skeleton. There is less ceruloplasmin production of unknown mechanism.
Presentation—age of 5 to 30 years. In child, commonly hepatitis and in adult, commonly neurological features.
Liver diseases are acute hepatitis which may be recurrent, fulminating hepatic failure, chronic persistent hepatitis, chronic active hepatitis and eventually cirrhosis.
Neurological abnormalities are extrapyramidal (parkinsonism, batswing tremor, dysarthria, chorea, athetosis, dystonia), cerebellar syndrome, dementia. Usually never sensory abnormality. Unusual clumsiness for age may be an early symptom.
In the eye, KF ring, commonly in 10 to 12 o'clock position (upper periphery), due to deposition of copper in the Descemet's membrane of cornea. It is greenish brown pigmentation at the sclerocorneal junction. Occasionally, seen by slit lamp examination, may be absent or less in young children, but always present in neurological Wilson's disease. Rarely, sunflower cataract due to deposition of copper in lens (does not impair vision).371
Occasionally, the patient may present with psychiatric problem such as personality change, suicidal tendency, MDP, etc.
Investigations: (i) Low ceruloplasmin, (ii) High serum-free copper, (iii) High 24 hour urinary copper (Normal <40 mg, in Wilson's disease– 100 to 1000 mg or 0.6 mmol/24 hrs). 24 hour urinary copper following penicillamine therapy >25 mmol is a confirmatory test. Hemolytic anemia, aminoaciduria or Fanconi syndrome, osteoporosis may occur. Liver biopsy with quantitative measurement of copper (high hepatic copper, usually not done).
Treatment: Penicillamine (drug of choice) or trientine. Zinc may be helpful. Liver transplantation in fulminating hepatic failure or in advanced cirrhosis.
Prognosis is excellent, provided the treatment is started before irreversible damage. Sibling and children must be investigated and treatment should be given, even they are asymptomatic.
 
Case No. 093
  1. Polycythemia rubra vera with iron deficiency anemia.
  2. Red cell mass, LAP score, serum iron and TIBC, bone marrow study.
  3. Venesection.
Note: Clue for the diagnosis of polycythemia is high RBC, WBC and platelet. In this case, low hemoglobin, low MCV, MCH, MCHC and hypochromic blood pictures indicate associated iron deficiency.
Polycythemia rubra vera is the stem cell disorder in which there is excess proliferation of erythroid, myeloid and megakaryocyte progenitor cells.
The hematological findings in PRV are high hemoglobin, hematocrit and RBC. Also neutrophilic leukocytosis in 70%, high basophil and platelet in 50%. Others are high LAP score, high vitamin B12 and B12 binding protein transcobalamin 1. Iron deficiency may occur following venesection. Erythropoietin is low or absent (high in secondary polycythemia). Bone marrow shows erythroid hyperplasia and increased megakaryocyte.
PRV is common in males, after 40 years. Common features are hyperviscocity syndrome (headache, dizziness, blackout), pruritus after hot bath or with warm body. Plethoric deep dusky cyanosis, splenomegaly (70%), hepatomegaly (50%). Thrombosis (CVD, peripheral vascular disease) is common. There may be hypertension, 372angina, intermittent claudication and tendency to bleed. Peptic ulcer is common, bleeding may occur.
Red cell mass measured with radioactive 51Cr labelled red cells—increased (>36 ml/kg in male, >32 ml/kg in female).
Diagnosis: Plethoric appearance, splenomegaly, increased WBC and increased platelets are highly suggestive of PRV.
PRV may transform to myelofibrosis (15%), acute myeloid leukemia and refractory state with anemia.
Treatment: (i) Venesection (400 to 500 ml of blood, every 5 to 7 days) until hematocrit is <45% and platelet <400 × 109/L, (ii) Radioactive phosphorus is reserved for elderly patient (5 mCi of 32P IV, may cause acute leukemia 6 to 10-fold), (iii) Other drugs are hydroxycarbamide (hydroxyurea) or interferon may be used, (iv) Aspirin reduces risk of thrombosis.
Median survival—10 years, some 20 years.
Criteria for diagnosis of PRV—High red cell mass and normal PO2 with either splenomegaly or 2 of the following:
  1. WBC >12,000/cmm.
  2. Platelet >400,000/cmm.
  3. High B12 binding protein.
  4. High LAP score.
 
Case No. 094
  1. von Willebrand's disease (D/D—mild hemophilia).
  2. Serum vWF factor, platelet aggregation by ristocetin, factor VIII level.
  3. Aspirin.
Note: von Willebrand's disease is an inherited disorder of hemostasis. It is inherited as autosomal dominant, rarely recessive, equally affects both sexes. There is defective platelet function and factor VIII: C deficiency, which are due to deficiency or abnormality of vWF. Coagulation defect is due to the deficiency of factor VIII activity in plasma.
Bleeding tendency is usually mild, such as easy bruising, epistaxis, which follow upper respiratory infection. Sometimes, severe bleeding may occur, prolonged bleeding up to 36 hours following minor laceration, tooth extraction may occur.
Investigations: (i) Platelet is normal and prothrombin time is also normal, (ii) Prolonged both bleeding time and APTT, (iii) Failure 373of aggregation of platelet by ristocetin, (iv) Factor VIII:C activity may be reduced, (v) von Willebrand's antigen is reduced or absent, (vi) Torniquet test is positive.
VWD is confused with mild hemophilia.
Differentiating points are:
Features
Mild hemophilia
vWD
1. Sex
Male
Male and female
2. Ecchymoses and epistaxis
Rare
Common
3. Traumatic bleeding
Delay onset, persist for days to weeks
Immediate onset, persist for 1 to 2 days
4. Bleeding time
Normal
Prolonged
5. APTT
Abnormal
Abnormal
6. Prothrombin time
Normal
Normal
7. Factor VIII:C
Reduced
Normal or reduced
8. vWD antigen
Normal
Reduced
Remember: vWF is a protein synthesized by endothelial cells and megakaryocyte. Its functions—(i) Acts as a carrier protein for factor VIII. So, its deficiency results in low factor VIII, (ii) Forms bridge between platelet and endothelial component (collagen), allowing platelet adhesion to damaged vessel wall.
 
Case No. 095
  1. de Quervain's thyroiditis (subacute thyroiditis).
  2. FNAC of thyroid gland.
  3. Postpartum thyroiditis, factitious hyperthyroidism, iodine-induced hyperthyroidism.
Note: de Quervain's thyroiditis is a virus-induced transient inflammation of thyroid gland, usually self-limiting thyroiditis. Commonly by Coxsackie B4, also may be associated with influenza, infectious mononucleosis, measles, mumps, common cold. Common in females, 20 to 40 years. Presents with pain over thyroid, radiate to jaw, ear, worse by coughing, swallowing or movement of neck. Systemic features are common. Thyroid gland is enlarged and tender. FNAC shows presence of giant cells. ESR is typically high.
Inflammation of thyroid releases thyroid hormones, which are high in the blood, responsible for the features of thyrotoxicosis. This may persist for 4 to 6 weeks.374
Iodine uptake is low, because damaged follicular cells are unable to trap iodine. Low titer of thyroid autoantibody appears transiently. Hyperthyroidism followed by transient hypothyroidism may occur. Complete recovery occurs in 4 to 6 months.
Treatment: Symptomatic (NSAID). Propranolol may be used. Prednisolone 40 mg/day for 3 to 4 weeks may be needed. Antithyroid drug should not be given.
 
Case No. 096
  1. Acute lymphoblastic leukemia (ALL).
  2. Bone marrow study, cytochemical test on blast cell.
Note: ALL is common in children, peak age is 1 to 5, whereas AML is 4 times more common in adult, lowest incidence in young adult life and there is striking rise over the age of fifty years. Clinical features are due to anemia, bleeding or infection. The patient may present with flu-like illness, bleeding manifestations such as purpura, ecchymoses, gum bleeding, epistaxis. There may be sore throat, mouth ulcer, herpes labialis.
For the diagnosis, blood film shows blast cells. Sometimes, blast cell count may be very low in the peripheral blood and bone marrow is the most valuable test for the diagnosis and the material is used for cytology, cytogenetics and immunological phenotyping. The marrow is usually hypercellular with replacement of normal elements by the blast cells.
ALL are of 2 types: (i) Precursor B ALL, (ii) Precursor T ALL.
 
Case No. 097
  1. Hypogonadotrophic hypogonadism.
  2. Anterior pituitary, hypothalamus.
  3. Pituitary tumor.
  4. MRI of brain.
Note: Hypogonadism may be primary (in testis) or secondary (anterior pituitary or hypothalamus). Causes are:
  1. Primary – congenital (anorchia, Leydig cell agenesis, 5 α-reductase deficiency), acquired (mumps orchitis, tuberculosis, leprosy, castration, hemochromatosis, radiation or chemotherapy, cirrhosis of liver or alcoholism).
  2. Secondary – hypopituitarism, Kallman's syndrome (isolated GnRH deficiency), hyperprolactinemia.
    375
Causes of primary hypogonadism in females are (i) Turner's syndrome, (ii) Ovarian dysgenesis, (iii) Autoimmune oophoritis, (iv) Oophorectomy, (v) Chemotherapy, (vi) Swyer's syndrome.
 
Case No. 098
  1. Leukoerythroblastic anemia.
  2. X-ray chest, bone marrow study, CT scan (or ultrasonography of abdomen), MT.
  3. Secondary deposit, lymphoma, tuberculosis.
Note: Leukoerythroblastic anemia is characterized by presence of immature myeloid and nucleated red cells in the peripheral blood, usually secondary to disturbance in the bone marrow. Key feature of the disease is presence of nucleated red cells.
Causes are myelofibrosis, secondary deposit in the marrow, active hemolytic anemia, thalassemia major (specially after splenectomy). Rarely, it may occur in multiple myeloma, lymphoma, Gaucher's disease, Niemann-Pick's disease, marble bone disease.
 
Case No. 099
  1. Chronic lymphatic leukemia and autoimmune hemolytic anemia.
  2. Increased reticulocyte count.
  3. Coomb's test, bone marrow study.
Note: CLL is common in middle age and elderly. Clinical features are insidious, diagnosis sometimes may be incidentally done in routine blood examination. In CLL, anemia may be due to autoimmune hemolytic and bone marrow suppression by increased lymphocyte. Because of high lymphocyte, during preparation of smears, lymphocytes are deranged producing smudge cells or busket cells. Most of the lymphocytes are B type. Not all cases or even high WBC is not an indication for therapy. Patient who presents with low platelets has advanced disease and a poor prognosis rarely, CLL may transform to high grade lymphoma called Richter's transformation.
Treatment: Depends on stage. It is necessary when evidence of marrow failure, massive or progressive lymphadenopathy or splenomegaly, symptoms (weight loss, night sweat), rapidly increasing lymphocyte, etc. Usually, oral chlorambucil is given. Fludarabine is also helpful. If there is bone marrow failure or autoimmune hemolytic anemia, steroid may be given. Other treatment is radiotherapy for huge lymphadenopathy causing obstruction or discomfort, also for symptomatic splenomegaly. 376Sometimes, splenectomy may be necessary for autoimmune hemolytic anemia or hypersplenism.
Median survival—5 to 6 years, depending on stage.
 
Case No. 100
  1. Serum morning and midnight cortisol, dexamethasone suppression test, serum ACTH, CT scan of adrenal glands.
  2. Cushing's syndrome.
  3. BP, hirsutism, striae.
  4. History of steroid intake.
Note: Combination of diabetes mellitus, hypokalemic alkalosis, hypertension, obesity, polycythemia is highly suggestive of Cushing's syndrome. If there is weight loss, severe muscular weakness, more pigmentation and severe hypokalemic alkalosis, it is suggestive of ectopic ACTH syndrome. High ACTH >200 ng/L and high cortisol >1000 nmol/L is highly suggestive of ectopic ACTH syndrome.
In Cushing's syndrome, first of all, investigation should be done to confirm the diagnosis, then other investigations should be done to find out the causes.
 
Case No. 101
  1. DIC with microangiopathic hemolytic anemia with iron deficiency anemia with septicemia.
  2. Septicemia, retained product of conception or amniotic fluid embolism.
Note: DIC is a hemorrhagic disorder in which diffuse intravascular clotting causes a hemostatic defect resulting from utilization of coagulation factor and platelet in the clotting process. For this reason, this is also called consumption coagulopathy. There is secondary activation of fibrinolysis, leading to production of fibrin degradation products (FDP). The consequence of these changes are a mixture of initial thrombosis, followed by bleeding tendency due to consumption of coagulation factors and fibrinolytic activity.
Causes of DIC are (i) obstetrical (abruptio placenta, amniotic fluid embolism, abortion), (ii) surgery (specially heart and lung), (iii) hemolytic transfusion reaction, (iv) septicemia (which is due to gram-negative and meningococcal), (v) pulmonary embolism, (vi) Others are falciparum malaria, malignant disease, liver disease, trauma, burn, snake bite, heat stroke, etc.377
Chronic DIC may occur in (i) acute leukemia (usually promyelocytic), (ii) IUD, (iii) septicemia, (iv) disseminated malignancy.
 
Case No. 102
  1. SLE.
  2. Echocardiography, ANA, anti-ds DNA.
  3. Antiphospholipid syndrome.
Note: Clue for the diagnosis of SLE in this case are high ESR, low platelet and X-ray features of pericardial effusion. In a patient with SLE, if associated with antiphospholipid syndrome, there may be persistent bleeding following delivery. Other systemic features including pericardial effusion are highly suggestive of SLE. Other causes to be excluded are tuberculosis, myxedema (though in these cases, there will be no thrombocytopenia).
 
Case No. 103
  1. Senile purpura, scurvy, drug-induced purpura, paraproteinemia, amyloidosis.
  2. Arterial biopsy.
Note: Non-thrombocytopenic purpura or bleeding or bruise or ecchymoses may occur following vascular defect.
Causes are senile purpura, drugs (NSAID, steroid), Henoch-Schonlein purpura, Cushing's syndrome, Ehlers-Danlos syndrome, pseudoxanthoma elasticum.
 
Case No. 104 (see also Case No. 85)
  1. Dissociative (conversion) disorder.
  2. Reassurance, sedation, followed by psychotherapy.
Note: In any young patient, there may be HCR or panic attack presenting as difficulty in breathing. Because of hyperventilation, there is respiratory alkalosis and tetany.
Reassurance, sedation followed by psychotherapy is important.
 
Case No. 105
  1. Myasthenia gravis, opium poisoning.
  2. For myasthenia gravis, IV edrophonium bromide 10 mg.
    For opium poisoning, IV naloxone.
    378
Note: Edrophonium IV relieves the symptoms of myasthenia gravis in 30 seconds. This is also used as a test called Tensilon test for the diagnosis of myasthenia gravis. Naloxone is an antidote to opium, which counteracts the action of opium.
Tensilon test—2 mg edrophonium bromide IV. If no side effect, 8 mg IV after half minute is given. Symptoms improve in 30 seconds, last for 2 to 3 minutes. This test should be done where there is resuscitation facility. It can cause bronchoconstriction and syncope.
 
Case No. 106
  1. SLE.
  2. ANA, anti-ds DNA.
Note: Blood picture is suggestive of ITP. However, splenomegaly is not common in ITP. Sometimes, SLE may present initially as ITP. Later on, other manifestations of SLE may occur. In any patient presenting as ITP, SLE must be excluded.
 
Case No. 107
  1. Syndrome of inappropriate ADH secretion (SIADH) with tuberculous meningitis (or bronchial carcinoma).
  2. X-ray chest, plasma and urine osmolality, CT scan or MRI of brain.
  3. Restriction of water.
Other investigations: Serum ADH, water load test (oral water 20 ml/kg given. Urine collected for next 5 hours. In SIADH, there is impaired excretion of water. Normally, 80% water is excreted).
Note: In SIADH, clue for the diagnosis is everything in serum may be low (such as low sodium, chloride, bicarbonate, urea, creatinine, total protein, uric acid, etc.). Plasma renin and aldosterone are also low. Usually no hypokalemia, no edema, no hypertension or no postural hypotension. Plasma osmolality is low, urine osmolality is high, urine sodium is high (>30 mmol/L).
Treatment: (i) Restriction of water (500 to 1000 ml daily), (ii) Demeclocycline 900 to 1200 mg daily may be used to make iatrogenic nephrogenic diabetes insipidus. (iii) Occasionally hypertonic saline 5% 200 cc may be given plus frusemide IV (hypertonic saline may be dangerous, may cause central pontine myelinolysis). (iv) Treatment of the primary cause should be done.
Commonest cause of SIADH—oat cell carcinoma of lung.379
 
Case No. 108
  1. Hemolytic uremic syndrome.
  2. PBF to see schistocyte or fragmented RBC, serum FDP, serum electrolytes.
Note: HUS is characterized by rapid onset of microangiopathic hemolytic anemia, thrombocytopenia and acute renal failure (triad) due to thrombosis in small arteries and arteriole. HUS is caused by verotoxin-producing organism, such as enterotoxigenic E. coli 0157 and H7. It occurs in up to 6% patients infected with this organism, by infected food (undercooked beef) or unpasteurized milk. The organisms excrete a toxin, responsible for the gastrointestinal effect, such as colicky abdominal pain diarrhea, may be bloody.
Pathogenesis: Infection triggers damage to the endothelial cells of microcirculation and derangement of hemostatic coagulation system. This is followed by cell swelling, platelet clumping, fibrinogen deposition, thrombosis and occlusion. Common in glomerular capillaries and renal arterioles. Other features are intravascular hemolysis, high bilirubin, high LDH, reduced haptoglobin and increased reticulocyte count. Confused with DIC, but coagulation tests are normal in HUS.
HUS is common in children, <3 years. It usually follows upper respiratory tract or gastrointestinal infection. There is fever, vomiting and diarrhea, often bloody called diarrhea associated HUS (D-HUS). Intravascular hemolysis is followed by oliguria or anuria. Purpura, anemia, bleeding, drowsiness and hypertension may occur.
Peripheral blood film shows schistocyte, spherocyte and thrombocytopenia.
Treatment: Supportive (transfusion, hydration, control of hypertension), FFP, plasmapheresis, dialysis. Heparin or antiplatelet drug may be helpful. No role of antibiotic. 5% die in acute episode, 5% develop CRF and 30% develop persistent proteinuria.
 
Case No. 109
  1. Porphyria cutanea tarda (PCT).
  2. Serum ferritin, liver biopsy.
  3. Use sunscreen, alcohol must be stopped.
Note: Blistering skin lesion associated with biochemical evidence of liver disease is suggestive of PCT. It is the commonest porphyria, may be hereditary (autosomal dominant) or sporadic, due to abnormality 380of hepatic uroporphyrinogen decarboxylase, which catalyses the conversion of uroporphyrin to coproporphyrin.
PCT may be acquired in cirrhosis of liver, benign or malignant hepatic tumor, chronic alcoholism. May be precipitated by hepatitis C, HIV, iron overload, estrogen therapy.
The patient presents with painless bullous eruption or blister on dorsal surface of hands. This may be followed by scarring. Also, there is hypertrichosis and hyperpigmentation. Abnormal liver functions, high iron and high transferin saturation may occur. Liver biopsy will show evidence of CLD and iron stain will show iron overload.
Features of prophyria cutanea tarda are (i) Urine shows coral pink fluorescence under Wood's light, (ii) Urine total uroporphyrin is increased, (iii) Stool for coproporphyrin is increased, (iv) Urine δ ALA is increased, (v) Biopsy of local skin lesion shows subepidermal blister, (vi) Liver function test—abnormal, (vii) Serum ferritin—high. Blisters on the skin lesion contain PAS-positive material.
Treatment: Treatment of primary cause. Sunscreen, venesection- one unit every 2 to 4 weeks, until hemoglobin is <12 gm/dl, low dose chloroquine or hydroxychloroquine—200 mg twice weekly (it increases urinary excretion of porphyrin). Desferrioxamine is given, if ferritin is high. Alcohol must be stopped.
 
Case No. 110
  1. Pseudopolycythemia.
  2. Red cell mass and plasma volume.
  3. Venesection.
Note: In pseudopolycythemia, both Hb and PCV are high due to hemoconcentration resulting from any cause of reduced extracellular volume (dehydration or diuretic therapy). A type of pseudopolycythemia, also called Gaisbocks syndrome, is a disorder of unknown etiology. It is also called polycythemia of stress, as half of the patients are in anxiety state or neurotic. Common in middle-aged males and have a smoking history, associated with obesity, hypertension, peripheral vascular disease. It may cause myocardial infarction, cerebral ischemia. In this type of polycythemia, only red cells are high and WBC and platelet are normal, plasma volume is low and red cell mass is normal (in contrast to PRV, red cell mass and plasma volume are high, other cells such as WBC and platelet are also high).
Treatment: Venesection. Smoking must be stopped.381
 
Case No. 111
  1. Hypokalemic alkalosis.
  2. Hypomagnesemia.
Note: Tetany may occur secondary to metabolic alkalosis, hypocalcemia, hypomagnesemia or hypokalemic alkalosis. Never forget hypomagnesemia as a cause of tetany other than hypocalcemia.
Features of hypomagnesemia are mostly neurological such as tremor, chorea, confusion, agitation, fit, hallucination, etc.
Causes of hypomagnesemia are prolonged diarrhea, less intake as in protein calorie malnutrition, prolonged parenteral nutrition without magnesium, vomiting, aspiration, fistula, laxative abuse, renal loss in diabetic ketoacidosis, renal tubular acidosis, diuretic phase of acute tubular necrosis. Others are chronic alcoholism, hyperparathyroidism, primary aldosteronism, Bartter's syndrome, acute pancreatitis, drugs (loop diuretic, cisplatin, gentamicin).
Treatment: It should be corrected by giving IV infusion of magnesium.
 
Case No. 112
  1. Left renal vein thrombosis.
  2. Left renal artery embolism.
  3. Ultrasonogram of kidney, isotope renogram, IVP.
Note: In nephrotic syndrome, if there is loin pain, hematuria and deterioration of renal function, it is highly suggestive of renal vein thrombosis.
In nephrotic syndrome, there is hyperlipidemia, commonly high LDL and cholesterol, high VLDL and triglyceride. So, more atherosclerosis. There is hypercoagulable state due to increased fibrinogen and factor VIII, also reduction of antithrombin III. Thrombosis may occur, especially in renal vein. Other complications in nephrotic syndrome are loss of TBG resulting in low T3 and T4, loss of transferin and iron, loss of vitamin D binding protein, infection such as pneumococcal peritonitis and septicemia.
Renal vein thrombosis is more common is membranous nephropathy, mesangiocapillary glomerulonephritis and amyloidosis.
Factors for thrombosis in nephrotic syndrome are (i) Loss of fibrinolytic factors in urine commonly antithrombin III, also protein C and S, plasminogen, (ii) Increased synthesis of clotting factors—factor V, VIII and fibrinogen, (iii) Thrombocytosis, (iv) Over diuresis causes dehydration, reducing renal blood flow and increasing viscosity.382
 
Case No. 113
  1. Catheterization to evacuate the urinary bladder.
  2. Hyponatremia.
  3. Water intoxication secondary to psychogenic water drinking.
  4. Restriction of water intake.
Note: Catheterization to evacuate the urinary bladder is the first therapeutic measure to be done. This is followed by correction of electrolyte imbalance. Excess intake of water is due to polydipsia, resulting in dilutional Hyponatremia. It may be confused with SIADH, but polydipsia and polyuria are not common in SIADH.
 
Case No. 114
  1. Tertiary hyperparathyroidism.
  2. Serum parathormone, X-ray of bone (skull, hands), isotope bone scan.
  3. Parathyroidectomy.
  4. Renal osteodystrophy
Note: Tertiary hyperparathyroidism means development of apparently autonomous parathyroid hyperplasia after long-standing secondary hyperparathyroidism. In this case, CRF is the cause. In tertiary hyperparathyroidism, serum calcium and phosphate are very high, latter is more.
USG, CT scan, MRI of the parathyroid gland may be done. Renal osteodystrophy is a group of metabolic bone disease in chronic renal failure which consists of osteoporosis, osteomalacia, osteosclerosis and osteitis fibrosa cystica.
Treatment: Parathyroidectomy is usually done.
 
Case No. 115
  1. Hereditary angioedema.
  2. Serum C2 and C4, C1 esterase inhibitor (C1-INH) measurement.
Note: Hereditary angioedema, inherited as autosomal dominant, is due to deficiency of C1 esterase inhibitor (C1-INH), a component of complement system. As a result, there is uncontrolled activation of C1. It is characterized by angioedema in skin, larynx but no urticaria, no itching. There may be recurrent acute abdomen (due to intestinal edema).383
Rarely, this condition may be acquired, associated with lymphoma or SLE. However, in such cases, there are low C1-INH and also low C1, C2. Acquired form usually occurs in older age.
Diagnosis: Common in late childhood or early adolescence. Attack may follow trauma, infection, dental procedures or emotional stress. Also, there is increasing frequency and severity during puberty, menstruation and ovulation. Usually family history is present. Patient can present with any combination of cutaneous angioedema, abdominal pain or acute airway obstruction. Serum C4 and C2 are low, but C1 and C3 are normal. Barium meal and follow through show intestinal sacked coin appearance during attack. Confirmed by measurement of serum C1-INH.
Treatment: Purified C1-INH infusion during acute attack. Fresh-frozen plasma may be used. Steroid and adrenaline are ineffective.
For prevention, danazol or stanozolol may be given, which increase C1-INH, C2 and C4 by hepatic synthesis.
 
Case No. 116
  1. History of taking steroid.
  2. Secondary adrenocortical failure.
  3. MRI of pituitary fossa, USG or CT scan of adrenal gland.
Note: Progressive increase of cortisol after high dose dexamethasone is typical of secondary adrenocortical failure. This may be due to prolonged use of steroid. However, disease secondary to pituitary may also cause this type of response.
 
Case No. 117
  1. Microcytic hypochromic with leukoerythroblastic blood picture.
  2. Venesection and transformation to myelofibrosis.
  3. Venesection.
Note: In PRV, depletion of iron store or iron deficiency is usual. After venesection, iron deficiency occurs. Peptic ulcer is also common, hematemesis may occur. Venesection should be done carefully if platelet count is very high, because of risk of thrombosis. PRV may transform to refractory anemia, myelofibrosis or acute leukemia.
 
Case No. 118
  1. Nephrogenic diabetes insipidus (NDI).
  2. History of intake of lithium.
    384
Note: During water deprivation test, normally urine osmolality should rise. In nephrogenic diabetes insipidus, there is impairment of active chloride transport in the ascending loop of Henle. Following desmopressin, failure to concentrate urine indicates renal insensitivity to ADH. In NDI, there is insensitivity of the renal tubule to ADH. In cranial diabetes insipidus, there is also failure to rise the urine osmolality during water deprivation test, but following desmopressin, there is rise of urine osmolality.
Causes of NDI: (i) Familial or hereditary, (ii) Drugs—lithium, demeclocycline, glibenclamide, (iii) Hypokalemia and hypercalcemia (act in part by antagonizing the effects of desmopressin on the collecting duct), (iv) Sickle cell disease, (v) Idiopathic.
 
Case No. 119
  1. Giant cell arteritis (GCA).
  2. Temporal artery biopsy.
  3. Infective endocarditis, disseminated tuberculosis, SLE, para-neoplastic syndrome.
Note: Giant cell arteritis or temporal arteritis is an important cause of PUO in elderly. Giant cell arteritis and polymyalgia rheumatica may present either separately or in combination, involves large artery with an internal elastic component causing arterial occlusion. Uniocular blindness may occur due to retinal artery occlusion, binocular blindness may also occur. Visual loss in 25% cases. Posterior ciliary artery occlusion causes acute anterior ischemic optic neuropathy, the disk is pale and swollen. GCA can cause sudden blindness, brainstem ischemia, cortical blindness, TIA and stroke (see also Case No. 27).
 
Case No. 120
  1. Primary hyperparathyroidism.
  2. Serum PTH, hydrocortisone suppression test, X-ray of hand, CT scan of neck, thallium and technetium substraction scan of parathyroid and thyroid.
Note: High calcium, low phosphate and high alkaline phosphatase are typical of primary hyperparathyroidism. In secondary hyper-parathyroidism, which is secondary to chronic renal failure, there is high phosphate and calcium is low or near normal, also high urea and creatinine. In tertiary hyperparathyroidism, autonomous parathyroid hyperplasia occurs which develops after secondary hyperparathyroidism.385
 
Case No. 121
  1. Chronic subdural hematoma.
  2. CT scan of brain.
Note: Accumulation of blood in subdural space is due to rupture of the vein, commonly during head injury, even mild. Interval between the injury and symptoms may be days or weeks or months. Symptoms may be delayed in elderly or alcohol abuse.
Common features are headache, drowsiness, confusion. There may be hemiparesis, sensory loss, epilepsy, stupor, even coma.
Treatment: Spontaneous resolution may occur. Otherwise, neurosurgical intervention (burr hole drainage).
 
Case No. 122
  1. Multiple myeloma, polymyalgia rheumatica, secondary deposit in lumbosacral region.
  2. For each case, likely investigations are:
    1. Multiple myeloma—bone marrow study.
    2. Polymyalgia rheumatica—therapeutic trial with prednisolone.
    3. Secondary deposit in the lumbosacral region—X-ray of lumbosacral spine or MRI.
    High ESR and pancytopenia are suggestive of bone marrow infiltration.
Note: Other causes are osteoarthrosis, osteomalacia, osteoporosis, Paget's disease. However, these are not associated with pancytopenia.
 
Case No. 123
  1. Multiple myeloma.
  2. Bone marrow study.
  3. Serum protein electrophoresis, X-ray of skull.
Note: Elderly patient with high ESR, hypercalcemia, renal failure, high protein, etc. is highly suggestive of multiple myeloma, which is characterized by neoplastic proliferation of plasma cells in bone marrow with production of monoclonal paraprotein.
In PBF, marked rouleaux formation is common. There may be leukoerythroblastic blood picture. Bone marrow shows atypical plasma cells >30%. Serum electrophoresis may show M band. Bence-Jones proteinuria is present in 20% cases.386
X-ray shows lytic lesion (due to production of cytokines which stimulate osteoclast responsible for bone resorption).
Treatment: (i) Supportive (correction of anemia, control of infection, etc.). (ii) Chemotherapy—melphalan or cyclophosphamide plus prednisolone in cycle. (iii) Other therapy—VAD or thalidomide plus dexamethasone. New drug bortezomib (valcade) is highly promising. (iv) Autologous stem cell transplantation in < 65 years. Allogenic bone marrow transplantation in < 55 years.
Bad prognostic factors are high urea, low hemoglobin, high β2 microglobulin, low albumin and high calcium at presentation.
 
Case No. 124
  1. Sinus bradycardia, complete heart block.
  2. Repeat ECG.
  3. Injection atropine IV or temporary pacemaker.
Note: Sinus bradycardia or complete heart block are quite common complications in acute inferior myocardial infarction. There is usually complete recovery from complete heart block after 10 to 14 days.
 
Case No. 125
  1. Water intoxication due to psychogenic excess drinking, SIADH.
  2. Plasma and urinary osmolality, serum ADH (low).
Note: In psychogenic polydipsia, both plasma and urinary osmolality will be low. In SIADH, plasma osmolality is low, but urinary osmolality is high.
In nephrogenic diabetes insipidus, plasma osmolality is high or normal, but urinary osmolality is low.
Chronic intake of a large amount of water impairs renal concentrating mechanism making the difference between primary polydipsia and true diabetes insipidus more difficult. Water deprivation test is helpful to differentiate between these two. A psychiatric history is also helpful.
 
Case No. 126
  1. Lag storage curve.
  2. Partial gastrectomy.
  3. Hypoglycemia.
  4. Combined iron deficiency anemia and deficiency of vitamin B12 or folic acid (less intake of food).
    387
Note: Following ulcer surgery, there may be dumping syndrome. Initially, increased glucose is due to rapid absorption. Later, there is increased insulin secretion, responsible for hypoglycemia, causing fainting attack.
In lag storage GTT, after 30 minutes, glucose level is high which exceeds renal threshold. This stimulates insulin secretion, which results in subsequent hypoglycemia.
Causes of lag storage GTT are (i) reactive hypoglycemia (normal variant), (ii) thyrotoxicosis, (iii) severe liver disease, (iv) gastrectomy or gastrojejunostomy.
Causes of flat glucose tolerance curve (failure to rise glucose in OGTT) are (i) normal (ii) malabsorption state, (iii) Addison's disease, (iv) hypopituitarism (with growth hormone deficiency).
 
Case No. 127
  1. Paracetamol.
  2. Hepatic failure due to toxic metabolite of paracetamol.
  3. Hepatic coma, sedative, hypoglycemia.
  4. Serum paracetamol level, blood glucose, prothrombin time.
Note: Intake of 15 gm of paracetamol is considered potentially serious in most cases. Toxic metabolite of paracetamol is NAPQI, which is normally conjugated with glutathione and is excreted. In paracetamol poisoning, there is production of excess toxic metabolite and deplete cellular glutathione. The liver is unable to excrete NAPQI, which is harmful to the liver, causes massive hepatic necrosis and hepatic failure (not by the drug itself). No liver damage until 18 hours. If blood level of paracetamol is >200 mg/ml, it indicates severe poisoning. Maximum liver damage occurs after 72 to 96 hours of ingestion. There is increased prothrombin time and aminotransferase activity (AST or ALT). There may be hypo- and hyperglycemia, metabolic acidosis, arrhythmia, GIT bleeding, cerebral edema, lactic acidosis and coma. Brainstem coning may occur after 96 hours.
Three important risk or prognostic markers for severe hepatic injury are (i) prothrombin time is >20 sec. in 24 hours, (ii) pH <7.3, (iii) serum creatinine >300 mmol/L. If a peak prothrombin time is >180 sec., mortality is 90%. If creatinine is >300 mmol/L, mortality is 70%. If pH <7.3, mortality is 85%. Without treatment, a few patients may develop fulminant hepatic failure. Renal failure may develop, due to acute tubular necrosis in 25% cases.388
Treatment: (i) Gastric lavage within 4 hours, (ii) N-acetylcystine IV or methionine orally. More effective, if given within 10 hours. Protective effects decline rapidly and ineffective after 15 to 16 hours, (iii) If PT prolonged, fresh-frozen plasma, (iv) Glucose may be needed. Forced diuresis and dialysis have no role. Dialysis only if renal failure.
Monitor: LFT (ALT and PT), electrolytes, glucose and creatinine.
 
Case No. 128
  1. Severe insulin resistance.
  2. Due to antibody against insulin receptor.
Note: This type of insulin resistant diabetes mellitus is rarely found in young female, associated with acanthosis nigricans and androgenic features (hirsutism, acne, oligomenorrhea). Also in older women, circulating immunoglobulin binds to insulin receptor and reduces their affinity to insulin.
Insulin resistance is defined as defective glucose disposal associated with raised glucose and insulin concentration.
Causes are (i) Physiological (puberty, pregnancy, old age, bed-rest), (ii) Metabolic (type 2 DM, obesity, PCOS, syndrome X or insulin resistance syndrome), (iii) Endocrine (acromegaly, Cushing's syndrome, thyrotoxicosis, glucagonoma, pheochromocytoma), (iv) Others are acanthosis nigricans, myotonic dystrophy, cirrhosis of liver, hemochromatosis.
Insulin resistance syndrome: It is a metabolic derangement characte-rized by (i) Insulin resistance, (ii) Hypertension, (iii) Dyslipidemia (high TG, low HDL), (iv) Obesity, (v) Type 2 DM, (vi) Accelerated cardiovascular disease. This syndrome is also called metabolic syndrome or syndrome X or CHAOS (coronary artery disease, hypertension, atherosclerosis, obesity, stroke).
 
Case No. 129
  1. Salicylate poisoning.
  2. Serum salicylate, serum electrolytes, CT scan of brain.
Note: There is severe metabolic acidosis with respiratory alkalosis, likely cause in this early age is salicylate poisoning. There is respiratory alkalosis, characterized by low PaCO2, which is due to hyperventilation and wash out of carbon dioxide by direct stimulation of respiratory center by salicylate.
Later on, a combined respiratory and metabolic acidosis may occur due to (i) Depression of respiratory center by the increased level of 389salicylate (respiratory acidosis), (ii) Retention of organic metabolic acids by the kidney because of hypotension and dehydration, (iii) Salicylate impairs carbohydrate metabolism with accumulation of acetoacetate, lactic acid and pyruvic acid.
In salicylate poisoning, urine for ferric chloride test is positive (shows presence of reducing substance). Salicylate interferes with metabolism of carbohydrate, lipid, protein and amino acid. Ketone bodies are increased due to excess of fat metabolism. Lactic acid and pyruvic acid are also increased, because of inhibition of Krebs cycle by salicylate.
Other causes of severe metabolic acidosis are (i) diabetic ketoacidosis, (ii) renal failure, (iii) lactic acidosis.
 
Case No. 130
  1. Cervical cord compression at C6 level.
  2. Cervical myelopathy, motor neuron disease, multiple sclerosis.
  3. Sensory sign and its level.
  4. X-ray of cervical spine, MRI of cervical spine.
Note: Progressive increase of neurological sign is highly suggestive of cord compression. Loss of biceps and exaggerated triceps indicates lesion at C6. At the level of lesion, there is LMN sign and below the level, UMN sign. This type of reflex is called inversion of reflex.
 
Case No. 131 (see also Case No. 99)
  1. Chronic lymphatic leukemia.
  2. Autoimmune hemolytic anemia and bone marrow infiltration.
  3. Coomb's test, bone marrow study.
Note: CLL may be asymptomatic in 25% cases, diagnosed incidentaly in routine investigation. There may be generalized lymphadenopathy, hepatosplenomegaly. Huge splenomegaly may occur if there is auto-immune hemolytic anemia. Coomb's test is positive in 20% cases.
CLL is common in males (M:F = 2:1), usually after 45 years, involving B lymphocyte. Hemolysis is usually by warm autoantibody and Coomb's test is positive.
 
Case No. 132
  1. Diabetic ketoacidosis (DKA).
  2. Blood sugar and urine for ketone bodies.
    390
Note: Dry tongue, weight loss, severe metabolic acidosis are the clue for the diagnosis of DKA. It is common in IDDM (type 1). The patient may present first time with DKA in younger age.
Three main problems in DKA are (i) hyperglycemia, (ii) hyper-ketonemia and (iii) metabolic acidosis. Marked dehydration is common.
Five percent cases develop coma in DKA. Average loss of fluid and electrolytes are (i) Water—6 liters, (ii) Sodium—500 mmol, (iii) Chloride—400 mmol, (iv) Potassium—350 mmol. Half of the lost fluid is intracellular. If bicarbonate is <12 mmol/L, it indicates severe acidosis.
Treatment: IV normal saline rapidly, IV soluble insulin preferably with pump, potassium therapy, control of infection, correction of acidosis by sodibicarb, if arterial pH is <7 (complete correction of acidosis is avoided, because there is correction of extracellular acidosis with isotonic, 1.26%, but acidosis in brain is persistent, which aggravates cellular dysfunction in brain).
Complications of DKA: Cerebral edema, ARDS, DIC, thromboembolism, acute circulatory failure. Mortality is 5 to 10%, more in elderly.
 
Case No. 133
  1. Primary hemochromatosis.
  2. Serum ferritin, liver biopsy for iron studies.
  3. Chondrocalcinosis.
  4. Cardiomyopathy.
Note: Combination of diabetes mellitus, cardiac abnormality, hepatic problem with arthritis, and pigmentation are highly suggestive of hemochromatosis. It may be primary or secondary. Primary hemochromatosis is a hereditary disorder, inherited as autosomal recessive, characterized by increased iron absorption by the intestine, increased total body iron and deposition of iron in different organs of body with its dysfunction. Commonly, iron is deposited in liver, heart, pancreas, endocrine organs, joints, testis, skin, etc. Common in males, >40 years, females are protected by menstruation and pregnancy, but hemachromatosis occurs after menopause. Chondrocalcinosis may occur. Deposition of iron in liver leads to cirrhosis and may cause hepatoma in 33% cases. Leaden grey skin pigmentation is called bronze diabetes. Cardiac involvement leads to dilated or restrictive cardiomyopathy, abnormal ECG. Hypogonadism occurs due to deposition of iron in gonads, also hypopituitarism may occur.391
Investigations: High iron, ferritin, high saturation of TIBC. CT scan or MRI of hepatobiliary system may be done. Liver biopsy for iron staining and quantitative measurement of iron (>180 mmol/gm liver tissue).
Treatment: Weekly venesection of 500 ml of blood (it reduces 250 mgm of iron) should be done until ferritin comes to <50 ng/ml. May take 2 to 3 years. Alcohol and vitamin C must be avoided (cause more absorption of iron).
Causes of secondary hemochromatosis are (i) repeated blood transfusion (>50 liters), (ii) chronic hemolytic disease, (iii) sideroblastic anemia, (iv) porphyria cutanea tarda, (v) alcoholism, (vi) Bantu siderosis (in Africa).
 
Case No. 134
  1. SLE.
  2. TPHA, ANA, anti-ds DNA.
Note: In this female patient, splenomegaly, thrombocytopenia, high ESR are the good clue for the diagnosis of SLE. In SLE, false positive VDRL may be present, especially when there is high antiphospholipid. False positive VDRL may also be found in leprosy, narcotic abuse, old age. High ASO titer is not significant, it indicates previous streptococcal infection.
 
Case No. 135
  1. Adrenal carcinoma and ectopic ACTH syndrome (oat cell carcinoma of bronchus in smokers is the likely cause).
  2. X-ray chest, ultrasonography to see suprarenal gland, serum ACTH, CT scan of adrenal glands.
  3. Hypokalemia, proximal myopathy, carcinomatous myopathy.
  4. Secondary deposit from bronchial carcinoma, osteoporosis.
Note: Cushing's syndrome may present in any carcinoma, commonly oat cell carcinoma of bronchus, ovarian carcinoma, etc. In ectopic ACTH syndrome, the patient is emaciated, pigmented with severe hypokalemic alkalosis. ACTH is very high and also cortisol is very high.392
 
Case No. 136
  1. Subacute combined degeneration of spinal cord with pernicious anemia.
  2. Serum B12 assay, bone marrow study.
  3. Vibration and position sense, fundoscopy to see optic atrophy.
Note: SCD is characterized by combination of peripheral neuropathy, signs of posterior column lesion (loss of vibration and position sense) and signs of pyramidal lesion (due to involvement of lateral column). Romberg sign may be positive. SCD occurs as a sequele to Addisonian pernicious anemia and rarely due to other causes of vitamin B12 deficiency. Other evidences of B12 deficiency such as anemia, glossitis, optic atrophy, dementia may be present.
Pathological changes are (i) demyelination of peripheral nerve, (ii) degeneration of ascending tract in posterior column, (iii) degeneration of desending pyramidal tract in lateral column of spinal cord.
Treatment: Injection vitamin B12, 1000 μgm IM daily for one week, then weekly for one month, then every three months. Treatment of primary cause.
 
Case No. 137
  1. Metabolic acidosis and metabolic alkalosis.
  2. Chronic renal failure.
  3. CRF with severe vomiting, CRF with bicarbonate therapy.
Note: Low bicarbonate indicates acidosis and high pH indicates alkalosis. In CRF, metabolic acidosis is common. Because of repeated vomiting, there may be metabolic alkalosis. If sodibicarb is given, there is also correction of acidosis.
 
Case No. 138
  1. Spontaneous pneumothorax.
  2. X-ray of chest (PA view).
Note: In any young patient, common cause of sudden severe chest pain is spontaneous pneumothorax. Other causes of chest pain may be pleurisy, reflux esophagitis, costochondritis, pneumonia and pulmonary embolism. Acute myocardial infarction and dissecting aneurysm are other rare possibilities.393
 
Case No. 139
  1. Bleeding in the ileopsoas muscle and retroperitoneal bleeding.
  2. Ileopsoas hematoma causing compression of left femoral nerve.
  3. Insufficient dose, poor quality antihemophilic factor, antibody against factor VIII.
  4. Abdominal ultrasonography or CT scan.
Note: Ileopsoas bleeding may occur in hemophilia. The patient has difficulty in flexion of hip and paraesthesia along the distribution of femoral nerve. Muscle bleeding may occur in any muscle causing ischemic damage to the nerve by entrapment, leading to muscle necrosis or Volkman's ischemic contracture. In hemophilia, clotting time and APTT are prolonged, but bleeding time, prothrombin time, platelet, fibrinogen are all normal.
 
Case No. 140
  1. Mixed respiratory alkalosis and metabolic acidosis.
  2. Portosystemic encephalopathy.
  3. Respiratory alkalosis is due to hypoxia and elevated diaphragm due to ascites. Metabolic acidosis is due to diarrhea.
Note: Diarrhea in cirrhosis is a cause of hyperchloremic acidosis. Hypoxia and elevated diaphragm are responsible for hyperventilation, resulting in respiratory alkalosis.
 
Case No. 141
  1. Partially treated bacterial meningitis, tuberculous meningitis, brain abscess, fungal infection.
  2. AFB staining, culture and sensitivity, immunoelectrophoresis for bacterial antigen, PCR.
  3. CT scan or MRI of brain, blood for C/S, serum electrolytes, urine for ketone bodies, X-ray chest.
Note: Causes of increased lymphocyte in CSF are (i) infection (tuberculous, viral, fungal, meningovascular syphilis), (ii) neoplastic (lymphoma, leukemia, secondary deposit), (iii) sarcoidosis, (iv) some-times, chronic brain abscess, (v) partially treated bacterial meningitis.394
 
Case No. 142
  1. Familial periodic paralysis, thyrotoxic periodic paralysis.
  2. Thyrotoxic periodic paralysis. Systolic hypertension, tachycardia, are in favor of thyrotoxicosis.
  3. Whether patient feels comfortable with activity.
Note: In familial periodic paralysis, inherited as autosomal dominant, there is membrane abnormality.
Three types are (i) hypokalemic—lasts for days. (ii) hyperkalemic—lasts for hours (in hyperkalemic type, myotonia of tongue and eye may occur, common in <10 years of age), (iii) normokalemic.
Hypokalemic periodic paralysis is rare, characterized by episodic extreme weakness, progress from proximal to distal. Cranial and respiratory muscles are spared. It usually occurs during rest after prolonged exercise, also while the patient is asleep. Precipitating factors are increased carbohydrate meal, cold, rest after exercise, alcohol, anxiety or tension and cause of hypokalemia is unknown, but shift of potassium from extracellular fluid to the intracellular fluid is responsible. Symptoms can be precipitated by intravenous glucose and insulin into this patient which would support the diagnosis of potassium shift theory. Long-term treatment with potassium supplement or potassium sparing diuretic are given.
Thyrotoxic periodic paralysis (TPP): If a thyrotoxic patient develops sudden or periodic weakness, it is called thyrotoxic periodic paralysis. It is due to hypokalemia (caused by entry of potassium into the cell), common in Asians. May occur following excess carbohydrate or glucose or heavy exercise. Persists up to 7 to 72 hours. Treatment of thyrotoxicosis improves the condition.
 
Case No. 143
  1. Acute myocardial infarction, spontaneous pneumothorax, pulmonary embolism.
  2. Repeat ECG, X-ray of chest (PA view), serum troponin 1 (and ventilation/perfusion scan).
Note: Following any chest surgery, if the patient develops sudden respiratory distress or chest pain, always exclude the above disorders. In pulmonary edema, PO2 is usually much lower. ECG may appear normal in early case of myocardial infarction. Enzymes such as troponin I, CPK, SGOT may be given.395
 
Case No. 144
  1. Pulmonary hypertension.
  2. Atrial fibrillation with pulmonary embolism.
  3. Pulmonary infarction.
  4. Eisenmenger's syndrome.
  5. Heart lung transplantation.
Note: This patient is suffering from VSD, now develops CCF, secondary to pulmonary hypertension. Patient with pulmonary hypertension and reversal of shunt is called Eisenmenger's syndrome. When it is due to VSD, then it is called Eisenmenger's complex. If there is Eisenmenger's syndrome, there is no surgical closure. Only supportive treatement is given. Heart and lung transplantation may be considered.
 
Case No. 145
  1. Prerenal acute renal failure.
  2. Vomiting.
  3. Correction of electrolytes by intravenous normal saline with potassium, also bicarbonate.
Note: Prerenal acute renal failure may occur in severe vomiting, diarrhea, bleeding, burn, excess aspiration, etc. If properly treated, it is reversible.
Remember the urea/creatinine ratio in relation to renal failure. In some cases, urea is very high, but creatinine is normal which does not indicate renal failure. Conversely, in some cases, urea is normal or low, but creatinine is high indicating renal failure. Causes of high urea (but normal creatinine), high urea/creatinine ratio- (i) High protein diet, (ii) Dehydration, (iii) Gastrointestinal hemorrhage, (iv) steroid therapy. Causes of high creatinine (but normal urea), low urea/creatinine ratio- (i) Vomiting (more urea loss), (ii) Liver disease (less urea production), (iii) Peritoneal or hemodialysis (urea reduces quickly), (iv) Rhabdomyolysis, (v) Drug (trimethoprim), (vi) Protein restricted diet.
 
Case No. 146
  1. Cirrhosis of liver with erythropoietin secreting hepatoma with secondary polycythemia.
  2. Cirrhosis of liver with secondary polycythemia due to pulmonary A-V shunting.
    396
  3. USG of hepatobiliary system, serum α-fetoprotein, CT scan of hepatobiliary system, liver biopsy.
Note: This patient developed hepatoma secondary to CLD, also there is high hemoglobin and RBC, which indicates secondary polycythemia. 80% hepatoma is secondary to cirrhosis of liver. Metabolic abnormalities in hepatoma are polycythemia, hypercalcemia, hypoglycemia, por-phyria cutanea tarda. α-fetoprotein is commonly high in hepatoma, >500 ng/ml is highly suggestive. Hepatorenal syndrome may occur.
 
Case No. 147
  1. Myelodysplastic syndrome (refractory anemia).
  2. Ring sideroblast.
Note: This patient has been suffering from transfusion dependent anemia, which is likely to be myelodysplastic syndrome. This is a group of acquired bone marrow disorders due to defect in stem cells, characterized by increasing marrow failure with quantitative and qualitative abnormality of all three cell lines. There are anemia, neutropenia and thrombocytompenia, usually with hypercellular or normocellular marrow. Common in elderly, transform to AML in 30% cases.
In MDS, blood picture shows- cytopenia (thrombocytopenia and leukopenia), hypogranular neutrophil, hyposegmented neutrophil (Pelger cells) or hypersegmentated neutrophil, MCV- high (macrocytic) or normal.
Bone marrow—Hypercellular with dysplastic change despite pancytopenia. Megaloblastic changes, ring sideroblast in all types, dyserythropoiesis, granulocyte precursor and megakaryocyte show abnormal morphology.
Chromosome analysis reveals abnormality in chromosome 5 or 7.
The patient presents with anemia, infection or bleeding mani-festations. Secondary MDS may be seen in any patient treated with radiotherapy, chemotherapy or combination (as in lymphoma).
Types of diseases in MDS:
(i) Refractory anemia (RA)- blast <5%, erythroid dysplasia only, (ii) Refractory anemia with sideroblast (blast <5%, ring sideroblast >15%), (iii) Refractory cytopenias with multilineage dysplasia- (blast <5%, 2 to 3 lineage dysplasia), (iv) Refractory anemia with excess blast (blast 5 to 20%, 2 to 3 lineage dysplasia), (v) Refractory anemia with excess blast in transformation (blast 20 to 30%), (vi) MDS with 3975q-MDS with del (5q) cytogenetic abnormality blasts <5%, normal or increased blood platelet. (vii) MDS unclassified- none of the above or inadequate material.
Treatment:
  1. If blast <5% – (i) Supportive therapy with platelet, red cell trans-fusion, (ii) Erythropoietin or G-CSF may be given.
  2. If blast >5% – (i) Supportive therapy, (ii) Chemotherapy- low dose hydroxyurea or etoposide, (iii) Allogenic stem cell transplantation in young < 55 years.
Prognosis: Slowest in refractory anemia, rapidly progresses in refractory anemia with ring sideroblast.
 
Case No. 148
  1. Ovarian androgen secreting tumor.
  2. Ultrasonography of abdomen, CT scan of abdomen to see ovarian mass, laparoscopy.
Note: Failure of testosterone to fall after low dose dexamethasone suppression test indicates that this is not a polycystic ovary or con-genital adrenal hyperplasia. In ovarian androgen secreting tumor, there is failure of suppression by dexamethasone.
 
Case No. 149
  1. Distal renal tubular acidosis.
  2. Osteomalacia with myopathy and hypokalemia.
  3. Osteomalacia.
  4. Ammonium chloride test (other test—pH of overnight urine).
  5. Sodium bicarbonate therapy.
Note: Hypokalemia is usually associated with alkalosis. In distal RTA, hypokalemia is associated with acidosis. High chloride, low potassium with metabolic acidosis is highly suggestive of RTA. Muscular weak-ness is due to severe hypokalemia and hypocalcemia.
 
Case No. 150
  1. Restrictive lung disease.
  2. Interstitial lung disease with chronic corpulmonale.
  3. Chest X-ray, CT scan of chest, ABG (arterioral blood gas analysis), bronchoscopy and bronchoalveolar lavage, transbronchial or open lung biopsy.
    398
Note: FEV1 and FVC are proportionately low and the ratio between these is normal. Co transfer is also low. There may be hyperventilation with hypocapnea, hypoxemia. X-ray chest will show reticulonodular shadow and honeycomb lung. Bronchoalveolar lavage may show high neutrophil and high eosinophil.
 
Case No. 151
  1. Cushing's disease (due to pituitary dependent bilaterial adrenal hyperplasia).
  2. Serum ACTH, MRI of pituitary fossa.
Note: Commonest cause of Cushing's syndrome is iatrogenic. Otherwise in 80% cases, it is due to Cushing's disease. It is characterized by increased production of ACTH from pituitary gland, commonly microadenoma, < 10 mm. Failure of dexamethasone suppression occurs in ectopic ACTH syndrome and adrenal carcinoma.
Treatment: Transphenoidal removal of adenoma, if possible. Pituitary irradiation may be helpful. Occasionally, bilateral adrenalectomy may be necessary (later on, there may be Nelson's syndrome, which can be prevented by pituitary irradiation).
 
Case No. 152
  1. Idiopathic thrombocytopenic purpura.
  2. Bone marrow study, antiplatelet antibody.
  3. SLE
Note: In ITP, low platelet, increased megakaryocyte in bone marrow, prolonged bleeding time, normal clotting time are common. There may be antiplatelet antibody and anticardiolipin antibody. Initially, SLE and antiphospholipid syndrome may present like ITP. In 10% cases, ITP may be associated with autoimmune hemolytic anemia, called Evan's syndrome.
ITP is due to autoantibody directed against platelet membrane glycoprotein IIb- IIIa, responsible for removal of platelet by monocyte-macrophage system.
Treatment: In child-usually self-limiting. If no improvement, (1) Predni-solone (2 mg/kg), (2) If persistent bleeding, I/V immunoglobulin. (3) platelet transfusion, if persistent bleeding (epistaxis, GIT bleeding, retinal hemorrhage, intracranial bleeding). (4) If thrombocytopenia persists more than 6 months, it is chronic. In that case, splenectomy 399should be considered. In adult, persistent thrombocytopenia is common. (1) Prednisolone– 1 mg/kg for 4 to 6 weeks, then taper the dose. Relapse is common after withdrawal of steroid. Then, steroid dose should be increased. (2) If persistent bleeding, I/V immunoglobulin may be given (1 gm/kg for 3 to 5 days). Its effect is temporary, persists for 1 to 2 weeks. (3) In frequent relapse (usually more than 2 times), splenectomy may be considered (effective in 70% cases). But, if bleeding or purpura after splenectomy, low dose prednisolone (5 mg daily) should be continued. (4) In 30% cases, no response to splenectomy. Then, if the patient is asymptomatic and platelet count >40,000 to 50,000, follow-up the case. But if bleeding or purpura, low dose prednisolone (5 mg daily) should be continued. (5) If still no response, other therapy - danazol 600 mg daily. Or repeated IV immunoglobulin, azathioprine, cyclophosphamide, vincristine, vinblastine, ciclosporin, anti-D infusion, interferon-α.
If no response after splenectomy, may be accessory spleen (confirm by radionuclide scan). After splenectomy, infection by pneumococcus, meningococcus and H. influnzae (vaccination against these is essential).
 
Case No. 153
  1. Failure to rise urinary osmolality after 8 hours water deprivation test. After desmopressin, there is rise of urinary osmolality.
  2. Cranial diabetes insipidus.
  3. Diabetes mellitus, primary polydipsia.
  4. Head injury, sarcoidosis, hemochromatosis, histiocytosis X, idiopathic.
Note: Cranial diabetes insipidus is responsive to desmopressin, but nephrogenic diabetes insipidus does not respond to desmopressin. In CDI, plasma osmolality is usually > 300 mOsm/kg and urine osmolality is < 600 mOsm/kg. If urine osmolality rises by at least 50% after DDAVP, it is diagnostic of CDI.
Other causes of cranial diabetes insipidus- craniopharyngioma, pituitary tumor with suprasellar extension, basal meningitis, surgery, encephalitis, DIDMOAD syndrome (diabetes insipidus, diabetes mellitus, optic atrophy, deafness).
In cranial diabetes insipidus, if associated cortisol deficiency is present, there may not be any features until steroid replacement is given.400
 
Case No. 154
  1. Chronic pancreatitis with malabsorbtion.
  2. Chronic alcoholism.
  3. Plain X-ray abdomen to see pancreatic calcification, pancreatic CT scan, ERCP.
Note: High MCV and γ GT, both occur in chronic alcoholism, which is the likely cause of chronic pancreatitis. Diabetes mellitus secondary to chronic pancreatitis is common. Vitamin B12 deficiency may occur in chronic pancreatitis.
 
Case No. 155
  1. Tuberculous meningitis with SIADH.
  2. Chest X-ray, CT scan or MRI of brain.
  3. SIADH, Addison's disease.
Note: In tuberculous meningitis, low sodium may occur as a part of SIADH. Low sodium may also occur, if there is involvement of adrenal glands by tuberculosis, causing Addison's disease. Fundoscopy may show choroid tubercle.
 
Case No. 156
  1. Acetazolamide therapy.
  2. Hyperchloremic hypokalemic metabolic acidosis.
  3. Urine for ketone bodies, serum lactate level.
Note: Long term use of carbonic anhydrase inhibitor such as acetazolamide is an important cause of hyperchloremic metabolic acidosis associated with hypokalemia. Sugar is not so high to cause DKA. Lactic acidosis should be excluded also.
 
Case No. 157
  1. Antiphospholipid syndrome.
  2. ANA, anti-ds DNA, serum antiphospholipid or anticardiolipin.
Note: In any young patient, if presents with features like CVA or TIA, antiphospholipid syndrome should be excluded. In this case, APTT is prolonged, which is not corrected by addition of normal plasma. Antiphospholipid syndrome is characterized by the presence antiphospholipid antibody, causing thrombosis by an effect on platelet membrame, endothelial cell and clotting components such as 401prothrombin, protein C and S. Antiphospholipid antibody collectively included lupus anticoagulant and anticardiolipin antibody. In some patients, only one of these is positive and in others, both are positive.
Presence of this antibody may be associated with thrombosis (venous or arterial), recurrent abortion, thrombocytopenia, neurological (stroke, TIA, epilepsy, migrane, chorea), sterile endocarditis (Liebmann Sach's), livedo reticularis, pulmonary hypertension, avascular necrosis of head of femur. Renal involvement with proteinuria in 50%, in some cases, lupus nephritis (membranous) like lesion may be seen. Recurrent abortions are thought to be due to placental infarction.
Indirect evidence of antiphospholipid antibody—false positive VDRL, thrombocytopenia, prolonged prothrombin time, prolonged APTT (not corrected by normal plasma).
It may be primary or secondary to other disease. Secondary causes are SLE, rheumatoid arthritis, systemic sclerosis, Behcet's syndrome, temporal arteritis, Sjogren's syndrome, psoriatic arthropathy.
Treatment: Low dose aspirin (in mild to moderate case), warfarin in severe case. In pregnancy, heparin and low dose aspirin may be used.
 
Case No. 158
  1. Obstructive airway disease.
  2. Emphysema.
  3. α1 antitrypsin deficiency, cigarette smoking.
Note: Antitrypsin (antiproteinase) has a protective action for lung tissue. In smoking, damage of lung tissue occur due to an imbalance between proteinase and antiproteinase, release of oxidants and proteinase from inflammatory cells, which are responsible for damage to the supporting connective tissue of alveolar septa. Also, there is increased proteinase synthesis and inactivation of antiproteinase.
 
Case No. 159
  1. Serum electrolytes, Tensilon test.
  2. Alveolar hypoventilation.
  3. Myasthenia gravis, GBS, hypokalemia.
Note: In myasthenia gravis, weakness of muscles usually occur after exercise, not associated with muscular pain. Respiratory muscle involvement may cause dyspnea, respiratory failure, etc. Hypokalemia may be associated with severe muscular weakness or even paralysis. 402Respiratory muscle paralysis may occur in GBS. Repeated measurement of vital capacity and PEFR should be done, may require IPPR.
 
Case No. 160
  1. Acute myocardial infarction, pulmonary edema, pulmonary embolism, severe bronchial asthma.
  2. Chest X-ray, ECG, troponin 1.
Note: Hypocapnia is due to hyperventilation. Hypoxemia is due to ventilation/perfusion defect.
 
Case No. 161
  1. Leukemoid reaction.
  2. Neoplastic infiltration in bone marrow.
  3. Bone marrow study, isotope bone scan.
Note: Leukemoid reaction means the peripheral blood picture resembles leukemia, but there is no leukemia. It may be myeloid or lymphatic.
Causes of myeloid leukemoid reaction are leukoerythroblastic anemia, infection, malignancy, acute hemolysis (LIMA).
Causes of lymphatic leukemoid reaction are viral (infectious mononucleosis, cytomegalovirus infection, measles, chicken pox), whooping cough. Rarely, tuberculosis and carcinoma.
 
Case No. 162
  1. Addisonian crisis.
  2. Serum cortisol and ACTH.
Note: This patient used to take steroid for long time. Sudden stopping of steroid or following infection or stress, there may be Addisonian crisis. It is an acute severe adrenocortical insufficiency, characterized by features of collapse, with nausea, vomiting, diarrhea, acute abdomen, even unconsciousness. There is profound hypotension and hypoglycemia, electrolyte imbalance.
Causes are sudden withdrawal of steroid, if the patient was on steroid for long time. Others are stress (infection, operation).
Treatment: Three problems in addisonian crisis—(i) Hyponatremia, (ii) Hypoglycemia, (iii) Cortisol deficiency.
So, I/V normal saline, glucose, hydrocortisone 200 mg 6 hourly should be given. Control of infection, if any.403
 
Case No. 163 (see also Case No. 68)
  1. Infectious mononucleosis, cytomegalo virus infection, dengue, typhus fever.
  2. Antidengue antibody, Monospot test, antibody to EBV, antibody to CMV (others—urinary isolation of CMV, Weil Felix reaction).
Note: Infectious mononeucleosis is caused by EB virus. Cervical lymphadenopathy (commonly posterior), splenomegaly and skin rash specially after ampicillin or amoxycillin are typical features.
Other diseases caused by EB virus are- Burkitt's lymphoma, nasopharyngeal carcinoma, hairy leukoplakia in AIDS, immunoblastic lymphoma in AIDS, posttransplant lymphoma and Hodgkins lymphoma.
Causes of fever with skin rash- chicken pox, scarlet fever, measles, rubella, typhus, enteric fever, dengue fever, drugs.
 
Case No. 164
  1. Black water fever.
  2. Chloroquine or quinine, blood transfusion, steroid.
  3. Hemolytic uremic syndrome.
Note: In a patient with high fever, signs of hemolysis with black urine in endemic area, suggest black water fever. It is a severe manifestation of falciparum malaria that occurs in previously infected person. It is characterized by sudden intravascular hemolysis and hemoglobinuria. Urine looks black. Occurs in those who took anti-malarial drug irregularly or in non-immune person, who took irregular anti-malarial drug prophylaxis. It is due to antibody production formed against red cells that have been altered by drug, parasite or both.
Treatment: Quinine IV, 20 mg/kg (maximum 1.4 gm), with 5% D/A, for 4 hours, then 10 mg/kg 8 hourly (maximum 700 mg) for 7 days, until patient can take orally. Injection artemether is a suitable alternative. Dose—80 mg IM twice daily for one day, followed by 80 mg daily for 4 days or 80 mg IM twice daily for 3 days (it is a synthetic anti-malarial, derived from artemisinin. It should be avoided in pregnancy, unless strongly indicated). Blood transfusion may be necessary. Prednisolone may be helpful.
 
Case No. 165
  1. Clubbing, signs of polycythemia (plethoric face, blood shot eyes).
  2. Cyanotic congenital heart disease, cystic fibrosis.
  3. Chest X-ray, echocardiogram.
    404
Note: Fallot's tetralogy is the commonest congenital heart disease which presents with short stature, cyanosis and clubbing. Other diseases are transposition of great vessels, tricuspid atresia, aortic and mitral atresia.
Cystic fibrosis may be associated with bronchiectasis, cyanosis and clubbing.
 
Case No. 166
  1. Sarcoidosis, hypervitaminosis D, hypercalcemia associated with malignancy.
  2. Primary hyperparathyroidism.
Note: Failure of suppression of calcium after hydrocortisone is suggestive of primary hyperparathyroidism. In tumor associated hypercalcemia, calcium may not fall in 50% cases. In other cases, there is suppression of calcium.
 
Case No. 167
  1. Myxoedema coma.
  2. Serum TSH and cortisol.
Note: In this patient, typical history, low voltage ECG and cardiomegaly is likely to be due to myxoedema, associated with pericardial effusion. Myxoedema coma is rare, common in elderly, characterized by depressed level of consciousness, low body temperature, even convulsion. Coma may be due to SIADH, hypoxemia, hypercapnia, hypothermia, hypoglycemia, infection. Cardiac failure may occur. 50% mortality. CSF shows high pressure with high protein.
Treatment: IV T3 (20 μgm) eight hourly. After 48 to 72 hours, oral thyroxine may be given. If T3 is not available, oral thyroxine. hydrocortisone IV 100 mg eight hourly.
Other treatment: Oxygen, slow rewarming, antibiotic, glucose infusion, assisted ventilation.
 
Case No. 168
  1. DIC.
  2. Serum FDP, D-dimer, serum fibrinogen.
Note: After myocardial infarction, there may be release of thromboplastic substance into the circulation, which activates extrinsic clotting system. In DIC, prothrombin time, APTT, thrombin time are prolonged. Platelet 405count and fibrinogen are all low. FDP and D-dimer are high. Other clotting factors are also low.
 
Case No. 169
  1. Juvenile hypothyroidism.
  2. Thyroxine.
Note: Prolactin is high in hypothyroidism. Pituitary fossa may be enlarged in hypothyroidism due to increased thyrotroph. Hyperprolactinemia may occur secondary to multiple causes. Commonest cause is prolactinoma of the pituitary gland. Other causes- (i) Physiological- pregnancy, lactation, sleep, stress, (ii) Pathological- disease of pituitary gland (prolactinoma, acromegaly), hypothalamic disorder (tumor, trauma, radiation), hypothyroidism, renal failure, chest wall injury, ectopic production by non-endocrine tumor (bronchial carcinoma, renal cell carcinoma), drug (metoclopramide, domperidone, phenothiazine, butyrophenones, antidepressant, pimozide, methyldopa, estrogen), idiopathic. Hyperprolactinemia may present with- (i) In female—amenorrhea, galactorrhea, infertility, dyspareunia (ii) In male— decreased libido, reduced frequency of shaving, lethargy. Prolactin stimulates milk secretion, not breast development. So, galactorrhea never occurs in male. Gynecomastia is present, if hypogonadism.
Causes of enlarged sella turcica (i) Pituitary macroadenoma, (ii) Hypothalamic mass or cyst, (iii) Aneurysm, (iv) Primary hypo-thyroidism, (v) Hypogonadism, (vi) Increased intracranial pressure, (vii) Empty sella syndrome.
Empty sella syndrome – it is charaterized by enlargement of pituitary fossa due to herniation of suprasellar sub-arachnoid space through incomplete diaphragma sella, so the sella is filled with CSF with arachnoid sac. The pituitary gland is pushed to one side, but function normally. The patient is usually obese, multipara having headache, hypertension (30%), hyperprolactinemia, CSF rhinorrhea, pseudotumor cerebri, visual field defect, etc. Endocrine function is normal. Diagnosis- MRI. Treatment- reassurance.
 
Case No. 170
  1. Restrictive lung disease.
  2. Fibrosing alveolitis, sarcoidosis, lymphangitis carcinomatosa.
Note: In restrictive lung disease, the ratio of FEV1/FVC is normal, but low PEF and low TLC, low transfer factor for carbon monoxide.406
 
Case No. 171
  1. Hyperkalemia.
  2. It is more likely to be spurious or pseudohyperkalemia. Probable cause is blood sample having been left for long time before biochemical analysis. As a result, red cell potassium leaks into the plasma, leading to falsely high level of potassium.
  3. Repeat serum potassium.
Note: Pseudohyperkalemia commonly occurs, if the blood sample is kept for long time. This may also be found in chronic lymphatic leukemia, myeloproliferative disorders, etc. in which leakage of potassium from high leukemic cells leads to high potassium. Rarely, high potassium may occur in familial pseudohyperkalemia and hypoaldosteronism.
 
Case No. 172
  1. Essential thrombocythemia with pulmonary embolism.
  2. Chest X-ray PA view, bone marrow study.
  3. Pleurisy.
  4. Pleural rub.
Note: Thrombocytosis may occur in- acute blood loss, hemolysis, postsplenectomy, CGL.
Essential thrombocythemia is a myeloproliferative disease with very high production of platelet. Platelet aggregation is impaired, blood film shows large or atypical platelet and megakaryocyte fragments. Bone marrow shows increased megakaryocyte. There is splenomegaly, hemorrhage and thromboembolic episodes.
 
Case No. 173
  1. MEN type 2a.
  2. Thyroid function test (radioiodine uptake test, FT3, FT4 and TSH, FNAC), USG of abdomen to see suprarenal gland, 24 hours urine for VMA, subtraction scan of thyroid and parathyroid (with thallium and technetium).
Note: In this patient, there is goiter, more likely to be medullary carcinoma of thyroid (MTC), high calcium (hyperparathyroidism) and periodic hypertension (pheochromocytoma), which suggest MEN type 2a. It consists of primary hyperparathyroidism, medullary carcinoma of thyroid and pheochromocytoma, also called Sipple's syndrome.
In MTC, serum calcitonin is high.407
In hyperparathyroidism, high calcium, low phosphate and high alkaline phosphatase.
In pheochromocytoma, high serum catecholamines, high 24 hours urine VMA, etc.
Treatment: Individual tumor should be treated surgically.
MEN type 2b: When there is associated Marfanoid body habitus, skeletal abnormality, multiple mucosal neuroma, abnormal dental enamel.
MEN type I (Werner's syndrome): Pituitary tumor, primary hyperpara-thyroidism and pancreatic neuroendocrine tumor (glucagonoma, insulinoma).
 
Case No. 174
  1. Turner's syndrome.
  2. Karyotyping.
Note: Turner's syndrome is associated with primary amenorrhea and short stature. It is characterized by a single X chromosome (45, X0). The main abnormality is gonadal dysgenesis, there is aplasia of ovaries (streak gonad), responsible for primary amenorrhea and infertility.
It is diagnosed by karyotyping from buccal smear which shows 45 (X0), occasionally 46 (XX) mosaic. There is low estrogen, high LH and FSH. USG shows small uterus, small fallopian tube and streak gonad.
Features of Turner's syndrome are:
  1. Skeletal: Short strature, short and webbing of neck, small lower jaw (micrognathia), mouth is small and fish like, high arched palate. Chest- broad, widely apart nipples (shield-like chest). Hand- short 4th metacarpal (other metacarpals may be short), lymphoedema of hands (also feet), nails- hypoplastic. Elbow- increased carrying angles (cubitus valgus).
  2. Cardiac abnormality: Coarctation of aorta (10 to 20% cases), ASD, VSD, AS, Hypertension.
  3. Renal abnormality: Horse shoe shaped kidney, hydronephrosis.
  4. Miscellaneous: Low hairline and redundant skin fold on the back of neck, low set and deformed ears. Others (incidence is more)- Diabetes mellitus, Hashimoto's thyroiditis (may be frank hypothyroidism in 20% cases), lymphoedema in infancy, red green color blindness, strabismus and ptosis, premature osteoporosis, pigmented naevi, mental retardation (rare).
    408
 
Case No. 175
  1. Budd-Chiari syndrome.
  2. Chronic constrictive pericarditis, congestive cardiac failure, inferior vena caval obstruction.
  3. LFT (bilirubin, SGPT, alk.phosphatase, prothrombin time), USG of HBS, CT scan or MRI of liver, technetium scan of liver.
Note: SLE with recurrent abortion is associated with antiphospholipid syndrome, which may cause venous thrombosis. Sudden and rapid enlargement of liver with ascites is highly suggestive of hepatic venous obstruction, called Budd-Chiari syndrome.
Other causes of thrombosis causing this syndrome are polycythemia rubra vera, PNH, oral contraceptive pill, deficiency of antithrombin III, protein C and S, malignancy (such as hepatic, renal or adrenal carcinoma, posterior wall abdominal sarcoma), congenital venous web, hydatid cyst, radiotherapy, trauma to liver. It is idiopathic in 30% cases. There may be cirrhosis of liver.
Investigation: Color Doppler USG or CT scan or MRI- will show hepatic venous occlusion with diffuse abnormal parenchyma on contrast enhancement, sparing caudate lobe because of its independent blood supply and venous drainage. Technetium scan of liver shows low uptake, but there may be excess uptake in caudate lobe, because it has separate venous drainage from the rest of the liver. Other investigations shoud be done to find out causes.
Treatment: (i) Treat primary cause, (ii) For ascites- diuretic, sodium and water restriction, paracentesis. (iii) Thrombolytic, such as streptokinase may be given in acute case, (iv) TIPSS- transjugular intrahepatic portosystemic shunt, (v) Liver transplantation may be necessary in chronic case and for fulminant failure.
 
Case No. 176
  1. SLE.
  2. ANA, anti-ds DNA.
Note: Flitting arthritis associated with high ESR, thrombocytopenia, positive VDRL in young female is highly suggestive of SLE. Flitting arthritis may occur in rheumatic fever, SLE, viral fever, lyme disease, chronic active hepatitis, secondary syphilis, etc. False positive VDRL may occur in atypical pneumonia, malaria, bacterial or viral infection, SLE, leprosy, rheumatoid arthritis, primary biliary cirrhosis.409
 
Case No. 177
  1. von Hippel-Lindau syndrome.
  2. MRI of brain.
Note: von Hippel-Lindau syndrome is characterized by combination of cerebellar signs and polycythemia. It is inherited as autosomal dominant, due to defective gene on chromosome 3p25-26. There is retinal and intracranial (cerebellar) hemangioma and hemangioblastoma associated with increased erythropoietin production causing polycythemia. Extracranial hamartoma lesion may occur which may undergo malignant change. 10% posterior cranial fossa tumors are hemangioblastoma.
 
Case No. 178
  1. DIC.
  2. Serum FDP, D-dimer, serum fibrinogen, prothrombin time, APTT.
  3. Antibody to CMV, antibody to EB virus, isolation of CMV from urine.
  4. Post-transfusion cytomegaloviral infection, infectious mono-nucleosis.
Note: Blood transfusion may be associated with any infection, such as CMV, infectious mononucleosis, hepatitis B and C, HIV, etc. DIC may occur after massive blood transfusion.
 
Case No. 179
  1. Primary hyperaldosteronism.
  2. Serum aldosterone and renin, 24 hour urine potassium, CT scan (or MRI of adrenal glands).
Note: Hypertension with hypokalemic alkalosis is highly suggestive of primary hyperaldosteronism, also called Conn's syndrome. It is an aldosterone secreting tumor, responsible for <1% cause of hypertension. Excess aldosterone secretion leads to sodium retention, hypokalemia and combination of hypertension with hypokalemia. It is due to adrenal adenoma in 60% (Conn's syndrome) and 30% bilateral adrenal hyperplasia. Adrenal adenoma is usually small, common in young female. Adrenal hyperplasia is common in male, after forty years. Serum renin is low in primary aldosteronism, whereas renin is high in secondary aldosteronism.
Investigations: (i) High sodium, low potassium and high bicarbonate (metabolic alkalosis), (ii) Urine potassium >30 mmol in 24 hours, 410(iii) High aldosterone and low renin, (iv) Failure of suppression of aldosterone by giving 0.9% sodium chloride 300 cc for four hours, (v) CT or MRI to localise tumor, (vi) 131iodo-nor-cholesterol scanning of adrenal may be done, (vii) Selective venous catheter of adrenal for aldosterone measurement, (viii) Urine for 18-OH cortisol is increased.
Simultaneous measurement of renin and aldosterone measurement should be done in lying and standing position. Renin is low and aldosterone is high in supine position. On standing suddenly, there is paradoxical drop of aldosterone in patient with adenoma and exaggerated rise in aldosterone in patient with hyperplasia.
Treatment: If adenoma, surgery. If hyperplasia- spironolactone 100 to 400 mg daily. For hypertension, amiloride and calcium channel blocker may be used.
Rare tumor with hypokalemia and hypertension occur in 11-β-hydroxylase deficiency in CAH, which respond to prednisolone and dexamethasone. Still rare, 17-hydroxylase deficiency (CAH plus hypokalemia plus hypogonadism), also respond to steroid.
Causes of hypokalemic alkalosis and hypertension – Primary hyperaldosteronism (Conn's syndrome), Cushing's syndrome, Liddle's syndrome, accelerated hypertension, hypertension treated with diuretics, renal artery stenosis, carbenoxolone therapy, liquorice abuse, congenital adrenal hyperplasia (11-β-hydroxylase deficiency).
 
Case No. 180
  1. Microangiopathic hemolytic anemia.
  2. Sulfasalazine induced.
Note: Sulfasalazine may cause blood dyscrasia such as agranulocytosis, megaloblastic anemia, hemolytic anemia, occasionally Heinz body hemolytic anemia. It can also cause GIT upset, skin rash, Stevens Johnson syndrome, reversible sterility in male.
 
Case No. 181
  1. Polycystic ovarian syndrome, late onset congenital adrenal hyper-plasia, Cushing's syndrome, virilizing tumor of ovary or adrenal.
  2. Ultrasonography of ovary and suprarenal glands.
  3. Serum testosterone, LH, FSH, androgen (androstenedione, dihydro-epiandrosterone), SHBG, 17-hydroxyprogesterone.
    411
Note: Any patient with hirsutism, first the above hormones should be measured. Combination of obesity, hirsutism and amenorrhea or oligomenorrhea, infertility is highly suggestive of polycystic ovarian syndrome. There may be insulin resistance.
In PCOS, there is high testosterone, low SHBG, high LH, normal or low FSH and LH:FSH > 2. Free androgen are also high. Mild rise of prolactin.
PCOS may be confused with late onset congenital adrenal hyperplasia, in which serum 17-hydroxyprogesterone and ACTH are high, urine pregnanetriol is also high.
It is also confused with virilizing tumor of adrenal or ovary.
 
Case No. 182
  1. Malabsorption syndrome.
  2. Barium meal and follow through X-ray, jejunal biopsy, fecal fat estimation.
  3. Osteomalacia.
Note: Frequent diarrhea, weight loss with combination of nutritional deficiency should always raise the possibility of malabsorption syndrome. In this case, both macrocytic and microcytic anemia, low calcium, low albumin indicate malabsorption. Proximal myopathy is due to osteomalacia.
 
Case No. 183
  1. Acute promyelocytic leukemia with DIC.
  2. Serum fibrinogen, FDP.
  3. Relatively good.
Note: Initial blood picture shows pancytopenia. Bone marrow is hypercellular with predominant promyelocyte, which indicates acute promyelocytic leukemia (AML type 3). DIC occurs only in this type of leukemia. Bone marrow may show hypergranular blast cells. The granules contain a procoagulant, which triggers DIC. Reciprocal translocation of the long arms of chromosome 15 and 17 is the basic genetic defect in the type of leukemia.
Treatment: All-trans-retinoic acid (ATRA) plus chemotherapy. Arsenic trioxide is also effective. Allogenic transplantation may be necessary. Complete remission in 80% cases in young, 60% may be cured. Lysis of blast cells during chemotherapy may worsen DIC, which needs 412appropriate support with platelet and fresh frozen plasma. Once in remission, acute promyelocytic leukemia has good prognosis.
 
Case No. 184
  1. Acromegaly.
  2. Wilson's disease, renal failure, heroin abuse.
Note: Normally, during GTT, with the rise of glucose, there is fall of growth hormone. Paradoxical rise of growth hormone during GTT occurs in acromegaly in 50% cases. However, it may occur in other cases such as- Wilson's disease, renal failure, heroin abuse, etc.
 
Case No. 185
  1. Hypopituitarism due to Sheehan's syndrome.
  2. Serum LH, FSH, ACTH, TSH and FT3, FT4, MRI of pituitary.
Note: Postpartum pituitary necrosis is called Sheehan's syndrome, which develops following severe postpartum hemorrhage and features of hypopituitarism. There is failure of lactation, persistent amenorrhea, atrophy of breast, reduced pubic, axillary and body hair, skin is pale, soft, fine and wrinkled. With progressive hypopituitarism, at first there is loss of growth hormone, then LH, FSH, ACTH, TSH. Fatigue and loss libido are early features of gonadotrophin deficiency. Gradual loss of secondary sexual characters and reduced muscle bulk occur later. Secondary hypothyroidism occur without myxoedema. Diabetes insipidus develops due to lack of ADH, though impaired glomerular filtration caused by cortisol deficiency may mask the symptomps. Coma in hypopituitarism may be due to- (i) Hypoglycemia, (ii) Hyponatremia, (iii) Water intoxication, (iv) Hypothyroidism, (v) Hypothermia.
Low sodium in hypopituitarism is due to SIADH, caused by cortisol deficiency. Potassium is usually normal, as aldosterone is normal or high.
Treatment: (i) Hydrocortisone 15 mg in morning and 5 mg in afternoon, (ii) Thyroxine (it is dangerous to start thyroxine without giving hydrocortisone, Addisonian crisis may occur), (iii) Hormone therapy in premenopause- estrogen (1 to 21 days) plus progesterone (day 14 to 21), (iv) For fertility- gonadotrophin may be used.
 
Case No. 186
  1. Aspirin therapy.
  2. Stop the aspirin.
    413
Note: Prolonged use of low dose aspirin may be associated with bleeding tendency, prolonged prothrombin time, prolonged bleeding time, prolong APTT, etc.
 
Case No. 187
  1. Lateral medullary syndrome (also called Wallenberg syndrome).
  2. MRI of brain.
  3. Horner's syndrome in left eye, cerebellar sign in left side, pain and temperature on opposite side.
Note: Lateral medullary syndrome is characterized by ipsilateral- (i) Horner's syndrome (lesion in descending spinothalamic tract lesion), (ii) Cerebellar signs (lesion in cerebellum and its connection), (iii) palatal palsy and diminished gag reflex (dysphagia and hoarseness due to IXth, Xth nerve involvement, (iv) decreased pain and temperature (Vth nerve nucleus and its descending tract lesion).
Contralateral decrease of pain and temperature (spinothalamic tract lesion).
This syndrome is produced by infarction of small wedge of lateral medulla posterior to inferior olivary nucleus, due to occlusion of posterior inferior cerebellar artery (PICA). Occlusion of any of the vessels may be responsible- (i) vertebral, (ii) posterior inferior cerebellar, (iii) superior, middle or inferior lateral medullary arteries.
In the majority of cases of lateral medullary syndrome, there is also occlusion of vertebral artery and pyramidal signs are present.
Vomiting is due to the involvement of nucleus ambiguus, hiccough is due to the lesion in reticular formation and vertigo is due to the involvement of vestibular nuclei.
Rarely, occlusion of lower basilar artery, vertebral artery or one of its medial branches produce medial medullary syndrome, characterized by contralateral hemiplegia which spares the face, contralateral loss of vibration and joint position sense and ipsilateral paralysis and wasting of tongue.
 
Case No. 188
  1. Hypothyroidism.
  2. Serum TSH estimation.
Note: Gradual weight gain, ascites, pericardial effusion, high MCV low sodium and hyperlipidemia with high CPK, all suggest hypothyroidism. Polyserositis may occur in myxoedema. Muscular pain, arthralgia, 414effusion in any serous cavity may occur. SIADH, high CPK, SGPT, SGOT, LDH may occur in myxoedema.
Other causes of polyserositis- disseminated TB, SLE, polyarteritis nodosa, disseminated malignancy and viral fever (dengue).
 
Case No. 189
  1. MCTD (or overlap syndrome).
  2. ANA, anti-ds DNA, anti-scl 70.
  3. Anti-RNP antibody.
Note: MCTD is a group or combination of features of systemic sclerosis, SLE and polymyositis (or other collagen disease, e.g. rheumatoid arthritis). It is better to be called overlap syndrome.
Commonly occurs in female, third to fourth decade, rare in children and elderly. Usual features are synovitis, edema of hands, Raynaud's phenomenon and muscular pain or weakness.
Lung involvement occurs in 85% cases, but frequently asymptomatic. CO transfer is diminished only. Pleurisy is common. Pulmonary hypertension is the common cause of death. In 25% cases, renal disease usually membranous glomerulonephritis may occur. Gastrointestinal involvement occurs in 70% cases. In heart, pericarditis (30%), myocarditis, arrhythmia, mitral valve prolapse may occur.
The serum shows high anti-RNP antibody. ANA may be positive with a speckled nucleolar pattern. There may be positive anti-scl 70, CPK may be high. Other investigations are- CBC (ESR is high), X-ray chest (may show reticulonodular shadow), barium swallow, skin biopsy etc.
Treatment: Prednisolone which responds quickly. 10 years survival in 80% cases.
 
Case No. 190
  1. Endoscopy and biopsy, serum CPK, EMG, muscle biopsy.
  2. Dermatomyositis with carcinoma of esophagus.
Note: This patient is likely to be suffering from carcinoma of esophagus. His skin manifestations is due to dermatomyositis. It is associated with malignancy occurs in 5 to 8% cases, as non-metastatic manifestation of malignancy. More in male. It occurs commonly in oat cell carcinoma of bronchus. There is also association with malignancy of ovary, breast, stomach which may predate the onset of myositis. Associated cancer may not become apparent for 2 to 3 years. Recurrent 415or refractory dermatomyositis should prompt a search for occult malignancy. Muscular pain and tenderness are found in 50% cases. Proximal myopathy, wasting are common. Ocular muscle involvement is rare.
Treatment of the primary cause may improve dermatomyositis.
 
Case No. 191
  1. Pheochromocytoma.
  2. 24 hour urinary VMA, urinary metanephrine, abdominal CT scan or MRI.
Note: Pheochromocytoma is characterized by recurrent, paroxysmal attack of hypertension due to increased release of catecholamines. It is the rare tumor of chromaffin tissue, that secretes catecholamines responsible for 0.1% cases of hypertension. Commonly in adrenal medulla (90%), but may occur in any part of sympathetic chain.
Rule of ‘10’– 10% malignant, 10% extraadrenal (in sympathetic chain), 10% familial. May be associated with MEN Type II (primary hyperparathyroidism, medullary carcinoma of thyroid and pheochromo-cytoma, also called Sipple's syndrome) or MEN Type II b (above plus Marfanoid habitus, skeletal deformity, neuroma of lip, tongue, conjunctiva, eyelid). 25% may be multiple. Hypotension may occur with dopamine secreting tumor.
It may present with pallor, fear of death or anxiety or panic attack or acute medical crisis (myocardial infarction, CVA, acute renal failure, paralytic ileus). Postural hypotension occurs in 70%, glycosuria in 30%.
Most tumors release adrenaline and noradrenaline, but extra-adrenal and large tumor release mainly noradrenaline. Phaeochromocytoma may occur in urinary bladder and micturition can precipitate an attack. Commonest extra-adrenal tumor is at bifurcation of aorta (organ of Zuckerkandl).
Other investigations: Serum and urine catecholamines, meta-iodobenzyl-guanidine scan (MIBG) helpful for extra-adrenal tumor (it is selectively taken by adrenergic cells).
Treatment: Surgical resection. For hypertension, α-blocker (phenoxy-benzamine 10 to 20 mg 6 to 8 hourly). If tachycardia, β-blocker may be added (only β-blocker should not be given, as it may cause crisis). Malignant pheochromocytoma is a slowly growing tumor and the patient may survive for long time.416
 
Case No. 192
  1. Anorexia nervosa.
  2. Psychotherapy, controlled supervised diet to increase the weight 1 kg weekly.
Note: It is common in young girl, onset between 16 to 17 years, rare after 30 years. Less in male (M:F = 1:10). More in higher social class, the girl is hard working, perfectionist and ambitious. Endocrine abnormality is present, which reverts to normal after improvement of the disease.
Diagnostic criteria are (i) Weight loss of at least 15% of the expected body weight, (ii) Avoidance of high calorie diet, (iii) Distortion of body image, so the patient regards herself fatty even when she is thin or grossly underweight, (iv) Amenorrhea for at least 3 months (in male, loss of sexual interest replaces amenorrhea).
There is profound body image disturbance so that despite emaciation, patient still feels overweight and terrified of weight gain. The patient may hide her emaciation by using loosely fitting clothes. Physically overactive, performs excessive exercise, may use laxative or diuretic and sometimes vomit after meal. There may be downy, lanugo hair on trunk and limb. Hypotension, bradycardia, increased sensitivity to cold, constipation, peripheral cyanosis may be found. Anxiety, depression are common. Psychosexual immaturity present. Osteoporosis may occur due to less estrogen. Bilateral parotid enlargement may be present.
Low LH, low FSH, low estradiol, low T3, but normal T4 and TSH, high cortisol and high GH are present. In male, low LH and testosterone. Dexamethasone suppression test may be abnormal. All revert to normal after therapy. Glucose intolerance may occur due to starvation.
Treatment: (i) In mild case- treated in outdoor basis, (ii) Moderate to severe case- hospitalization, (iii) Controlled diet to increase the weight 1 kg weekly, (iv) Psychotherapy.
Prognosis: 50% full recovery, 30% partial recovery and 20% none. 2 to 5% death from suicide or physical complication.
 
Case No. 193
  1. Carcinoid syndrome.
  2. USG of abdomen (to see hepatic metastasis), color Doppler echocardiography, 24 hours urine for 5-HIAA (hydroxy-indolo acetic acid).
    417
Note: In this patient, recurrent attack of diarrhea associated with flushing of face and respiratory distress is consistent with the diagnosis of carcinoid syndrome. Presence of firm, irregular liver is suggestive of metastasis in the liver. Carcinoid tumor is derived from enterochromaffin cells, 90% found in GIT (common in ileum, also appendix, rectum) and 10% in the lung. In appendix, it is usually benign, presents as appendicitis (10%).
Carcinoid tumors are asymptomatic, until metastasis. Only 5% develop carcinoid syndrome, when there is metastasis to the liver. Episodic release of 5-HT and other neuroendocrine mediators (such as bradykinin, histamin, tachykinin, prostaglandin) released from the tumor produce the symptoms, when enter into the systemic circulation.
The features are- recurrent attack of flushing, wheezing, abdominal pain, recurrent diarrhea, vomiting, pellagra and photosensitive dermatitis may occur. Flushing is hallmark. There may be hypotension, bradycardia, facial edema. Cardiac abnormalities are found in 50%, such as tricuspid regurgitation, pulmonary stenosis and endocardial plaques, which may lead to heart failure. Left sided cardiac valves are not affected, but bronchial carcinoid causes left sided valvular lesion. Cardiac involvement is a very poor prognostic marker.
Unexplained right sided heart failure with periodic flushing, wheezing and hypotension is highly suggestive of carcinoid syndrome.
Diagnosis: 24 hours urine for 5-HIAA (5-hydroxy-indole-acetic acid).
Treatment: Surgery or embolization for solitary liver metastasis or bronchial carcinoid. Octreotide or lanreotide improves the syndrome in 90%. Long acting octreotide sometimes inhibits tumor growth. Interferon and chemotherapy may be used which reduces tumor growth, but does not prolong survival. Cyproheptadine and methysergide may help in diarrhea. Nicotinamide is helpful for pellegra. Survival is 5 to 10 years.
 
Case No. 194
  1. Turner's syndrome.
  2. Karyotyping from buccal smear.
  3. Coarctation of aorta.
Note: This patient presents with persistent amenorrhea with short stature. Estrogen is low, but LH and FSH are high, which suggests primary hypogonadism. The most probable diagnosis is Turner's syndrome, which is a sex chromosomal abnormality, characterized by 418the absence of one X chromosome (45, X0). Diagnosis is confirmed by a buccual smear which reveals an absent Barr body. The patient presents with amenorrhea and short stature. Other features are- (i) Neck- short, webbing, low hair line, (ii) Face- small lower jaw, small fish like mouth, low set ears, (iii) Chest- shield like, (iv) Hand- short 4th metacarpal, lymphoedema, (v) elbow- increased carrying angle. Normal I.Q.
Investigation: (i) Karyotyping from buccal smear (45, X0 is classical, 46 XX in mosaic), (ii) Ultrasonogram (small uterus, fallopian tube, streak gonad), (iii) Low estrogen, but high LH and FSH.
Association with Turner's syndrome- Coarctation of aorta (10 to 20%), horse-shoe shaped kidney, diabetes mellitus, Hashimoto's thyroiditis and hypothyroidism.
 
Case No. 195
  1. Multiple sclerosis.
  2. MRI of brain.
  3. Retrobulbar neuritis.
Note: There are optic neuritis, nystagmus (cerebellar), scanning speech, monoplegia (UMN type), suggestive of multiple sclerosis. History of recurrent attack is common (relapsing and remitting in 80 to 90%).
Multiple sclerosis is a demyelinating disorder characterized by the involvement of optic tract, pyramidal tract, cerebellar peduncle and posterior column of spinal cord. Usual presentations are- (i) Weakness of one or more limbs, (ii) Spastic paraplegia (confused with spinal cord compression), (iii) Cerebellar signs, (iv) Brainstem dysfunction- vertigo, diplopia, nystagmus, dysphagia, pyramidal signs in limbs, (v) Bladder dysfunction- incontinence, dribbling, hesitency, (vi) Others (rarely)- epilepsy, trigeminal neuralgia, facial palsy (may be recurrent), 6th nerve palsy, tonic spasm or brief spasm of limbs, dementia, organic psychosis, depression, (vii) Euphoria despite of disability.
Diagnosis of multiple sclerosis is usually clinical. It should be considered in any patient who presents with neurological features that are scattered in time (2 or more separate episodes) and space (2 or more separate locations). There is no single test for diagnosis, but MRI scanning with clinical findings are highly suggestive. CSF study shows oligoclonal bands.
Treatment: Optic neuritis and other acute features that are serious enough to cause distress or limitation of actitivies, should be treated 419with high dose steroid. Intravenous or oral methylprednisolone for 3 to 5 days is the treatment of choice.
 
Case No. 196
  1. Alzheimer's disease (AD).
  2. Multi-infarct dementia, Creutzfeld-Jakob disease.
Note: AD is a primary degenerative brain disease of unknown cause characterized by gradual deterioration of memory and progressive dementia.
Key features: Inability to remember information acquired in the past. Both short and long term memory are involved, commonly short term. In early stage, patient may complain of problem, but in later stage, usually deny of any problem (anosognosia).
Pathology: Macroscopically, there is diffuse atrophy of brain, particularly cerebral cortex and hippocampus with secondary enlargement of ventricular system. Microscopically- (i) Neuritic senile plaque containing Ab amyloid, (ii) Silver staining neurofibrillary tangles in neuronal cytoplasm of cerebral cortex, (iii) Accumulation of Ab amyloid in arterial wall of cerebral blood vessels.
Biochemical abnormality: Impairment of acetylcholinesterase trans-mission. Other neurotransmitters such as noradrenaline, 5HT, glutamate and substance P are also involved.
Treatment: Usually acetylcholinesterase inhibitor drugs are used (donepezil, galantamine, rivastigmine). Another drug- memantine (affects glutamine transmission) may be used. Anti-depressant may be needed.
 
Case No. 197
  1. MRI of brain, EEG, lumbar puncture and CSF study (also TPHA).
  2. Multiple sclerosis, Alzheimer's disease and Creutzfeld-Jakob disease.
  3. Creutzfeld-Jakob disease.
Note: Creutzfeld-Jakob disease is a slow viral encephalopathy which leads to rapidly progressive dementia with myoclonus, multifocal neurological signs including aphasia, cerebellar ataxia, cortical blindness, spasticity and extrapyramidal signs. Common in middle aged and elderly.
It is characterized by profound neuronal loss, astrocytosis, and a typical spongioform degeneration of the brain. EEG is typical which shows repetitive slow wave complex.420
Iatrogenic transmission to human may occur by surgical specimen, autopsy, corneal graft, depth EEG electrode, neurosurgery (cadaveric dura mater graft), pooled cadaveric growth hormone.
No treatment is available. Quinarine is in trial. The disease usually rapidly progresses, leading to death within 4 to 6 months.
One variant of CJD called vCJD, rare variety common in young age, slowly progressive, prolonged course. Early symptoms are neuropsychiatric, followed by ataxia, dementia, myoclonus or chorea.
 
Case No. 198
  1. Pseudomembranous colitis (secondary to antibiotic).
  2. Stool for RME and C/S, isolation of clostridium difficile toxin (A or B) from stool by ELISA, colonoscopy.
  3. Antibiotic is stopped, correction of dehydration. Add metronidazole 400 mg TDS or vancomycin 125 mg QDS.
Note: If a patient following antibiotic therapy developes bloody diarrhea, pseudomembranous colitis should be considered. It is an inflammatory condition due to clostridium diffcile. 5% healthy adults and 20% elderly are healthy carrier of this organism. Occurs by using any antibiotic, commonly cephalosporin, ampicillin, amoxycillin and clindamycin. Common in the first few days of using antibiotic or even upto 6 weeks after stopping drug. Two types of toxins- A (enterotoxin) and B (cytotoxic) are responsible. It is common (80%) in elderly, above 65 years. Colonic mucosa may be ulcerated, occasionally covered by creamy white membrane like material, so it is called pseudomembranous colitis. May be confused with ulcerative colitis. Complications such as toxic dilatation, perforation, ileus may occur.
Diagnosis: (i) Detecting A or B toxin in stool by ELISA, (ii) Stool culture is postive in 90% cases, (iii) Colonoscopy- erythema, wide plaque, adherent pseudomembrane. Biopsy should be taken.
Treatment: (i) Offending drug should be stopped, (ii) Metronidazole 400 mg 8 hourly or vancomycin 125 mg qds for 7 to 10 days, (iii) In severe case, IV immunoglobulin may be given.
 
Case No. 199
  1. Ulcerative colitis, Crohn's disease, angiodysplasia of the colon.
  2. Barium enema, colonoscopy and biopsy, color Doppler echo-cardiogram (others- prothrombin time, APTT).
  3. Angiodysplasia of colon.
    421
Note: Angiodysplasia is characterized by vascular malformation, which consists of ectatic areas of mucosal microvessels, specially of capillaries and venules associated with dilatation of submucosal veins. It is common in cecum and ascending colon, but may occur in upper and lower intestine.
Common in elderly, >70 years. The patient usually presents with fresh blood per rectum, which is acute and profuse, usually stops spontaneously but commonly recurs. Angiodysplasia may be associated with aortic stenosis, chronic renal failure, von Willebrand's disease.
Cause- unknown. Dysplasia are degenerative lesions arise from chronic colonic muscular contraction that obstructs the venous mucosal drainage. Later, mucosal capilaries dilate and incompetent with formation of arterio-venous communication.
Best way of diagnosis is by colonoscopy, which shows vascular spots like spider naevi. Selective mesenteric angiography may be helpful.
Treatment: Vessels are obliterated by electrocautery or laser therapy during colonoscopy. In some severe cases, hemicolectomy may be required, if bleeding continues.
 
Case No. 200
  1. Antibody against factor VIII, insufficient dose, poor quality factor VIII.
  2. Porcine factor VIII, or activated clotting factors- VIIa or Feiba (Factor eight inhibitor bypassing activity- an activate concentrate of II, IX and X).
Note: Following repeated transfusion of factor VIII, there may be production of antifactor VIII antibody, occurs in 20 to 30% cases of severe hemophiliac. This antibody neutralises the factor VIII, reducing its efficacy. This case is difficult to treat. Factor VIII from other species such as porcine factor VIII may be helpful. Activated clotting factors may be necessary. Other options are- immunosuppressive therapy (steroid, cytotoxic drugs) may be helpful.
 
Case No. 201
  1. Infective endocarditis with left sided septic cerebral embolism or left atrial myxoma with left sided cerebral embolism.
  2. Full blood count, blood for C/S, echocardiography and CT scan of brain.
    422
Note: Low grade continuous fever in any patient with cardiac murmur should suggest the possibility of infective endocarditis. However, with other systemic features, myxoma should also be excluded. Myxoma is a gelatinous, benign, polypoid, pedunculated, friable tumor arising from the heart. 75% from the left atrium, arises from fossa ovalis or its rim of atrial septum. May arise from mitral valve leaflet, posterior wall of left atrium, right atrium and rarely from ventricle. Involves any sex, any age, may be familial.
Clinical features: Three groups of features (i) Systemic features- fever, malaise, weight loss, arthralgia, Raynaud's phenomenon, skin rash, clubbing, (ii) Obstructive features- like mitral stenosis as it obstructs the valve. In addition to loud 1st heart sound and MDM, tumor plop (3rd sound). Patient may have breathlessness, PND, arrhythmia, syncope or sudden death (iii) Embolic- cerebral, splenic, kidney or any part of the body.
ESR- high, there may be anemia, may be hemolytic, polycythemia, leukocytosis, thrombocytopenia or thrombocytosis, abnormal serum protein, hypergammaglobulinemia may occur.
Confused with mitral stenosis, but there is no opening snap and the murmur is variable with postural change.
It is confirmed by transthoracic echocardiography, but a trans-esophageal echocardiogram is more accurate.
Treatment: Surgical excision as early as possible. In 5%, chance of recurrence.
 
Case No. 202
  1. Lumbar puncture and CSF study, CT scan or MRI of brain, EEG, antibody to HSV.
  2. Herpes simplex encephalitis.
  3. Brain abscess, viral encephalitis, meningitis.
Note: Herpes simplex type 1 may cause encephalitis, type 2 may cause benign recurrent lymphocytic meningitis. The virus usually affects the inferior aspect of frontal lobes and medial aspect of temporal lobe.
Herpes simplex encephalitis is characterized by flu-like illness, followed by fever, severe headache, altered consciousness, behavior abnormality and speech disturbance. There may be focal neurological deficit, such as dysphasia, hemiparesis, focal or generalized seizure, commonly temporal lobe seizure. Olfactory and gustatory hallucinations and impairment of memory are recognized. There may be multiple 423cranial nerve palsy and ataxia. Untreated patient develop convulsion and laps into comatose state. Mortality is high.
Diagnosis: Serum anti-HSV antibody. CSF study which shows lymphocytic leukocytosis, normal protein and sugar. In CSF, HSV-DNA polymerase chain reaction (PCR) is highly sensitive for rapid diagnosis. EEG shows distinctive periodic pattern in some cases. CT scan shows low density lesion in temporal lobes, that enhance with contrast. MRI shows orbitofrontal and medial temporal lobe involvement (not found in other virus).
Treatment: Aciclovir 10 mg/kg 8 hourly IV for 10 days. Anticonvulsant may be necessary. Dexamethasone for raised intracranial pressure.
 
Case No. 203
  1. SLE.
  2. Liebmann-Sach's endocarditis.
  3. ANA, anti-ds DNA, color Doppler echocardiography.
Note: In young female with unexplained fever, polyarthralgia, mouth ulcer, persistently high ESR, SLE should be excluded.
In heart, it can cause non-infective, atypical verrucous endocarditis, called Liebmann-Sach's endocarditis, which itself remains asymptomatic, but may cause valvular insufficiency. Commonly involves mitral valve, causing mitral regurgitation, which may be a source of systemic embolism. Anti-phospholipid antibody may be associated with this.
Treatment: High dose steroid is given.
 
Case No. 204
  1. Bronchial carcinoma with non-metastatic extrapulmonary mani-festations (or paraneoplastic syndrome).
  2. Sputum for malignant cells, CT guided FNAC.
Note: Low sodium, chloride, urea are due to SIADH and high calcium is due to PTH like secretion of substance from bronchial carcinoma. These may be due to non-metastatic extrapulmonary manifestations of bronchial carcinoma, occurs in 15 to 20% cases. These are (i) Endocrine—SIADH, ectopic ACTH secretion, carcinoid syndrome, hypercalcemia, gynecomastia, (ii) Neurological- peripheral neuropathy, subacute cerebellar degeneration, cortical degeneration, MND, (iii) Musculoskeletal—polymyositis or dermatomyositis, myasthenic myopathic syndrome (Eaton Lambert syndrome), clubbing and hypertrophic osteoarthropathy, (iv) Hematological—anemia, migrating 424thrombophlebitis, DIC, thrombotic thrombocytopenic purpura, (v) Marantic endocarditis (non-bacterial thrombotic endoarditis), (vi) Skin-acanthosis nigricans, dermatomyositis, (vii) Kidney-nephrotic syndrome (membranous glomerulonephritis).
 
Case No. 205
  1. JIA (Still's disease).
  2. SLE.
  3. RA test, ANA, FNAC of lymph nodes.
Note: Fever, arthritis, neck stiffness, lymphadenopathy, hepato-splenomegaly and leukocytosis in early age should suggest the possibility of JIA, Still's variety. However, other possibilities such as-lymphoma, SLE, disseminated tuberculosis, acute aleukemic leukemia, infection should be excluded.
Still's disease is characterized by JIA with systemic features, such as high swinging fever, evanescent pink maculopapular skin rash with fever called Salmon rash, arthralgia or arthritis, myalgia, pleurisy, pericarditis, hepatosplenomegaly, lymphadenopathy. Equally affects both the boys and girls upto 5 years, after that more in female. Leukocytosis, lymphocytosis, thrombocytosis, high ESR and CRP may be present. Autoantibody—negative.
Treatment: (i) To relieve pain- NSAID, (ii) Disease modifying drugs in all cases (such as methotrexate sulfasalazine is used, if enthesitis), (iii) Steroid may be required, if systemic features are present (pulse methylprednisolone), (iv) Anti-TNF agents may be used if methotraxate fails (e.g. etanercept, infliximab, adalimumab), (v) Others—Physiotherapy, passive movement to prevent contracture, orthopedic measures, if needed.
 
Case No. 206
  1. Visceral leishmaniasis.
  2. ICT for leishmaniasis, urine for latex agglutination test (Katex), bone marrow for LD body, FNAC of lymph node.
  3. Lymphoma, malaria.
Note: Prolonged fever, hepatosplenomegaly, increased monocyte and low albumin are all in favor of visceral leishmaniasis.
CBC, if repeated, will show progressive leukopenia. There may be pancytopenia, granulocytopenia and increased monocyte. For definitive diagnosis, isolation of LD body from splenic puncture (98%), bone 425marrow or aspiration from lymph node. Among the non-invasive tests, ICT and urine for latex agglutination test are highly sensitive and specific for early diagnosis. Other tests—DAT, CFT, hemagglutination, ELISA, PCR may be done.
IgG is markedly increased, IgM less so. Total protein is also very high, with low albumin. Cell mediated immunity to visceral leishmaniasis is undetectable.
Treatment: Sodium stibogluconate 20 mg/kg, IV for 30 days. Miltefosine is the only oral drug for leishmaniasis, 50 mg if <25 kg weight, 100 mg in > 25 kg, or 2.5 mg/kg in children.
 
Case No. 207
  1. Pyogenic liver abscess.
  2. USG of hepatobiliary system.
  3. Blood for C/S, fluorescent antibody test, indirect hemagglutination test, ELISA.
Note: High fever, hepatomegaly, high alkaline phosphatase, low albumin, leukocytosis indicate the diagnosis of pyogenic liver abscess. PUO may be the only presentation.
Common organisms in pyogenic liver abscess are E. coli, streptococcus (S. milleri). Others are S. faecalis, Staph. aureus, anaerobic organism, bacteroids.
Amebic liver abscess also occurs commonly. With history of <10 days, multiple liver abscesses occur in 50% cases. After ten days, 80% cases occur with single abscess. 5 to 10% cases will have negative serology. USG or CT scan is diagnostic to see the abscess. Aspiration of the pus may demonstrate the organism. Rupture of the abscess into the lung and development of hepatobronchial fistulae is a recognised complication (with anchovy-sauce sputum production). Peritonitis, pericarditis and cutaneous sinus formation are recognized rare complications.
Local intercostal tenderness in right lower chest, local edema should be looked for. Abnormality in right lung base occurs in 25% cases.
 
Case No. 208
  1. Sarcoidosis.
  2. Cardiomyopathy.
  3. MT, serum calcium, serum ACE, gallium67 scanning, lung biopsy.
    426
Note: Polyarthritis, skin rash, reticulonodular shadow in the lung, lymphopenia are all in favor of sarcoidosis. Dilated cardiomyopathy may also occur. 5% cases of sarcoidosis affect the heart with arrhythmia, conduction disturbance, pericarditis, dilated cardiomyopathy, ventricular aneurysm in presence of normal angiogram. Lymphopenia is characterastic, high serum ACE (may be done to monitor the activity rather than the diagnosis). Anergy is common with negative MT. Lung functions show restrictive pattern and impaired gas exchage.
X-ray of hands may show cystic change in the bone.
 
Case No. 209
  1. Proximal renal tubular acidosis (type 2).
  2. Ammonium chloride test, serum and urinary pH, USG of renal system.
Note: Proteinuria, glycosuria with low potassium and hyperchloremic acidosis indicate proximal renal tubule dysfunction. In this condition, the amount of bicarbonate that can be reabsorbed is lower than the normal, resulting in bicarbonate loss and acidosis. Proximal RTA may present with myopathy, osteomalacia, etc.
Causes of proximal RTA – (i) Fanconi syndrome (ii) Wilson's disease, (iii) Cystinosis, (iv) Degraded tetracycline, (v) Hyperparathyroidism, (vi) Multiple myeloma, (vii) Amyloidosis, (viii) Drugs - Carbonic anhydrase inhibitor, ifosfamide, (ix) Heavy metal poisoning- Pb, Hg, Cd.
 
Case No. 210
  1. Chronic constrictive pericarditis.
  2. ECG, Echocardiography, CT scan or MRI of heart.
  3. Cardiac catheterization.
Note: Chronic constrictive pericarditis is suggested by raised JVP, no cardiomegaly, enlarged, tender liver and ascites. Liver function tests may be slightly abnormal, because of passive venous congestion. Serum protein may be low, because of protein losing enteropathy.
Chronic constrictive pericarditis is due to progressive thickening, fibrosis and calcification of the pericardium.
Signs of chronic constrictive pericarditis are (i) Pulse- low volume, tachycardia, pulsus paradoxus may be present, (ii) JVP- raised, Kussmaul's sign may be positive, rapid Y descent, (iii) No 427cardiomegaly, (iv) Third sound (pericardial knock), (v) Hepatomegaly, (vi) Ascites- earlier than edema.
Differential diagnosis- restrictive cardiomyopathy and CCF.
Chest X-ray- pericardial calcification in 50% cases, no cardiomegaly.
Echocardiography shows thickening of pericardium. Cardiac MRI is more usefull than echocardiography.
Cardiac catheter- Pericardial constriction encompasses the whole heart, so right and left end diastolic pressures are equal, also the same atrial pressures (shows equal diastolic pressure in all the chambers of heart).
Treatment: Surgery plus treatment of causes.
 
Case No. 211
  1. Phenytoin induced SLE.
  2. ANA, anti-ds DNA, skin biopsy.
  3. Anti-histone antibody.
Note: Drug induced SLE is suggested by history of taking drug, fever, mild systemic features, polyarthralgia, skin rash and pericarditis. There is positive ANA, but negative anti-ds DNA, normal complements. Sex ratio- equal. Lung involvement is common, but renal and neurological involvement are rare. Anti-histone antibody is positive in 95% cases.
Treatment: (i) Withdrawal of the drug, (ii) Occasionally, short course steroid.
Drugs causing SLE are hydralazine, procainamide, anticonvulsant (phenytoin, carbamazepine), phenothiazine, INH, oral contraceptive pills, penicillamine, methyldopa, ACE inhibitor, monoclonal antibody, minocycline.
 
Case No. 212
  1. Pneumatosis cystoides intestinalis.
  2. Plain X-ray abdomen in erect posture, stool for R/M/E and C/S, sigmoidoscopy.
  3. Blind loop syndrome.
  4. High flow oxygen.
Note: In any patient with systemic sclerosis, if there is severe abdominal pain, always exclude pneumatosis cystoides intestinalis. It is a rare disease, characterized by presence of air filled pseudocysts in the bowel wall, either submucosal or subserosal, commonly colon. 428May cause severe colicky abdominal pain, diarrhea, tenesmus, bleeding and intestinal obstruction. Causes unknown, in some cases, may be associated with other diseases, such as COPD, necrotising enterocolitis, ischemic colitis, systemic sclerosis, etc. Rupture of cyst produces pneumoperitoneum. Characteristic presence of pneumoperitoneum without signs of peritoneal irritation is a clue that surgery is not required immediately. In idiopathic type, sigmoidoscopy may show blue sessile polyp with normal overlying mucosa. Plain X-ray will show gas pockets in bowel which is highly characteristic.
Treatment: (i) High concentration of oxygen, in some cases, hyperbaric oxygen (ii) Broad spectrum antibiotic IV with metronidazole.
 
Case No. 213
  1. Aspirin induced platelet dysfunction.
  2. Aspirin should be stopped.
Note: Prolonged use of some drugs such as aspirin, other NSAID, antiplatelet drugs may cause bleeding due to functional abnormality of platelet. Following any bleeding, there may be high leukocyte, platelet, etc. without infection (as in this case).
Other causes of bleeding should be excluded such as DIC, collagen diseases, hemophilia, Christmas disease, von Willebrand's disease, chronic liver disease from the history and physical findings.
Causes of prolonged bleeding time – (i) Thrombocytopenia due to any cause (ii) Abnormal platelet function (drug - Aspirin, NSAID, ticlopidine) (iii) von Willebrand's disease, Glanzmann's syndrome, dysproteinemia, etc. (iv) Factor V deficiency (v) Liver failure.
Causes of prolonged prothrombin time: It measures extrinsic pathway (I, II, V, VII, X) and it is prolonged with deficiency of these factors.
Other causes are (i) Anticoagulant therapy (Warfarin) (ii) Hepatic failure (iii) DIC (iv) Malabsorbtion of vitamin K (vi) Hemorrhagic disease of newborn. PT is most sensitive to deficiency of the factors V, VII and X.
Causes of prolonged APTT: Also called PTTK (partial thromboplastin time with kaolin). It measures intrinsic pathway (I, II, V, VIII, IX, X, XI) and it is prolonged with deficiency of these factors. Other causes are- (i) Hemophilia (ii) Christmas disease (iii) von Willebrand's disease (iv) Anticoagulant (heparin) (v) Hepatic failure (vi) DIC (vii) Lupus anticoagulant.
Causes of prolonged thrombin time: (i) Hypofibrinogenemia (ii) Liver failure (iii) Heparin therapy (but not warfarin), (iv) DIC.429
APTT is not prolonged by low molecular weight heparin.
TT is used to monitor heparin therapy.
 
Case No. 214
  1. Hemolysis caused by dapsone.
  2. Heinz body.
  3. Estimation of serum methemoglobin and sulfhemoglobin.
  4. Hemolysis, vitamin B12 deficiency, folic acid deficiency.
Note: Macrocytic anemia with high reticulocyte count indicate hemolysis, more likely caused by dapsone. Other causes of macrocytosis may be due to vit B12 or folic acid deficiency, as the patient also looks malnourished. Dapsone may cause mild hemolysis. It can also cause methemoglobinemia and sulfhemoglobinemia, responsible for chemical cyanosis.
Treatment of leprosy: (i) Paucibacillary (2 to 5 skin lesions)- rifampicin 600 mg monthly (supervised) plus dapsone 100 mg daily (self-administered) for six months, (ii) Multibacillary (>5 skin lesions)- rifampicin 600 mg and clofazimine 300 mg monthly (supervised) plus dapsone 100 mg and clofazimine 50 mg daily (self-administered) for twelve months, (iii) Paucibacillary single lesion- Ofloxacin 400 mg plus rifampicin 600 mg plus minocycline 100 mg- in single dose.
N.B. First line drugs in leprosy—rifampicin, clofazimine and dapsone.
Second line drugs—pefloxacin, ofloxacin, minocycline and clari-thromycin.
 
Case No. 215
  1. Occupational asthma.
  2. Pneumoconiosis.
  3. Serial measurement of PEFR, serum IgE antibody, skin prick test.
  4. Avoid exposure to the responsible agents, use of mask during working, step care treatment of bronchial asthma.
Note: Clue for the diagnosis of occupational asthma in this patient- (i) Textile mill worker, (ii) Relief of symptoms while at home, (iii) Normal X-ray. It is more in smoker.
Poor response of reversibility test indicates severe asthma, not necessarily COPD.
Occupational asthma may be defined as “asthma induced at work by exposure to occupation-related agents, which are mainly inhaled at 430the work place”. The most characteristic features in the medical history is symptoms that worsens on workdays and improves on rest days or holidays. It comprises about 5% of all adult onset bronchial asthma.
This type of asthma may be found in chemical workers, pharma-ceutical workers, farmers, grain handlers, cigarette manufacturers, fabric, dye, cosmetics workers, press and printing workers, laboratory workers, poultry breeders, textile workers, wood workers and bakery workers, etc. Atopic individuals and smokers are at risk of developing occupational asthma.
The diagnosis of occupational asthma is made by demonstating improvement of peak expiratory flow rate or lung volumes when the patient is away from allergen. Measurement of 2 hourly peak flow at and away from work is helpful for diagnosis. In some cases, IgE antibody is increased, specially in atopic individuals.
Keystone of effective management is cessation of further exposure, appropriate work place measure like masks, barriers. If no response, step care asthma management plan should be done.
 
Case No. 216
  1. Gitelman's syndrome.
  2. 24 hours urine potassium, serum renin and aldosterone (others— 24 hours urinary calcium).
  3. Bartter's syndrome, diuretic abuse.
Note: Gitelman's syndrome is characterized by hypokalemia, metabolic alkalosis, normal blood pressure, hypocalciuria, hypomagnesemia, high renin and high aldosterone.
Diagnosis: Low serum potassium (<3 mmol) and high 24 hours urine potassium (>20 mmol) plus high renin and high aldosterone, hypocalciuria, hypomagnesemia.
Treatment: Potassium plus potassium sparing diuretic (spironolactone).
Gitelman's syndrome- variant of Bartter's syndrome, but hypocalciuria, hypomagnesemia are present in Gitelman's syndrome, not in Bartter's syndrome.
 
Case No. 217
  1. History of induced vomiting that relieves the symptom.
  2. Hypochloremic hypokalemic metabolic alkalosis.
  3. Gastric outlet obstruction due to chronic duodenal ulcer.
  4. Endoscopy.
431
Note: Long history of indigestion or dyspepsia may be due to chronic duodenal ulcer. In young patient with vomiting is highly suggestive of gastric outlet obstruction. In such a case, vomiting of acid stomach contents leads to loss of H+ and Cl. HCO3 generated in extracellular fluid leads to hypochloremic alkalosis. Hypokalemia is due to vomiting, secondary aldosteronism, alkalosis (enhance entry of potassium into cell), increased renal loss in exchange of sodium. Serum pH is high and urine pH is low (acidic).
 
Case No. 218
  1. Acute pyelonephritis.
  2. Perinephric abscess.
  3. Loss of concavity in right flank or bulging in right flank with scoliosis.
  4. Ultrasonography of abdomen, specially renal system.
  5. Involvement of L3 and L4, deep venous thrombosis of right calf.
Note: Classic triad of pyelonephritis are- fever, loin pain and tenderness over kidney. Fever may be very high, associated with chill and rigor. Urinary complaints like dysuria, frequency, urgency are present.
Perinephric abscess is characterized by severe pain and tenderness in loin, may be bulging and very high fever. Urinary symptoms may be absent and urine examination may be entirely normal.
Other causes of dull or severe loin pain- Renal stone, renal tumor, acute pyelonephritis, obstruction in renal pelvis, renal tuberculosis, congenital anomaly of pelvi ureteric junction (pain increased after drinking more fluid).
 
Case No. 219
  1. Pemphigus vulgaris.
  2. Erythema multiforme, bullous pemphigoid, bullous lichen planus.
  3. Skin biopsy for histopathology and immunofluorescence test (an early intact bullae < 12 hours duration with perilesional area) should be taken.
Note: Pemphigus are a group of autoimmune blistering diseases of skin and mucous membranes, which include pemphigus vulgaris (PV), pemphigus foliaceus and paraneoplastic pemphigus.
PV is an autoimmune blistering disease affecting the skin and mucous membranes, mediated by circulating autoantibodies, directed against keratinocyte cell surfaces. It is characterized by thin walled, 432flaccid, easily ruptured, intra-epidermal bullae that appear in apparently normal skin and mucous membrane or on erythematous base (associated with mouth ulcer). Causes- unknown. Probable factors- autoimmunity, suggested by intercellular deposition of IgG and C3 complement in epidermis, genetic predisposition (associated with HLA DR4 and DR6), drugs (penicillin, penicillamine, captopril, rifampicin), ultraviolet light, PUVA, ionizing radiation, increased incidence in myasthenia gravis and thymoma. There is recognized association with human leukocyte antigen DR4 and DRw6.
PV presents with oral lesions in the majority of patients and most patients develop cutaneous lesions.
Nikolsky sign- rubbing of uninvolved skin results in separation of epidermis.
Treatment: (1) High dose prednisolone– 100 to 200 mg daily. When remission occurs with no new blister, taper the dose. Maintenance dose for long time (may require life long). If new blister during treatment, increase the dose of prednisolone. (2) Other therapy- Intravenous methylprednisolone– 1 gm/day for 5 days (pulse therapy). Mycophenolate mofetil 1 to 1.5 gm twice a day (commonly used as steroid sparing drug). Other drugs- azathioprine, cyclophosphamide, methotrexate, ciclosporin, dapsone. In resistant case- intravenous immunoglobulin may be tried. Biologic agents (infliximab, rituximab, etanercept). Extracorporeal photochemotherapy. (3) General treatment- Topical- daily bath, 1% silver sulphadiazine, antiseptic mouth wash, care of eye. Antibiotic, fluid and electrolyte balance, blood transfusion (if necessary). Prognosis- bad, recurrence is common with high mortality.
 
Case No. 220
  1. Renal tuberculosis, partially treated bacterial UTI.
  2. MT, urine for AFB and mycobacterial culture and sensitivity, ANA and anti-ds DNA.
Note: Renal or genitourinary tuberculosis is quite common. This lady has a sterile pyuria, which could be due to partially treated bacterial infection. However, genitourinary tuberculosis should be excluded. Genitourinary TB develops in approximately 5% cases of pulmonary TB and is usually due to hematogenous spread to the renal cortex during the primary phase of infection. The cortical lesion may ulcerate, ultimately involving the pelvis, bladder, also in male seminal vesicles 433and prostate. Other clinical features include hematuria, urethral strictures, cold abscess and chronic epididymo-orchitis. Renal failure may occur due to extensive destruction of the kidneys or by obstruction secondary to fibrosis. The diagnosis can be made with ultrasound or CT scan, intravenous urography, in combination with several early morning urine samples for mycobacterial culture.
Treatment: Like pulmonary TB, to be continued for 9 months to 1 year.
 
Case No. 221
  1. Central pontine myelinolysis (CPM).
  2. MRI of brain.
Note: Rapid correction of hyponatremia by hypertonic saline results in central pontine myelinolysis (CPM), which is a dangerous complication. There may be various types of neurological deficits, from quadriparesis to coma and death. In CPM, there is demyelination of central basal pons or other areas of brain. Magnetic resonance imaging is the investigation of choice to demonstrate classical demyelination. Usually, rapid correction of hyponatremia by hypertonic saline should be avoided. Correction with IV normal salaine and oral salt may be sufficient.
 
Case No. 222
  1. Analgesic nephropathy.
  2. CT scan of renal system, IVU, renal biopsy.
  3. Withdrawal of offending drug, maintaining a fluid intake of 2 to 3 liter per day, control of hypertension and biochemical correction.
Note: The patient was suffering from long standing arthritis for which she used to take multiple analgesic, which are responsible for chronic tubulo-interstitial nephritis with papillary necrosis. It is twice as common in women, and is an important cause of chronic renal failure. 60% of patients are hypertensive at presentation. Sloughing of renal papillae can cause urinary tract obstruction, which may precipitate acute renal failure. Recurrent urinary tract infections are common. There may be sterile pyuria, also occasionally a salt-losing nephropathy. Diagnosis can be made by the history and characteristic appearances on intravenous urography, urine microscopy and biochemical evidence of tubular dysfunction. Renal biopsy is sometimes performed, which shows interstitial fibrosis and tubular atrophy. There is risk of tumors of uroepithelium. Total recovery of renal function occur in 25% patients.434
 
Case No. 223
  1. Churg Strauss syndrome, Wegener's granulomatosis, microscopic polyangitis (also polyarteritis nodosa).
  2. Churg-Strauss syndrome.
  3. P-ANCA, C-ANCA, kidney biopsy, ANA and anti-ds DNA.
Note: It is likely to be a case of Churg-Strauss syndrome, as suggested by asthma, hypertension, glomerulonephritis, eosinophilia. It is associated with positive p-ANCA.
Churg-Strauss syndrome is a small-vessel granulomatous vas-culitis characterized by cutaneous vasculitic lesions, respiratory involvement giving asthmatic symptoms, eosinophilia, mononeuritis or polyneuropathy and rarely glomerulonephritis. Gastrointestinal and cardiac involvement are recognized. Nasal polyp, allergic rhinitis and adult onset asthma usually occur before vasculitis by many years.
Diagnosis is usually clinical and supported by the presence of necrotizing granulomatous vasculitis with extravascular eosinophilic infiltration in lung, renal or sural (calf) muscle biopsy.
American College of Rheumatology has defined criteria for the diagnosis of Churg Strauss syndrome. Presence of 4 or more is highly indicative of Churg Strauss syndrome:
  • Asthma.
  • Eosinophilia (10% on WBC differential).
  • Mononeuropathy or polyneuropathy.
  • Migratory or transient pulmonary infiltrates.
  • Systemic vasculitis (cardiac, renal, hepatic).
  • Extravascular eosinophils on a biopsy including artery, arteriole or venule.
Treatment: Glucocorticoid is the drug of choice. Major life threatening organ involvement may require treatment with pulse doses of IV methylprednisolone as well as other cytotoxic agents. Other treatments include intravenous immune globulin, interferon-alpha, and plasma exchange. Plasma exchange has not improved the course of the disease.
 
Case No. 224
  1. Heparin induced thrombocytopenia (HIT).
  2. Heparin should be stopped, and alternative drug like lepirudin or a heparinoid such as danaparoid should be started.
    435
Note: Thrombocytopenia may occur as a complication of heparin therapy. 2 types of heparin-induced thrombocytopenia. Type 1 usually occurs within 48 to 72 hours after therapy with unfractionated heparin, and platelet count rarely falls below 1,00,000. Platelet count usually normal over the next 4 days and there is no increased risk of thromboembolism. Type 2 is much rare and usually occurs 5 to 10 days after treatment with heparin and platelet count usually drops below 1,00,000. It is thought to be immune mediated, resulting in the formation of antibodies against heparin-platelet factor 4 complex. There may be thromboembolism, venous more than arterial. Also, there may be skin necrosis. This condition is suspected in patient who develops thrombocytopenia or >50% drop from the original platelet count with thrombosis, few days after starting heparin. Type 2 HIT can be prevented by using low molecular weight heparin or limiting the use of heparin not >5 days.
All heparin products should be stopped and the patient should be commenced on alternative drug such as lepirudin or a heparinoid such as danaparoid.
 
Case No. 225
  1. Pseudo-Cushing's syndrome.
  2. Insulin induced hypoglycemia.
Note: Cortisol excess due to other illness is called pseudo-Cushing's syndrome. There is increased urinary excretion of steroid, absent diurnal variation of cortisol and failure of suppression by dexamethasone. This disorder may occur in chronic alcoholic, severe depression and in simple obesity. All revert to normal after removal of the cause.
To differentiate from Cushing's syndrome- insulin induced hypoglycemia is helpful. In Cushing's syndrome- almost no response, but in pseudo-Cushing's syndrome, there is excess cortisol secretion.
Overnight dexamethasone suppression test is usually used as a screening test for Cushing's syndrome. This test has a false-positive rate of up to 2 to 12%, Dexamethasone is primarily metabolized by the cytochrome P-450 system. Considerable increase in cytochrome P-450 enzyme can be seen in regular smokers and people who drink alcohol regularly. Several drugs such as phenobarbital, primidone, ethosuximide, carbamazepine and rifampicin induce the activity of the enzyme and can lead to false positive dexamethasone suppression test.436
 
Case No. 226
  1. Diabetic nephropathy with retinopathy.
  2. Losartan therapy.
Note: The patient is suffering from type 2 diabetes mellitus with moderate proteinuria and renal failure, highly suggestive of diabetic nephropathy. For the control of hypertension, ARB is thought to be more effective, specially in nephropathy. ACE inhibitor is helpful to reduce micro-albuminuria in patient without overt nephropathy and retard progression of nephropathy in Type 1 diabetes mellitus.
Control of hypertension and good glycemic control are effective in retarding nephropathy in both Type 1 and Type 2 diabetes mellitus.
 
Case No. 227
  1. Non-alcoholic fatty liver disease.
  2. Supervised exercise program to reduce body weight, metformin, statin.
  3. Liver biopsy.
Note: Non-alcoholic fatty liver disease (NAFLD) should be considered in obese individuals with abnormal liver function tests without evidence of alcohol excess. It is often asymptomatic, but hepatic enlargement due to lipid deposition within hepatocytes is common.
NAFLD may be associated with insulin resistance and dyslipidemia. Patient with NAFLD are therefore at high risk of developing cardiovascular disease and diabetes. Good control of diabetes and hyperlipidemia will reduce mortality and morbidity. Statins are not contraindicated in NAFLD despite the widespread misconception that these cause significant derangement of LFTs.
Rapid weight loss may increase fat deposition in liver and may precipitate NASH. Weight loss of 1 kg/week with an initial target loss of 10% body weight has been shown to be effective at restoring normal liver function. Exercise programs are an effective way to aid weight loss and reduce insulin resistance.
Various drugs have been used in the treatment of NAFLD, without proved benefit. Metformin is recommended for the treatment of insulin resistance associated with NAFLD and has been shown to reduce transaminases and lead to histological improvement. Insulin has little proven benefit over oral agents in the treatment of Type 2 diabetes.437
 
Case No. 228
  1. Disseminated tuberculosis, lymphoma, neurosarcoidosis, SLE.
  2. Neurosarcoidosis.
  3. MT, ANA and anti-ds DNA, FNAC or biopsy from lymph node. (Others- serum calcium, lung biopsy).
Note: This patient has neurological findings with dehydration, pyrexia, bilateral iritis and also lymphopenia, X-ray shows reticulonodular shadow. These findings are all consistent with neurosarcoidosis. Cerebral involvement in sarcoidosis is rare, features are cranial nerve lesions, aseptic meningitis, psychosis, multiple sclerosis type syndrome, etc. Mononeuritis multiplex is a recognized manifestation in peripheral nervous system.
(See also Case Nos 007 and 208).
 
Case No. 229
  1. Myasthenic crisis.
  2. Plasmapheresis.
  3. Arterial blood gas analysis, serial measurement of FVC.
Note: The patient's history and physical findings support the diagnosis of myasthenia gravis. Severe generalized muscle weakness and respiratory failure, deterioration suddenly is due to myasthenic crisis. This may be precipitated by intercurrent illness or may occur spontaneously or with drugs.
Plasmapheresis directly removes anticholinergic receptor antibodies from the circulation. Treatment is given for up to 2 weeks and is effective for 1 to 2 months. Intravenous immunoglobulin is as effective as plasmapheresis in acute myasthenic crisis. Anti-cholinesterase drug therapy may cause cholinergic crisis that is also characterized by muscle weakness. Differentiation between cholinergic crisis and myasthenic crisis is based upon the presence of muscle fasciculation, pallor, abdominal cramps, excessive sweating, excessive salivation, small pupil and bradycardia in the former. Anticholinesterase drugs are withdrawn in myasthenia gravis, who present with severe generalized muscle weakness.
 
Case No. 230
  1. Wegener's granulomatosis (WG).
  2. Microscopic polyangitis (or polyarteritis nodosa).
  3. c-ANCA, biopsy from nasal crust.
    438
Note: Wegener's granulomatosis is a necrotising granulomatous vasculitis, involving the upper and lower respiratory tract and kidneys. It causes epistaxis, sinusitis, destruction of the nasal cartilage and glomerunephritis and renal failure. c-ANCA is positive. It may involve any organ system.
WG, microscopic polyangitis (MPA) and Churg-Strauss syndrome (CSS) are associated with primary systemic small-to-medium-sized vessel vasculitis associated with antineutrophil cytoplasmic antibodies (ANCA). However, WG can also affect medium and even large arteries, and may lack an association with ANCA. Usually, Wegener's granulomatosis is characterized by the presence of c-ANCA (antigen is proteinase 3), whereas microscopic polyangitis is characterized by p-ANCA (antigen is myeloperoxidase).
Diagnosis of WG is based on combination of clinical, laboratory and, if necessary, pathological features. If typical clinical picture is associated with positive ANCA, with specificity for proteinase 3 (PR3), the diagnosis of WG can be presumed. Biopsy of the involved tissue will show necrotising granulomatous lesion in small vessels (biopsy taken from nasal crust, lung or kidney, if these are involved).
Treatment with glucocorticoid and cyclophosphamide.
 
Case No. 231
  1. Postural hypotension, hypoglycemic attack, pacemaker failure, pacemaker syndrome.
  2. Pacemaker syndrome.
  3. Dual chamber pacing should be done.
Note: Following single chamber pacing, there may be pacemaker syndrome. It is a disorder characterized by transient hypotension, fatigue, dizziness, syncope and distressing pulsation in the neck and chest. This occurs at the onset of ventricular contraction due to loss of atrio-ventricular synchrony. It occurs in single chamber pacing, which can be prevented by dual chamber pacing or by reducing the pacemaker rate so that, sinus rhythm predominates. However, as the patient is diabetic and hypertensive, other causes as mentioned above should be excluded.
 
Case No. 232
  1. Occupational asthma, extrinsic allergic alveolitis.
  2. Extrinsic allergic alveolitis (EAA).
    439
  3. Lung function test, CT scan of the chest, bronchoscopy with bronchoalveolar lavage.
Note: This is likely to be a case of extrinsic allergic alveolitis, evidenced by her work in the farm, with the symptoms consistent with hypersensitivity pneumonitis. Recently, she has developed progressive breathlessness, suggesting parenchymal lung disease. Repeated episodes of pneumonitis progress to pulmonary fibrosis. In chronic cases, CXR is similar to cryptogenic fibrosing alveolitis except that in EAA, fibrosis is usually more pronounced in the upper zones. Also in such cases, usually there are clubbing with bilateral end inspiratory coarse crepitations.
Allergic bronchopulmonary aspergillosis is another possibility, which is characterized by wheeze, high eosinophil count, and proximal bronchiectasis in chest X-ray.
Diagnostic features of extrinsic allergic alveolitis- (1) Evidence of exposure to a recognized antigen, (2) clinical and radiographic features (cough, wheeze, fever, micronodular shadows in upper, mid or lower zones, restrictive lung defect), (3) Bronchoalveolar lavage with lymphocytosis (with low CD4 to CD8 ratio), (4) Positive inhalation challenge test, (5) Compatible histopathological changes. The diagnosis is possible without histological confirmation, if criteria 1 to 3 are present. Farm workers have a higher risk of developing extrinsic allergic alveolitis.
 
Case No. 233
  1. Kallmann's syndrome.
  2. Anosmia.
  3. Measurement of gonadotropin releasing hormone (GnRH).
Note: Kallmann's syndrome results from disordered migration of gonadotropin releasing hormone (GnRH) producing neurons into the hypothalamus. This leads to failure of episodic GnRH secretion and subsequent failure of luteinising hormone (LH), follicle stimulating hormone (FSH) and testosterone production. It is often familial, may be inherited as X-linked, autosomal dominant or autosomal recessive. 75% of sufferers have absent sense of smell (anosmia, which is due to hypoplasia of olfactory bulb). It is common in male, rare in female (male to female ratio 4:1). The syndrome is associated with cleft lip and palate, high arched palate, nystagmus and sensorineural deafness. In 55% cases, renal agenesis may occur. Also, there may 440be cryptorchidism, cerebellar dysfunction, cerebral abnormality (e.g. color blindness). Treatment- GnRH hormone therapy restore pituitary function. In female, cyclic estrogen and progesterone therapy should be given. Fertility is possible.
 
Case No. 234
  1. Osteomalacia.
  2. X-ray of pelvis (to see loozer's zone), serum 25-hydroxyproline (low or absent).
Note: Combination of bone fracture, hypocalcemia, hypophosphatemia, high alkaline phosphatase and high parathyroid hormone are highly suggestive of osteomalacia, due to vitamin D deficiency.
In vitamin D deficiency, there is increased parathyroid hormone, which maintains normal serum calcium concentration at the expense of the skeleton. Osteomalacia is a metabolic bone disease characterized by softening of the bone due to deficiency of vitamin D, resulting in inadequate mineralizatin of osteoid tissue. Ratio of osteoid tissue to calcium and phosphate is increased (low calcium, low phosphate and increased osteoid tissue. Alkaline phosphatase is also high).
 
Case No. 235
  1. Pneumonia due to mycoplasma pneumoniae.
  2. Erythema multiforme.
  3. Coomb's positive autoimmune hemolytic anemia.
  4. Serum cold agglutinin estimation, serological test for mycoplasma.
Note: In mycoplasma pneumonia, systemic upset, dry cough, fever, myalgia and arthralgia are common. WBC is usually normal. Extrapulmonary manifestations of mycoplasma occur in up to 10% of cases, such as cold autoimmune hemolytic anemia, thrombocytopenia, renal failure, hepatitis, myocarditis, meningism and meningitis, transverse myelitis and cerebellar ataxia. Cutaneous manifestation include erythema multiforme. Hemolysis is associated with the presence of cold agglutinins, found in up to 50% of cases of mycoplasma pneumoniae. Low sodium is due to SIADH complicating pneumonia. Diagnosis is based on demonstration of anti-mycoplasma antibodies in paired sera.
Treatment: Macrolide antibiotic (clarithromycin or erythromycin). Other drugs—tetracycline or doxycycline.441
 
Case No. 236
  1. Eaton Lambert syndrome.
  2. SIADH.
  3. EMG, measurement of anti-VGCC (voltage-gated calcium channel) antibodies.
Note: The clinical picture is myasthenic myopathic syndrome (also called Eaton Lambert syndrome). This is an autoimmune paraneoplastic phenomenon, most often associated with small cell lung cancer. It can present up to 2 years before a primary malignancy is diagnosed. Clinical features are weakness and pain, especially in limb girdle muscle groups. Proximal myopathy of lower limb is more common. Ptosis and diplopia are present in 70% cases. Ocular-bulbar features are usually less severe than in myasthenia gravis. In contrast to myasthenia gravis, reflexes are diminished or absent, autonomic disturbance like dry mouth or impotence are common in myasthenic myopathic syndrome. EMG shows progressive incremental response (opposite to myasthenia gravis, where there is progressive decremental response). The underlying pathophysiology involves production of autoantibodies against P/Q type voltage-gated calcium channels (VGCC) at the neuromuscular junction, which impair release of acetylcholine from nerve terminals.
Treatment: 3, 4-diaminopyridine and pyridostigmine are effective.
 
Case No. 237
  1. Subarachnoid hemorrhage.
  2. CT scan of brain.
  3. Lumbar puncture and CSF study.
  4. Polycystic kidney disease.
Note: Subarachnoid hemorrhage is characterized by severe headache of sudden onset, as if being hit on the back of head, called hammer headache, followed by unconsciousness. It is due to rupture of berry aneurysm. Polycystic kidney disease is associated with berry aneurysm in 10% cases. CT scan should be done immediately. Fundoscopy may show subhyaloid hemorrhage.
Treatment: Control of blood pressure and administration of nimodipine. Surgical obliteration of aneurysm is the mainstay of treatment with an increasing tendency for early clipping of the aneurysm (within three days of initial bleeding).442
 
Case No. 238
  1. Cerebellar syndrome due to phenytoin toxicity.
  2. Serum phenytoin and sodium valproate.
Note: The patient has signs of cerebellar lesion, which is more likely due to phenytoin toxicity. Nystagmus is present even in mild toxicity. Abnormal liver function is more likely due to sodium valproate, but may occur due to phenytoin. Hypotonia and reduced muscle power may be due to cerebellar syndrome. But in this case, it may be due to electrolyte imbalance. Though, CT scan is normal, MRI is the investigation of choice to detect lesion of cerebellum.
 
Case No. 239
  1. Listeria meningo encephalitis.
  2. MRI of brain.
Note: This patient has meningitis with brain stem involvement and CSF study shows high neutrophil, but no organism in gram staining. These features are consistent with listeria meningitis. It should always be considered in patient with meningitis associated with brain stem involvement. It is common in neonate, elderly, alcoholic, pregnant woman and in immunosuppressed patients. Focal neurological signs and cranial nerve palsy are recognized features. Viral, TB and fungal meningitis usually show a lymphocytic pleocytosis. Brain stem involvement with other bacterial meningitis is rare. Herpes simplex encephalitis involves the temporal lobes and therefore, presents with seizures, cognitive (memory) and behavior change. Brain stem involvement is absent.
Treatment: Gentamicin (5 mg/kg IV daily) and ampicillin (2 gm IV daily) intravenously.
 
Case No. 240
  1. Allergic bronchopulmonary aspergillosis.
  2. X-ray chest, serum IgE measurement, skin test to aspergillus fumigatus (other- sputum for fungal hyphae of A. fumigatus).
Note: History of bronchial asthma, eosinophilia and perihilar infiltrates is consistent with allergic broncho-pulmonary aspergillosis.
Diagnosis of allergic bronchopulmonary aspergillosis is made by (i) Bronchial asthma, (ii) Proximal bronchiectasis and parenchymal 443infiltrates in the perihilar area. (iii) High titers of IgE and IgG antibodies, (iv) Positive skin test to Aspergillus fumigatus, (v) Peripheral high eosinophil count, (vi) Fungal hyphae of A. fumigatus on sputum examination. X-ray chest shows pulmonary infiltrate, bronchiectasis (usually proximal).
Treatment: Prednisolone 40 to 60 mg daily and chest physiotherapy. Maintenance dose of steroid is 7.5 to 10 mg daily. Itraconazole may be helpful in some cases.
Differential diagnoses are- drug induced, tropical pulmonary eosinophilia or Churg-Strauss syndrome. In this case, no drug history to indicate an eosinophilic pneumonitis, nor a history of travelling to the tropics to suggest tropical pulmonary eosinophilia. Churg-Strauss syndrome is unlikely in the absence of vasculitis, neuropathy or renal involvement. Pure bronchial asthma does not cause pulmonary infiltrates.
Tropical pulmonary cosinophila is an immune reaction to the infection with human filarial parasites Wuchereria bancrofti and Brugia malayi. It is characterized by non-productive cough, wheeze, fever, weight loss, lymphadenopathy, eosinophilia and patchy infiltrates in chest X-ray. The condition occurs in patient infected in the tropics. It is treated with diethylcarbamazine.
 
Case No. 241
  1. Mixed obstructive and restrictive defect with a reduced Kco.
  2. Diabetes insipidus.
  3. Histiocytosis X (also called Langerhans cell histiocytosis).
  4. Serum and urine osmolality, bone marrow study.
Note: In this patient, low FEV1/FVC ratio is suggestive of obstructive airway defect and low residual volume suggests restrictive defect. Polydipsia, polyuria with respiratory involvement is suggestive of diabetes insipidus. Blood pictures show pancytopenia. In a boy with a mixed restrictive and obstructive defect, diabetes insipidus and pancytopenia are more likely due to histiocytosis X.
Transbronchial lung biopsy, liver biopsy or trephine aspirate may show histiocytes and small round cells.
Histiocytosis X is a syndrome of unknown etiology, characterized by formation of Langerhans cells from dendritic cells, that involves the skin, bones, ears, lungs, eyes and reticuloendothelial and central nervous systems. Skin rash is common. Pathology- there is proliferation 444of Langerhans cells with collection of histiocyte, eosinophil and lymphocyte. Langerhans cells secrete prostaglandin E and IL-1, responsible for widespread osteolytic bony lesion.
Three types – (1) Letterer Siwe disease - occurs in infant, fulminant visceral form of disease and confused with lymphoma. Involves liver, spleen, lymph node, skin, bone marrow. (2) Hand Schuller Christian disease -posterior pituitary is commonly involved. diabetes insipidus is common, multiple bone lesions, exophthalmos may occur. (3) Eosinophilic granuloma (adult type) - common in male and in smoker. Interstitial lung disease may occur. It may be focal, involve the bone (skull, jaw, spine).
Treatment:
  1. Unifocal- no treatment needed, sometimes surgery or curettage may be needed.
  2. Progressive disease- low dose radiation, chemotherapy (etoposide).
  3. Systemic involvement-
    • Stop smoking.
    • Drugs- steroid, vinblastin, methotrexate, etoposide.
    • If interstitial lung disease- penicillamine may be given.
    • Bone marrow transplantation in some cases.
 
Case No. 242 (see also Case No. 161)
  1. Leukemoid reaction.
  2. PBF may show toxic granulations or Dohle bodies in the white cells, neutrophils show toxic shift to the left.
  3. Bone marrow study.
Note: A leukemoid reaction may follow any sepsis or infection. In some cases of overwhelming infection, the ‘sick’ marrow releases immature cells into the blood stream. Thus, there is plenty of myelocytes and normoblasts, confuses with leukemia, where the marrow is characteristically replaced by neoplastic cells. In leukemoid reaction, there is no lymphadenopathy or hepatosplenomegaly.
 
Case No. 243 (see also Case No. 157)
  1. Lupus anticoagulant syndrome.
  2. ANA, anti-ds DNA, serum antiphospholipid or anticardiolipin.
  3. Anti-phospholipid antibody.
    445
Note: In any young patient specially female, it is important to exclude SLE or anti-phospholipid syndrome as a cause of CVD with hemiplegia. There is history of miscarriages, thrombocytopenia, high ESR, all of which are characteristic of lupus anticoagulant syndrome or anti-phospholipid syndrome. Echocardiography may show evidence of verrucous (Libmann-Sack) endocarditis. ASD is non-significant in this case.
 
Case No. 244
  1. Pneumonia due to legionella pneumophila.
  2. Urinary legionella antigen, direct immunofluorescent staining of sputum, anti-legionella antibody.
  3. Vomiting, SIADH, acute tubulo-interstitial nephritis.
  4. Oxygen therapy, antibiotic (erythromycin or azithromycin intravenously) plus rifampicin, correction of electrolytes.
Note: Pneumonia associated with confusion, hyponatremia, lympho-penia, hematuria and proteinuria are highly suggestive of legionella pneumophila infection. Abnormal liver biochemistry (especially aminotransferases) and renal function occur in 50% of patients.
Outbreaks due to contaminated water-cooling systems, air-conditioning or showers. May be occupational history or recent hotel holiday. Usually, occurs in previously fit individuals. Males are affected more than female.
Incubation period is 2 to 10 days and it typically presents with flu-like symptoms such as fever, malaise, myalgia and headache. The patient later develops cough, breathlessness and confusion, 50% develop gastrointestinal symptoms. Neurological features like confusion, hallucination, peripheral neuropathy, myelitis, rarely cerebellar syndrome may occur. Renal involvement like tubulointerstitial nephritis may occur.
Investigations: Hyponatremia, lymphopenia, hematuria, proteinuria and hypoxia. High SGPT, SGOT, alkaline phosphatase, CPK may occur.
Diagnosis: Quickest way is to detect urinary legionella antigen (90% positive in first week) and direct immunofluorescent staining of the organism in sputum (may be found in pleural fluid, or bronchial washings). Gram stain does not reveal organism. Fourfold rise in antibody titre to > 1:128 is helpful.
Treatment: A macrolide (azithromycin, clarithromycin, etc.) is given.446
 
Case No. 245
  1. Brucellosis.
  2. Brucella agglutination test or Brucella immunoglobulins (ELISA).
  3. Tetracycline and rifampicin.
Note: Malaise, myalgia, headache and weight loss in association with neutropenia, hepatosplenomegaly, orchitis and erythema nodosum are highly suggestive of brucellosis.
Brucellosis is caused by Brucella, a gram-negative bacillus, contacted from cows, goats, pigs or sheep. 3 species cause infection in man. B. abortus (cattle), B. melitensis (goats/sheep) and B. suis (pigs). Infection occurs usually through gastrointestinal tract, due to consuming unpasteurized milk. From gastrointestinal tract, bacilli travel to the Iymphatics and infect lymph nodes, and eventually there is hematogenous spread to other organs.
Onset is insidious. Malaise, headache, myalgia, weakness and night sweats are common. There is an undulant high fever. Lymphadenopathy is common. Hepatosplenomegaly may be present, splenomegaly usually indicates severe infection. Tenderness is relatively common. There may be arthritis, orchitis, endocarditis, osteomyelitis, and meningoencephalitis.
Brucellosis may be chronic, associated with fatigue, myalgia, depression, occasionally fever. Splenomegaly is characteristic. Infection may be localized to specific organs such as the bones, heart or central nervous system. In such cases, systemic features are absent in 60% cases and antibody titres are low.
Diagnosis is based on positive blood culture and rising anti-brucella immunoglobulin titres. Blood cultures are positive in 50% cases. Brucella agglutination test is positive within four weeks of the onset of illness.
Treatment: Tetracycline and rifampicin for six weeks. Co-trimoxazole is also effective.
 
Case No. 246
  1. Cervical myelopathy, cervical cord compression, motor neuron disease, multiple sclerosis.
  2. Cervical myelopathy.
  3. MRI of cervical spine.
  4. C5 and C6 level.
    447
Note: Progressive weakness of arms and hands with combination of lower motor neuron signs in upper limbs and upper motor neuron signs in lower limbs, more likely diagnosis is cervical myelopathy. Loss of vibration at ankles may be a normal finding in elderly, but may be due to dorsal column lesion.
Both cervical myelopathy and cervical cord tumor present with involvement of upper limb, which usually produces lower motor neuron disturbances at the level of the lesion and upper motor signs below it. MRI is the best investigation to differentiate between these two.
Absent supinator and biceps reflex, but exaggerated triceps reflex, indicates lesion at C5 and C6. Small muscle wasting in cervical myelopathy is due to reduced blood flow to the lower segments of the cord. Sensory disturbance in the upper limbs may be absent or very mild in cervical myelopathy.
Cervical myelopathy is due to bulging or extrusion of the disk into the cervical spinal canal, which results in pressure atrophy and ischemia. Common cause is osteoarthritis of cervical spine. Posterior columns (dorsal tracts carrying proprioception, vibration sense and light-touch fibers) and the lateral column (pyramidal tracts carrying upper motor neurones) are affected. C5 to C7 cervical segments are most commonly affected. Symptoms are those of lower motor neuron signs at these levels and upper motor neuron signs below. Neck stiffness and upper limb pain may be present. Spasticity of the lower limbs is common. Ataxia may occur owing to dorsal column involvement (sensory ataxia). Lhermitte's sign (electrical shock feeling down the spinal cord and into the legs) is recognized. This sign may also be found in cervical cord tumor, multiple sclerosis and subacute combined degeneration of spinal cord.
 
Case No. 247
  1. Postpartum thryoiditis.
  2. β-blocker.
Note: Postpartum thyroiditis occurs usually 2 to 6 months after delivery and is associated with transient hyperthyroidism, followed by hypothyroidism, which usually resolves but permanent hypothyroidism may occur. It occurs in 5% of females. Radioactive iodine test will show less uptake. Cause is unknown, but lymphocytic infiltration in thyroid is suggestive of autoimmunity. Symptoms usually resolve spontaneously, β-blocker may be given.448
It may be confused with de Quervain's thyroiditis, which is associated with tender enlargement of thyroid gland, constitutional symptoms and high ESR.
Causes of high T3 and T4, but low radioiodine uptake are found in subacute thyroiditis, postpartum thyroiditis, iodine induced hyperthyroidism, factitious hyperthyroidism.
 
Case No. 248
  1. Chronic lymphocytic leukemia.
  2. Hemolytic anemia.
  3. Autoimmune hemolytic anemia (warm type) and bone marrow infiltration.
  4. May be due to high reticulocyte count secondary to hemolysis, megaloplastic anemia due to folate deficiency.
  5. Direct Coomb's test.
Note: In elderly person with the above blood count, is highly suggestive of chronic lymphocytic leukemia.
This disease is common in elderly, 65 to 70 years. Male: Female = 2:1, involving B lymphocyte. It may be asymptomatic, diagnosed incidentaly in routine examination. General features like malaise, weakness, fatigue, weight loss, night sweating, features of anemia. There may be recurrent infection, generalized lymphadenopathy, hepatosplenomegaly. Blood picture shows high lymphocyte count with smear or smudge cells. Bone marrow examination shows increased lymphocytes. Reticulocyte is high in autoimmune hemolytic anemia and Coomb's test is positive. Paraproteins may be increased and uric acid is also high, immunophenotyping- B cell antigen (CD19 and CD23) and T cell antigen (CD5). No treatment is necessary in asymptomatic case. If indicated, chlorambucil 5 mg daily, adjust the dose according to blood count. Fludarabine may be given. Prednisolone is given in marrow failure or autoimmune hemolytic anemia.
Radiotherapy: Local radiotherapy of lymph node causing discomfort or local obstruction and huge splenomegaly.
 
Case No. 249
  1. Stevens Johnson's syndrome.
  2. Toxic epidermal necrolysis.
  3. Stop carbamazepine.
    449
Note: Involvement of skin and mucous membrane of mouth is highly suggestive of Stevens Johnson's syndrome (or toxic epidermal necrolysis). In this patient, it is likely to be due to carbamazepine therapy. Stevens Johnsons' syndrome is a severe form of erythema multiforme with widespread bullous lesion in skin and mucous membrane of mouth, eyes, genitalia associated with severe constitutional symptoms. In this case, <10% body surface area (BSA) is involved. When >30% BSA is involved, it is called TEN. Most commonly induced by same medications. Patient initially presents with SJS, may progress to TEN.
It is often idiopathic, but may be associated with viral infections, leukemia, lymphoma and drugs (sulphonamides and anticonvulsants).
Treatment: Carbamazepine is stopped. Symptomatic (I/V fluid, anti-pyretic, antibiotic). Local care of eyes and mouth. In severe cases- IV immunoglobulin. Steroid- its use is controversial. Can be used and tapered rapidly. Because, once skin loss has occured, it may aggravate morbidity and mortality, because of immunosuppression.
 
Case No. 250
  1. Mixed restrictive and obstructive airway disease.
  2. Pleural thickening, respiratory muscle weakness and chest wall disease (such as thoracoplasty or scoliosis).
  3. Chest X-ray.
Note: In this case, FEV1 and FVC are both reduced, producing a restrictive pattern. Also, there is low RV. TLco is also reduced in restrictive defect. But high Kco is due to extrapulmonary restriction. This can be caused by pleural disease like thickened pleura, respiratory muscle weakness and chest wall disease like thoracoplasty or scoliosis.
Emphysema produces an obstructive airway defect on spirometry with increased RV, but decreased TLco.
Cryptogenic firbrosing alveolitis produces a restrictive pattern, but Kco is reduced, although it can be normal but is never elevated.
 
Case No. 251
  1. Cryptogenic fibrosing alveolitis.
  2. Restrictive airway disease.
  3. Chest X-ray, CT scan of chest, lung biopsy.
Note: In restrictive lung disease, ratio of FEV1/FVC is normal, or increased, but low TLC, low transfer factor for carbon monoxide. 450In this case, lung function demonstrates a restrictive defect with an increased ratio of FEV1/FVC and decreased lung volumes. Both Tlco and Kco are reduced. Tlco is sensitive (but not specific), indicator of the integrity of alveolar capillary membrane and gas exchange function of the lung.
 
Case No. 252
  1. Tumor lysis syndrome (TLS).
  2. Serum phosphate (high), calcium (low).
  3. Dialysis (mainly hemodialysis).
  4. Serum uric acid (may be high) and LDH (high level is considered a marker of bulky disease, with a risk of TLS).
Note: Tumor lysis syndrome (TLS) is characterized by hyperuricemia, hyperkalemia, hyperphosphatemia, and hypocalcemia and is caused by the destruction of a large number of rapidly proliferating neoplastic cells. Acidosis may also develop. Acute renal failure occurs frequently.
TLS is most often associated with the treatment of Burkitt's lymphoma, acute lymphoblastic leukemia and high-grade non-Hodgkin's lymphoma, but also seen in chronic leukemias and rarely, with solid tumors. This syndrome may be found in patients with chronic lymphocytic leukemia after treatment with nucleosides. TLS has been observed with administration of glucocorticoids, hormonal agents such as letrozole and tamoxifen, and monoclonal antibodies such as rituximab and gemtuzumab. TLS usually occurs during or shortly (1 to 5 days) after chemotherapy. Rarely, spontaneous necrosis of malignancy can cause TLS.
Effective treatment with chemotherapy kills malignant cells and leads to increased serum uric acid levels from the turnover of nucleic acids. Owing to the acidic local environment, uric acid can be precipitated in the tubules, medulla, and collecting ducts of the kidney, leading to renal failure. The finding of uric acid crystals in the urine is a strong evidence for uric acid nephropathy.
Hyperphosphatemia, which can be caused by the release of intracellular phosphate pools by tumor lysis, produces a reciprocal depression in serum calcium, which causes severe neuromuscular irritability and tetany. Deposition of calcium phosphate in the kidney and hyperphosphatemia may cause renal failure. Potassium is the principal intracellular cation, and massive destruction of malignant cells may lead to hyperkalemia.451
Treatment: Recognition of risk and prevention are the most important steps in the management of this syndrome. Preventive measures are allopurinol, urinary alkalinization, and aggressive hydration. Care should be taken to prevent worsening of symptomatic hypocalcemia by induction of alkalosis during bicarbonate infusion. Dialysis is often necessary and should be considered early in the course. Hemodialysis is preferred.
Prognosis is excellent and renal function recovers after the uric acid level is lowered to 10 mg/dL.
 
Case No. 253
  1. Secondary polycythemia.
  2. Doppler ultrasonography of left lower limb, venography of left lower limb.
  3. Red cell mass estimation, arterial blood gases, erythropoietin level.
Note: This is likely to be a case of secondary polycythemia, as suggested by high hemoglobin and a raised PCV. Secondary polycythemia is due to raised levels of circulating erythropoietin (due to living at high altitude, chronic lung disease, or cyanotic heart disease) or due to inappropriate secretion of erythropoietin from various sources (such as renal cell carcinoma, polycystic kidney disease, hepatocellular carcinoma, cerebellar hemangioblastoma). In these cases, white cells and platelets are unaffected.
In pseudopolycythemia, both Hb and PCV are raised owing to hemoconcentration, resulting from any cause of reduced extracellular volume (dehydration, diuretics). Pseudopolycythemia can be differen-tiated from true polycythemia by red cell mass estimation, which is normal in former. In polycythemia rubra vera, in addition to raised Hb and red cell count, white cells are also increased in 70%, and platelets in 50% of cases. Physical examination may show palpable spleen.
Bone marrow examination will show erythroid hyperplasia and increased megakaryocytes, leukocyte alkaline phosphatase score is also high.
 
Case No. 254
  1. Spontaneous pneumothorax.
  2. Chest X-ray, ECG.
  3. Marfan's syndrome.
  4. Dissecting aneurysm.
    452
Note: In any young patient presenting with severe central chest pain, one should consider spontaneous pneumothorax. Appearance of this patient is highly suggestive of Marfan's syndrome, in which pneumothorax may occur, also dissecting aneurysm may occur rarely.
Marfan's syndrome is characterized by triad of eye, skeletal and cardiac abnormalities. (1) Eye- Blue sclera, sublaxation or dislocation of lens (ectopia lentis), iridodonesis (tremor of iris), heterochromia iris (various color of iris). Myopia, retinal detachment, glaucoma. (2) Skeletal- Tall, lean and thin, arachnodactyly. Hyperextensibility of joints, high arched palate, kyphosis or scoliosis. High pedal arch, pes planus, pectus excavatum or carinatum or asymmetry of chest. Arm span is greater than height and pubis-sole of foot is greater than pubis-vertex. (3) CVS-AR (due to aortic root dilatation, secondary to cystic medial necrosis), MR, mitral valve prolapse.
Aortic dissection or rupture are the commonest cause of death. Patient with Marfan's syndrome have a higher incidence of pneumothorax than the general population.
Marfan's syndrome may cause dissecting aneurysm, infective endocarditis and it may be associated with coarctation of aorta.
 
Case No. 255
  1. Cystic fibrosis.
  2. Meconium ileus equivalent.
  3. Sweat test.
  4. Pseudomonas aeruginosa.
Note: Diagnosis of cystic fibrosis should be suspected in any young patient who presents with chronic respiratory and chronic gastrointestinal problem. This girl is suffering from recurrent chest infections, bronchiectasis, malabsorption due to pancreatitis. She is also diabetic, secondary to pancreatic disease. Gastrointestinal problem in patient with cystic fibrosis is due to pancreatic insufficiency.
Diagnosis of cystic fibrosis is made by a sweat test. Sweat sodium concentration over 60 mmol/L is indicative of cystic fibrosis.
P. aeruginosa is the commonest organism, causing recurrent respiratory infection.
Male with cystic fibrosis are infertile, due to failure of development of vas deferens and epididymis. About 20% of female with cystic fibrosis are infertile.453
Cystic fibrosis is an autosomal recessive disease with a prevalence of 1/2,500, characterized by viscid respiratory and gastrointestinal secretions. Abnormality is in the gene encoding a chloride ion channel in the nasal epithelium, lungs, salivary glands, pancreas, intestine and bile ducts.
Clinical features are (i) Respiratory- recurrent infection, bronchiectasis, lung abscess, asthma, otitis media, nasal polyps, etc. (ii) Abdominal- meconium ileus (birth), rectal prolapse (neonate), steatorrhea (due to pancreatic insufficiency), malabsorption, gall stone, secondary biliary cirrhosis. There may be meconium ileus equivalent (infancy onwards), which is a bowel obstruction resulting from a combination of steatorrhea and viscid intestinal secretions, causing fecal impaction in ascending colon or in ileocecal junction.
Treatment: Pancreatic supplements, regular antibiotic, strict glucose control, mucolytic and regular physiotherapy with postural drainage.
If the patient presents with acute abdomen due to intestinal obstruction- nothing by mouth, intravenous fluid, nasogastric suction should be given.
Other treatment: Acetylcysteine given intravenously or through the nasogastric tube has been shown to be very useful in resolving bowel obstruction.
Lung transplantation, if there is severe respiratory compromise.
 
Case No. 256
  1. Achalasia of esophagus with aspiration pneumonia.
  2. Barrium swallow X-ray of the esophagus, endoscopy and biopsy, esophageal manometry.
Note: Achalasia of esophagus or cardia is a motility disorder, characterized by failure to dilate the lower esophageal sphincter due to absence or reduction of ganglion cells of Auerbach's plexus. Cause is unknown. There is failure of non-adrenergic non-cholinergic (NANC) innervation related to abnormal nitric oxide synthesis within the lower esophageal sphincter. Degeneration of ganglion cells within the sphincter and body of esophagus occurs. Common in middle age, but may occur at any age. Patients present with dysphagia, which is intermittent initially, worse for solid. Later, both solid and liquid. There is regurgitation of food, chest discomfort or pain (due to spasm). Sometimes, severe chest pain due to esophageal spasm (vigorous achalasia), repeated respiratory infection or aspiration pneumonia, loss 454of weight. Manometry is the confirmatory investigation in achalasia cardia. Complications- respiratory (recurrent aspiration pneumonia, bronchiectasis, collapse of lung), carcinoma of esophagus (squamous type in 5 to 10%), malnutrition (due to dysphagia).
Treatment: (i) Endoscopic dilatation by pneumatic bougie, may be repeated if necessary. Effective in 80% of cases, (ii) If repeated dilatation fails - surgery, open or laparoscopic. Heller's cardiomyotomy is done (circular muscle layer is cut). This may be complicated by perforation and reflux esophagitis. So, sometimes myotomy plus partial fundoplication may be done to prevent reflux esophagitis. Proton pump inhibitor (omeprazole) should be used, (iii) Endoscopic guidance of injection botulinum toxin in lower esophageal sphincter may be given. It helps in remission, but relapse may occur.
 
Case No. 257
  1. Dementia of Lewy body type.
  2. MRI of brain, thyroid function test, vitamin B12 assay.
Note: This patient has Parkinson's disease, hallucination and frequent falls (syncope), which is highly suggestive of dementia of Lewy body type. It is found in 10 to 20% of cases of dementia. Lewy bodies are intracytoplasmic inclusion bodies, composed of α-synuclein and ubiquitin, found in cerebral cortex and brainstem. Main clues to the diagnosis are severe neuroleptic sensitivity reactions, and presence of visual hallucinations for people and animal. Features of Parkinsonism are usually mild, memory impairment is also mild in early stage. Daytime drowsiness and sleepiness, falls, syncope and delusions may be present. There is overlap between Parkinsonism, Lewy body dementia and Alzheimer's disease. Day time sleepiness and drowsiness are more common in Lewy body dementia.
 
Case No. 258
  1. Hypogonadotrophic hypogonadism.
  2. History of anosmia.
  3. Kallmann's syndrome.
Note: Kallmann's syndrome is a rare disease, inherited as X-linked recessive trait, may be variable. It is characterized by deficient gonadotrphin production by the pituitary, anosmia, cleft palate and color blindness. This patient has primary amenorrhea, with 455hypogonadotrophic hypogonadism, but normal prolactin. Most likely diagnosis is Kallmann's syndrome.
 
Case No. 259
  1. MRI of brain, lumbar puncture and CSF study.
  2. Normal pressure hydrocephalus.
  3. CSF drainage via repeated lumbar puncture.
Note: Triad of dementia, gait disturbance and incontinence is highly suggestive of normal pressure hydrocephalus. CT scan of brain shows enlarged ventricles with no obstruction to CSF. Lumbar puncture will show normal CSF pressure and normal composition. In this condition, dilatation of ventricular system is caused by intermittent rise of CSF pressure, related to impaired CSF absorption, occurs particularly at night and therefore may benefit from CSF shunting, but unpredictable. It occurs in old age.
 
Case No. 260
  1. Iodine deficiency goitre with menopausal syndrome.
  2. USG of thyroid gland, FNAC of thyroid, antithyroid antibody.
  3. Thyroxin therapy.
Note: Normal FT3, FT4 and TSH with high radioiodine uptake is highly suggestive of iodine deficiency goitre. It may be confused with thyrotoxicosis, because of high radioiodine uptake. It is one of the commonest cause of diffuse goitre in endemic area. Intake of iodinated salt is the simplest way to prevent such goitre.
 
Case No. 261
  1. Leptospirosis.
  2. Dengue hemorrhagic fever, viral hepatitis due to E virus, falciparum malaria.
  3. Blood culture, urine culture, anti-leptospira antibody (or microscopic agglutination test- MAT).
  4. DIC.
Note: Evidence of hepatitis, renal failure and carditis is highly suggestive of leptospirosis. However, fulminant hepatic failure complicated by hepatorenal syndrome may follow viral hepatitis.
Leptospirosis is caused by the spirochete leptospira ictero-hemorrhagica. Rodents, particularly rats, are the most important 456reservoir of infection. The organism is excreted in the urine and may survive in the soil for several weeks. Entry into the human host is through cuts and abrasions on the skin, or through intact mucous membranes. Occupations- most susceptible are sewerage workers, fishermen, vets and farmers. Replication occurs in the blood and tissue and multi-system involvement may occur, the kidneys and liver are commonly affected. Glomerular injury occurs, causing an acute interstitial nephritis and tubular necrosis. In liver, there is centrilobular necrosis in severe cases.
The incubation period is 1 to 2 weeks.
Initial or septicemic phase is 4 to 7 days, characterized by fever, headache, myalgia, abdominal pain, vomiting, skin rash (macular, maculopapular or hemorrhagic) or conjunctival ingestion (blood shot eyes). Persistent headache may be due to meningitis. Proteinuria and hematuria may be present during this phase, and renal failure is evident in 50% of patients. Jaundice and impairment of liver function are present only in severe cases. 90% cases are anicteric. Hepatosplenomegaly is present in approximately 20%. Respiratory involvement is common, and manifests as dry cough, hemoptysis and confluent shadow in chest X-ray.
Second phase or immune phase last for 4 to 30 days. The patient is afebrile. Antibody titres to leptospira are rising. Deterioration in liver and renal function may continue. Meningism, uveitis and rash are common. Hematological manifestations include thrombocytopenia, hemolytic uremic syndrome characterized by fragmented red cells on the blood film and intravascular hemolysis. Endothelial injury may cause blood loss from the gastrointestinal tract and lungs.
Blood culture in special media is positive in the first ten days. By the second week, urine culture is positive. IgM antibodies to leptospira are detectable in the first week. Microscopic agglutination test (MAT) is the investigation of choice. PCR is also helpful in blood and urine.
Treatment: Intravenous benzyl penicillin (1.5 mega unit 6 hourly) for one week. I/V ceftriaxone 1 gm daily equally effective. Doxycycline 100 mg 12 hourly for one week is also helpful. Renal failure may require dialysis.
 
Case No. 262
  1. Heart failure.
  2. Cardiomyopathy, most likely hypertrophic cardiomyopathy.
  3. Friedreich's ataxia.
    457
Note: In young patient with pes cavus plus combination of cerebellar lesion (bilateral), UMN lesion (extensor plantar) and posterior column lesion (loss of vibration and position senses) is highly suggestive of Friedreich's ataxia. Differential diagnoses are multiple sclerosis, tabes dorsalis, spinocerebellar degeneration. Friedreich's ataxia is a type of hereditary ataxia, inherited as autosomal recessive (in some cases AD). Cause is unknown. Family history may be present. Features of Friedreich's ataxia are usual onset in young, <15 years (8 to 16 yrs). There is progressive difficulty in walking (truncal ataxia and ataxia of lower limbs), weakness of lower limbs, dysarthria. Diabetes mellitus is common. May be associated with kyphoscoliosis, pes cavus, cocking of toes, nystagmus, optic atrophy, hypertrophic cardiomyopathy. Normal mentation (may have mild dementia). Prognosis—usually progresses slowly, death occurs before 40 years of age (usually 20 years after the onset of symptoms).
 
Case No. 263
  1. Infective endocarditis.
  2. Serial blood culture, transesophageal echocardiogram (TOE).
Note: Any patient with fever, splenomegaly with cardiac murmur, infective endocarditis should be considered first. Blood culture is the single most important investigation, positive in 90% cases. Serial blood cultures should be performed, a single blood culture may be negative. Transthoracic echocardiography is useful to detect vegetation, which is more sensitive. However, absence of vegetation does not exclude endocarditis, as vegetations <3 mm may not be seen in transthoracic echocardiogram. Routine blood culture may be negative, due to prior treatment with antibiotics before culturing the blood, or that the endocarditis is due to a fastidious organism. In immunosuppressed patient, endocarditis may be due to fungus that may be difficult to culture. Endocarditis may be a manifestation of SLE (Libmann Sacks endocarditis) or a manifestation of malignancy, called marantic endocarditis.
Features of endocarditis—fever, cardiac murmur, splinter hemorr-hages, Osler's nodes (small, painful), Janeway lesions (macular, painless), Roth spots, splenomegaly, microscopic hematuria, nephritis.
Causes of culture negative endocarditis – Fungus, brucella, coxiella burnetti, chlamydia, mycoplasma, legionella, HACEK organisms.
Causes of non-infective endocarditis – Marantic endocarditis, SLE (Libman-Sacks).458
 
Case No. 264
  1. Magnesium toxicity.
  2. Stop magnesium infusion, start IV fluid, diuretic, injection calcium gluconate.
  3. Serum magnesium level and creatinine.
Note: Signs of hypermagnesemia include nausea, hypotension (due to myocardial depression and vasodilatation), bradycardia, drowsiness, respiratory depression. In severe cases, double vision, loss of deep tendon reflexes, coma and cardiac arrest may occur. Magnesium toxicity is more common when intravenous magnesium is given in the absence of hypomagnesemia such as preeclampsia and toxicity is exacerbated in presence of renal failure.
Treatment: Stop any drugs containing magnesium, promote urinary excretion of magnesium by loop diuretic, plenty of fluid, IV Ca-gluconate may be given to prevent cardiac arrest, dialysis may be needed in severe renal failure.
 
Case No. 265
  1. Prolactinoma, drug induced hyperprolactinemia.
  2. MRI of brain.
  3. Drug induced hyperprolactinemia (due to metoclopramide or domperidone).
  4. Stop antiemetic drug.
Note: In this young girl, after antiemetic drug, more likely domperidone or metoclopramide, she has developed galactorrhea and amenorrhea, with hyperprolactinemia. This is highly likely to be due to drug induced hyperprolactinemia. Pregnancy should be excluded, next possibility is prolactinoma. Prolactin above 5000 mU/L is suggestive of a prolactinoma.
Features of hyperprolactinemia in female are galactorrhea, secondary amenorrhea, inferlity, etc. in male, decreased libido, less secondary sexual character, lethargy.
 
Case No. 266
  1. Primary hyperaldosteronism (or Conn's syndrome).
  2. High resolution CT scan of suprarenal gland, lying and standing plasma renin and aldosterone.
  3. Spironolactone.
459Note: The patient has hypokalemic alkalosis with hypertension, highly suggestive of primary hyperaldosteronism. It is also called Conn's syndrome, caused by excess secretion of aldosterone by adrenal gland, which leads to salt and water retention, causing hypertension. Also, it causes hypokalemia resulting in muscular weakness, polyuria, polydipsia. Common causes is adrenal adenoma, sometimes may be due to hyperplasia of adrenal gland. High resolution CT scan of suprarenal gland is a very helpful investigation, >0.5 cm adenoma may be detected. Simultaneous measurement of serum renin and aldosterone in lying and standing position should be done. Renin is low, aldosterone is high in supine position. On standing suddenly, there is paradoxical drop of aldosterone level in adenoma, but exaggerated high aldosterone in patient with hyperplasia.
Treatment: If adenoma, surgery should be done. In hyperplasia, aldosterone antagonist like spironolactone should be given.
Causes of hypokalemic alkalosis with hypertension- primary hyperaldosteronism, Cushing's syndrome, Liddle's syndrome, hyper-tension treated with diuretic, renal artery stenosis, carbenoxolone therapy, congenital adrenal hyperplasia (11-β-hydroxylase deficiency).
 
Case No. 267
  1. Kikuchi's disease.
  2. Steroid should be given.
Note: Necrotising lymphadenitis is suggestive of Kikuchi's disease. It is a rare, benign disorder of unknown etiology, characterized by enlargement of commonly cervical lymph nodes with or without systemic features. It is also known as histiocytic necrotising lymphadenitis. Common in female.
Common manifestation of Kikuchi's disease is lymphadenopathy. It is usually localized, commonly affecting the cervical lymph nodes (80%), specially the posterior chain. Generalized involvement affecting axillary, inguinal and mesenteric nodes may occur. In 50% cases, lymphadenopathy is accompanied by fever and a flulike prodrome. There may be headache, nausea, vomiting, malaise, arthralgia, myalgia, night sweat, rash, abdominal or thoracic pain and weight loss.
There may be maculopapular lesions, morbilliform rash, nodules, urticaria, and malar rash (resembling SLE), hepatosplenomegaly. Involvement of bone marrow, uvea, thyroid, parotid glands and myo-cardium is rare. Neurologic involvement include aseptic meningitis, acute cerebellar ataxia and encephalitis. Clinically, KD may be confused 460with lymphoma, tubercular lymphadenitis and SLE. Other differential diagnoses include viral or bacterial lymphadenitis, metastatic carcinoma, rheumatoid arthritis, infectious mononucleosis, sarcoidosis, cat scratch disease, Still's disease, tuberculosis.
CBC may show mild granulocytopenia or atypical lymphocytes. ESR and CRP may be elevated. LDH may be elevated, if liver is involved. Immunological studies for autoantibodies are generally negative. FNAC or biopsy of lymph nodes shows crescentic histiocytes, plasmacytoid monocytes and extracellular debris, that suggest the diagnosis of Kikuchi disease.
Treatment is generally supportive. Reassurance is important. Fever and pain may be treated with NSAIDs. Corticosteroid is indicated in patient with severe extranodal disease, involvement of liver (elevated LDH) or nervous system, severe lupus-like syndrome (positive ANA) or generalized KD. Prednisolone 50 to 60 mg daily orally with tapering as symptoms resolve. Immunosuppressive drugs are used with steroid in severe, life-threatening cases.
Some patients respond to minocycline, ciprofloxacin, chloroquine and hydroxychloroquine.
Kikuchi's disease is generally benign and self-limiting with a good prognosis. It usually resolves over several weeks to 6 months. Recurrence is unusual (3%) and fatalities are rare.
 
Case No. 268
  1. Multiple myeloma.
  2. Autonomic neuropathy.
  3. Bone marrow study.
Note: Anemia, high ESR, hypercalcemia associated with severe bony pain are highly suggestive of multiple myeloma. Patient also has heart failure and nephrotic syndrome, which may occur in multiple myeloma with amyloidosis. Low sodium in this case is due to pseudohyponatremia, which occurs due to paraproteinemia in myeloma. Amyloidosis may cause autonomic neuropathy, which is responsible for dizziness or syncope.
 
Case No. 269
  1. Sumatriptan subcutaneous or intranasal, high flow oxygen.
  2. Cluster headache, migraine.
  3. Verapamil.
461Note: Most likely diagnosis is cluster headache. This is characterized by recurrent episodes of unilateral retro-orbital pain with associated autonomic features. Cluster headache can occur at any age, but common in 30 to 40 years, more in male, male:female is 9:1. Pain is severe, last for 15 minnutes to 3 hours, radiates up over the frontotemporal region down to the jaw, neck or shoulder, associated with autonomic features like lacrimation, facial flushing, nasal or conjunctival congestion, meiosis. Ipsilateral eye is often red and watery, rhinorrhea or a blocked ipsilateral nostril.
Treatment in acute attack: Sumatriptan subcutaneous or intranasal and high flow oxygen 100% (10 to 12 L/min). Ergotamine, verapamil, lithium, sodium valproate and prednisolone have been used as prophylaxis, once the acute attack has resolved. Verapamil is the drug of choice for prophylaxis.
Differential diagnosis is migraine. Also, it may be confused with chronic paroxysmal hemicrania (CPH, which is treated with indomethacin). Features that distinguish CPH are: shorter duration of attacks (2 to 45 minutes), increased frequency of attacks, female preponderance and selective response to treatment with indomethacin.
 
Case No. 270
  1. Another myocardial infarction, acute pericarditis, rupture of chorda tendineae or papillary muscle causing mitral regurgitation, rupture of ventricular septum causing VSD, pulmonary embolism.
  2. Pericardial rub to detect acute pericarditis, pansystolic murmur due to mitral regurgitation (or due to VSD).
  3. ECG, transesophageal echocardiogram (TOE).
Note: In any patient with acute myocardial infarction, new onset of chest pain may be due to new myocardial infarction, acute pericarditis, rupture of the chorda tendineae or papillary muscle causing mitral regurgitation, pulmonary embolism. Presence of pansystolic murmur indicats mitral regurgitation due to papillary muscle rupture or ventricular septal defect (due to rupture of interventricular septum). Echocardiography, mainly transthoracic echocardiogram is preferable to transesophageal echocardiogram. Acute pericarditis is another complication, that occurs on 3rd or 4th day, a pericardial rub may be present. ECG will shows elevation of ST, with upward concavity. Pulmonary embolism should also be considered in such case.462
 
Case No. 271
  1. T3 thyrotoxicosis.
  2. Serum FT3 estimation.
Note: This patient's features are suggestive of thyrotoxicosis, TSH is very low, normal FT4 but high radioiodine uptake. T3 thyrotoxicosis is the likely diagnosis. The diagnosis of T3 toxicosis should be suspected in patient presenting with symptoms of thyrotoxicosis, with or without goitre, in whom serum FT4 is normal, low TSH with high radioiodine uptake.
 
Case No. 272
  1. Superior vena caval obstruction.
  2. Lymphoma.
  3. Chest X-ray P/A view.
  4. CT scan of chest, FNAC or biopsy of lymph node.
Note: The symptoms and signs in this patient are highly suggestive of superior vena caval obstruction, secondary to mediastinal lympha-denopathy. This is the common cause of SVC obstruciton in early age. In elderly, commonest cause is bronchial carcinoma. First chest X-ray should be done, which will show mediastinal widening. CT scan of chest and also CT guided FNAC may be done. Blood count is done to exclude acute lymphoblastic leukemia. If lymphadenopathy is present, FNAC or biopsy of the lymph node should be done to confirm the diagnosis.
Treatment: High dose steroid, chemotherapy, radiotherapy to the mediastinum. To relieve SVC obstruction - IV frusemide, dexamethasone, expandable metalic stenting may be used.
 
Case No. 273
  1. Panacytopenia due to azathioprine toxicity.
  2. Septicemia with consolidation.
  3. Broad spectrum antibiotic and azathioprine should be stopped temporarily.
  4. Dose of azathioprine should be reduced.
Note: Allopurinol acts by inhibition of xanthine oxidase and thus inhibit the metabolism of 6-mercaptopurine, an active metabolite of azathioprine. Concomitant use of allopurinol with azathioprine is not recommended. If concomitant use is necessary dose of azathioprine 463should be reduced along with regular blood count monitoring. Azathioprine toxicity - commonly bone marrow suppression, blood dyscrasias such as thrombocytopenia, leucopenia and agranulocytosis can also occur.
Ciclosporin levels can also rise with concomitant administration of allopurinol, but to lesser extent. Ciclosproin toxicity more commonly causes renal toxicity.
 
Case No. 274
  1. Family history.
  2. Huntington's disease.
  3. MRI of brain, DNA analysis.
Note: Chorea may be due to many causes. In this case, there is no other history and taking any drugs. In Huntington's disease, chorea is associated with progressive dementia. It is inherited as autosomal dominant. Gene responsible is on short arm of chromosome 4.
Symptoms typically appear between 30 and 50 years of age. Genetic testing now provides an accurate method of establishing the diagnosis. Pathological changes - Cerebral atrophy with neuronal loss in caudate nucleus and putamen. Changes of neurotransmitters- (i) Reduction of acetylcholine transferase and GAD (glutamic acid decarboxylase) in the corpus striatum. (ii) Depletion of gamma aminobutyric acid (GABA), substance P, angiotensin converting enzyme and metencephalin in substantia nigra. (iii) High somatostatin level in corpus striatum. CT scan or MRI shows atrophy of caudate nucleus and also cerebral atrophy.
Treatment: No curative treatment. Haloperidol or phenothiazine for dyskinesia. Tetrabenazine may be given. Psychological support. Institutional care for dementia. Genetic counseling is essential. Average life span after clinical onset is about 15 years.
 
Case No. 275
  1. Amyloidosis.
  2. Renal transplantation.
  3. It can be reduced by using high flux dialysis membranes in patients who are likely to be on dialysis for a prolonged period.
Note: Amyloidosis is a common cause of neurological impairment in patient on longstanding dialysis. It is caused by beta 2 microglobulin accumulation. It causes joint pain and stiffness, usually upper limbs 464are involved more than lower limbs. The only treatment is renal transplantation. It can be reduced by using high flux dialysis membranes in patients who are likely to be on dialysis for a prolonged period.
Uremia may cause neurological symptoms. This is more common in men, and mostly affects the Iegs. Sensory symptoms (paraesthesia, burning sensations and pain) occur before motor symptoms (muscle atrophy, myoclonus, paralysis). Sensory symptoms may improve with starting or increasing the frequency of dialysis, but the motor symptoms are not reversible.
 
Case No. 276
  1. Microangiopathic hemolytic anemia.
  2. Reticulocyte count.
  3. Metallic Valve.
  4. Anemic heart failure.
Note: In this patient, blood picture shows anemia, high bilirubin and LDH, but other normal enzymes, highly suggestive of hemolytic anemia. Blood film suggest microangiopathic hemolytic anemia. In any patient with replacement of metallic valve, mechanical hemolytic anemia may occur as a complication. In mild case, no interference is necessary. If severe anemia, metallic valve replacement may be necessary by tissue valve.
 
Case No. 277
  1. Methanol poisoning.
  2. CT scan of brain, serum methanol level.
  3. Metabolic acidosis.
Note: As the patient is alcoholic and no history of other illness and there is severe metabolic acidosis, likely causes are methanol toxicity or ethylene glycol poisoning. Also, may be due to acute severe pancreatitis and acute hepatic failure. In this case, normal amylase excludes acute pancreatitis. Actual plasma osmolality is higher than the calculated plasma osmolality approximately 284 mOsm/l, suggesting the presence of an osmotically active substance, likely to be methanol or ethylene glycol. Methanol is metabolized to formic acid and ethylene glycol is metabolized to glycolic acid, which are relatively toxic.
Patient with methanol toxicity may complain of headache, nausea, vomiting, abdominal pain, dizziness, confusion, convulsion, stupor and coma. Formic acid can cause retinal injury. The patient suffers 465from reduced visual acuity, due to optic nerve damage by formic acid resulting from increased concentrations of formic acid. Mydriasis, reduced visual reflexes to light and hyperemia of optic disk are early features of methanol toxicity. If untreated, patient may develop blindness. Presence of visual abnormality in this patient indicates methanol poisoning (which is not a feature in ethanol poisoning).
Treatment: (i) Gastric lavage within 1st four hours, (ii) IV fluid, oxygen, (iii) Correction of acidosis- sodibicarb in large dose (alkalinization enhances formic acid excretion), (iv) In early stage- ethanol is given (it inhibits methanol oxidation by competing the inhibition of enzyme). Ethanol is given 10 ml/kg of 10% ethanol IV or 1 ml/kg of 95% ethanol orally. (v), Thiamine (100 mg qid), pyridoxine (50 mg qid) and folate (50 mg qid), (vi) Folinic acid 30 mg I/V every 6 hourly. It reduces ocular toxicity (accelerates metabolism of formic acid), (vii) Dialysis- indicated, if ingestion of methanol is >30 gm or metabolic acidosis or blood methanol >500 mg/L.
Patient with ethylene glycol poisoning may present in the same way. However, reduced visual acuity is not a feature, whereas flank pain and renal failure due to crystallization of oxalic acid in the renal tubules is common. Urinalysis under Wood's light in patient with ethylene glycol poisoning may reveal oxalate crystals in the urine, but an absence does not exclude the diagnosis. Patient with ethylene glycol poisoning also develop severe cardiac failure if untreated. Presence of visual symptoms favor methanol intoxication in this case.
 
Case No. 278
  1. Hypokalemic alkalosis.
  2. Laxative abuse, self-induced vomiting.
Note: This patient has hypokalemic alkalosis and an appropriate hypokaluria. Urea is elevated, due to hypoperfusion of afferent glomerular vessels. This may occur in dehydration. Most probable cause of hypokalemia is loss of potassium from gastrointestinal tract. This is also causing dehydration and hence the elevated urea. There is no history of diarrhea or vomiting, but for her obesity, probably she is abusing laxatives or has self-induced vomiting in order to loss weight. Potassium may be lost via GI tract (diarrhea, vomiting, intestinal fistulae, villous adenoma, nasogastric aspiration) or the kidneys. Other causes of hypokalemia may occur in primary and secondary hyperaldosteronism, Cushing's syndrome, Liddle's syndrome or drugs.466
Hypokalemia is generally associated with metabolic alkalosis, with the exception of RTA.
Many conditions associated with renal loss of potassium also cause hypertension. Most common cause of hypertension, hypokalemia and alkalosis are primary hyperaldosteronism (Conn's syndrome). However, other recognized causes include Cushing's, Liddle's syndrome acce-lerated hypertension, renal artery stenosis or hypertensive therapy with diuretics, Liddle's syndrome, carbenoxolone therapy, congenital adrenal hyperplasia (11-β-hydroxylase deficiency).
 
Case No. 279
  1. Bronchial carcinoma with metastasis to the hilar lymph nodes, sarcoidosis, lymphoma.
  2. CT scan of chest, FNAC of hilar lymph node (CT or USG guided).
  3. Bronchial carcinoma with metastasis to the hilar lymph nodes.
Note: This patient has persistent dry cough, weight loss, hypercalcemia which are suggestive of squamous cell carcinoma of lung. Unilateral hilar lymphadenopathy may due to sarcoidosis, where hypercalcemia may occur. Other causes of unilateral hilar lymphadenopathy are lymphoma, tuberculosis. But in these cases, usually there is no hypercalcemia. Sometimes, in early case with small lung lesion, plain chest X-ray may not reveal the lung lesion. CT scan is more helpful. Sputum for malignant cells, bronchoscopy and biopsy should be done.
 
Case No. 280
  1. Cerebral toxoplasmosis.
  2. Tuberculous meningitis, cryptococcal meningitis, cerebral lym-phoma, brain abscess, cerebral malaria.
  3. CT scan of brain, anti-HIV, CSF study for tuberculosis, lymph node FNAC or biopsy.
Note: The patient was working abroad, may be associated with a high level of promiscuity and high risk for HIV infection. Confusion may be due to encephalitis, meningitis, cerebral abscess or cerebral malaria.
Toxoplasma encephalitis is the commonest cause of focal brain disease in HIV patient. This patient is confused, has focal neurological signs and chorioretinitis, which are suggestive of cerebral toxoplasmosis, TB or cryptococcal meningitis. Cerebral lymphoma does not cause 467chorioretinitis. Other recognized causes of chorioretinitis in patient with HIV include CMV and syphilitic infection.
Treatment: Pyrimethamine plus sulphadiazine.
 
Case No. 281
  1. Drug induced SLE.
  2. Anti-histone antibody.
Note: History of this patient suggest SLE. Her ANA is strongly positive and anti-ds DNA is negative. Also, history of taking minocycline. All the features are suggestive of drug induced SLE. CRP is usually normal in SLE. If it is high, indicates bacterial infection (or serositis). Minocycline may cause SLE, in which CRP, ESR are high, and ANA is strongly positive, but anti-ds DNA is negative.
Features of drug induced SLE: Sex ratio—equal, lung involvement is common, but renal and neurological involvement are rare. ANA is usually positive, anti ds DNA is negative, complements are normal. Antihistone antibody is positive in 95% of cases (characteristic, but not specific).
Drugs causing SLE like syndrome usually do not aggravate primary SLE.
Treatment: Withdrawal of drugs. Short course of steroid is necessary.
Drugs causing SLE - Hydralazine is common (90%, slow acetylator), procainamide (rapid acetylator), anticonvulsant (carbamazepine, pheny-toin), phenothiazine, INH, oral contraceptive pill, ACE inhibitor, penicillamine, methyldopa, minocycline.
 
Case No. 282
  1. Behcet's syndrome.
  2. Crohn's disease, herpes simplex, virus infection.
Note: This patient has oral and genital ulcer, iritis, arthritis, erythema nodosum. There is episode of bloody diarrhea and deep venous thrombosis, all are suggestive of Behcet's syndrome. Crohn's disease is another possibility, where genital ulcer and deep venous thrombosis are rare, which are common in Behcet's syndrome.
Behcet's syndrome is a vasculitis of unknown cause, characterized by recurrent oral, genital ulcer, ocular lesion and skin, joint and neurological lesion. Common in male. Oral and genital ulcers are present in most patients.468
Ocular lesions are recurrent uveitis and iridocyclitis, retinal vascular lesions and optic atrophy, can lead to loss of vision in 50% of patients with ocular involvement. Erythema nodosum is a recognized feature. Other skin manifestations include diffuse pustular rash affecting the face, erythema multiforme. Pathergy test is a useful diagnostic sign. It is demonstrated by pricking the skin by needle, there is pustule formation at venepuncture site within 24 to 48 hours.
A seronegative arthritis occurs in about 40% patients, commonly involves knees, ankles and wrists. Recurrent thrombophlebitis is a significant feature of Behcet's syndrome, leading to venous thrombosis. Less often, superior or inferior vena caval thrombosis may occur. Abdominal pain and bloody diarrhea have also been documented. Asymptomatic proteinuria is a recognized feature, but on a few occasions may cause renal amyloidosis. Neurological complications occur in 5% patients. Organic confusional states, meningoencephalitis, transient or persistent brainstem syndromes, multiple sclerosis and Parkinsonian type disorders are all recognized.
Behcet's syndrome is a clinical diagnosis. There is no specific diagnostic test. Criteria for diagnosis—Recurrent oral ulcer, at least three times in twelve months plus 2 of the - (a) Recurrent genital ulceration, (b) Eye lesion, (c) Skin lesion, (d) Positive Pathergy test.
Genital ulcers and oral complications are treated with topical steroid. Thalidomide is effective in resistant oral and genital ulcer.
In severe cases, systemic steroid is necessary, together with azathioprine. Colcichine, ciclosporin and levamisole have also been used in the management of this condition.
 
Case No. 283
  1. Gastroesophageal reflux disease.
  2. Endoscopy.
Note: Gastroesophageal reflux disease (GORD) is one of the commonest causes of chronic cough.
Her clinical presentation is typical of cough secondary to GORD, in which symptoms are worsen in talking and laughing. Lower esophageal sphincter closes at night leading to cessation of symptoms. Asthma is also a possibility. Cough secondary to ACE inhibitors should take a maximum of 3 months to disappear after the drug is stopped.
Gastroesophageal reflux disease usually causes heart burn. May also cause odynophagia, sometimes atypical chest pain which can mimic angina.469
 
Case No. 284
  1. Cardiac arrest.
  2. ICD (implantable cardiac defibrillator).
Note: The patient is suffering from IHD, more likely ischemic cardiomyopathy. In such case, ICD implantation is the best treatment, as there is a clear morbidity and mortality benefit over the anti-arrythmic drugs. β-blocker, ACE inhibitor, amiodarone, statin, etc. have benefit to reduce mortality, but use of ICD should be the first line. ICD resembles large cardiac pacemaker, consists of a generator device and lead or leads that are implanted into the heart via subclavian or cephalic vein. They can automatically sense and terminate life threatening ventricular arrythmia. ICD have all the functions of pacemaker. In addition, it can be used to treat ventricular tachy-arrhythmias, using overdrive pacing synchronized cardioversion or defibrillator.
 
Case No. 285
  1. Presence of anti-Ro/SSA antibody from maternal serum.
  2. Pacemaker implantation.
  3. To know whether the mother is suffering from SLE.
Note: If the mother is suffering from SLE having anti-Ro/SSA antibody in the serum, the baby may have congenital complete heart block, due to transplacental transfer of this antibody into the baby. Occurs in < 3% cases. Heart block is usually permanent, insertion of pacemaker may be needed. Though in many cases of congenital heart block, heart rate remains high and no treatment is necessary. In any mother suffering from SLE, screening for anti-Ro/SSA antibody must be done, if she wants to be pregnant.
 
Case No. 286
  1. Strict glycemic control and control of hypertension.
  2. ACE inhibitor, angiotensin receptor blocker.
  3. Diabetic nephropathy.
Note: The patient have hypertension and probably microalbuminuria, that indicates nephropathy. Prevention of nephropathy is done by strict glycemic and blood pressure control. Good glycemic control delays the onset of diabetic nephropathy in patients who have no hypertension. In patient who has both diabetes and hypertension, diabetic nephropathy can be prevented by strict glycemic control and good control of 470blood pressure. Better glycemic control reduces glomerular basement membrane thickening and microalbuminuria.
If microalbuminuria has developed, there is little evidence that improving glycemic control alone delays progression of nephropathy. In such cases, control of blood pressure using ACE inhibitor or angiotensin II receptor blocker, can retard the progression of nephropathy.
In young type I diabetic with microalbuminuria or proteinuria, ACE inhibitor should be given regardless of blood pressure. The target blood pressure in microalbuminuria is <120/70 mm Hg and with proteinuria is < 130/75 mm Hg.
In patient with type 2 diabetes mellitus, angiotensin II receptor blocker prevent development of proteinuria in patient with microalbuminuria. Target blood pressure is <130/75 mm Hg.
Smoking is a risk factor for the development of microalbuminuria. However, there are no prospective studies showing the benefit of cessation of smoking on renal function in type 1 diabetics.
 
Case No. 287
  1. Q fever endocarditis.
  2. Antibody against coxiella burnetti.
  3. Doxycycline and rifampicin.
Note: Q fever endocarditis is due to Coxiella burnetii, a rickettsial zoonotic disease. It is common in farmer, due to close contact with animal products of farm animals. Aortic valve is usually affected, chronic infection may cause hepatitis, osteomyelitis or endocarditis. Features include finger clubbing, hepatosplenomegaly and purpura or leukocytoclastic vasculitic rash. Laboratory tests often show features of hepatitis, anemia, high ESR, thrombocytopenia and hyper-gammaglobulinemia. Microscopic hematuria may be present. The disease may be complicated by immune complex-mediated glomerulonephritis and arterial emboli. Diagnosis is best made serologically and a Phase I antibody titre to Coxiella burnetti (IgG and/or IgA) greater than 1:200 is virtually diagnostic of Q fever endocarditis. Treated with doxycycline and rifampicin, may require to continue the drugs for long time.
 
Case No. 288
  1. Chronic constrictive pericarditis.
  2. Restrictive cardiomyopathy.
  3. Echocardiography, cardiac MRI, cardiac catheterization.
    471
Note: This patient has signs of severe right sided heart failure (raised JVP, enlarged tender liver and edema), but chest X-ray reveals a normal-sized heart. Also, dyspnea on exertion, atrial fibrillation, ascites, all are typical of chronic constrictive pericarditis. Differential diagnosis of severe heart failure in the absence of significant cardiac enlargement is between pericardial constriction and a restrictive cardiomyopathy. Rarely, cardiac amyloidosis may occur, which is usually associated with multiple myeloma. In this case, patient is suffering from rheumatoid arthritis, also was suffering from tuberculosis, both may cause chronic constrictive pericarditis.
Causes of pericardial constriction are infection (tuberculosis and coxsackie B infection), hemopericardium (due to trauma, myocardial rupture after infarction, dissecting aneurysm), collagen disease (rheumatoid arthritis), cardiac operation, mediastinal irradiation, fungal infection (histoplasmosis), rarely, after acute purulent pericarditis and also idiopathic.
Signs of chronic constrictive pericarditis – (1) Pulse- low volume, tachycardia. Pulsus paradoxus may be present. (2) JVP- Raised. Kussmaul's sign positive (raised JVP on inspiration). Fall of Y descent (Friedrich's sign). (3) Precordium- No cardiomegaly. On auscultation- pericardial knock (a third heart sound), due to rapid ventricular filling. (4) Ascites- early feature, edema is a late feature (found in advanced stage). (5) Hepatomegaly.
Treatment: Surgery.
 
Case No. 289
  1. Paradoxical cerebral embolus via patent foramen ovale.
  2. Transesophageal echocardiography (TOE).
Note: This patient has long flight from Australia and therefore developed DVT, the thrombus passed through a patent foramen ovale, has caused left cerebral infarction. Other possibility of paradoxical embolus could be ASD, VSD, etc. 2D echocardiography may be unable to detect foramen ovale. Transesophageal echocardiography is the investigation of choice to detect patent foramen ovale, although transthoracic echocardiography with contrast may be an alternative. A patent foramen ovale occurs when the primum and secundum septa fail to fuse completely, leaving a small flap-like communication, allowing the possibility of shunt. It may be present in 30% of general population, may be associated with right to left intracardiac shunt, responsible for systemic embolism in such case.472
Treatment is closure using a mechanical device via right femoral vein.
 
Case No. 290
  1. 24 hours ECG monitoring, blood pressure drops of 20 mmHg during peak exercise tolerance testing.
  2. Ventricular tachycardia (VT), ventricular fibrillation (VF).
  3. Implantable automatic cardioverter defibrillator.
  4. Hypertrophic cardiomyopathy (HCM).
Note: This patient is suffering form hypertrophic cardiomyopathy, presented with dizzy spells, is at increased risk of sudden cardiac death due to VF or VT. 24 hours ECG monitoring may demonstrate non-sustained ventricular tachycardia, presenting in 25% cases of HCM and it is a high risk marker of sudden death. 5 poor prognostic markers which are predictive of sudden cardiac death are (1) Syncope, (2) Family History of HCM and sudden cardiac death, (3) Maximum left ventricular wall thickness >3 cm, (4) BP drop during peak exercise stress testing, (5) Documented runs of non-sustained VT on 24 hours tape.
This patient is a high risk group of sudden death, his father also died, more likely due to HCM. Ideal treatment is implantable automatic cardioverter defibrillator.
In HCM, 50% patients have chest pain and breathlessness, 20% present with dizziness or syncope. Sudden death is frequently the first presentation, particularly in adolescent, young adult and athlete. Overall, mortality rate is 3%. Sudden death commonly occurs during or immediately after strenuous exertion. Diagnosis is made using echocardiography to demonstrate left ventricular hypertrophy, which is usually asymmetric, but can be concentric in upto 40% of cases. M-mode echo shows systolic anterior motion of mitral valve, which is highly suggestive of HCM.
 
Case No. 291
  1. Diabetes lnsipidus.
  2. Urine osmolality.
  3. Hypernatremia should be corrected slowly by giving 5% dextrose in aqua infusion.
Note: In this patient, there is severe hypernatremia with elevated chloride, but normal potassium and urea. There is history of severe 473head injury. The likely cause of this presentation is diabetes lnsipidus. Urine osmolality is therefore likely to be low.
Head injury is causing cranial diabetes insipidus. In such case, plasma osmolality is high (> 300 mOsm/l), urine osmolality will be low (<150 mOsm/l). Also, plasma sodium may be normal or high and urine sodium is usually low. To confirm the diagnosis, water deprivation test should done, which will show inability to concentrate urine, which response to administration of desmopressin.
Causes of cranial diabetes insipidus are familial, trauma, sarcoidosis, tuberculosis, histiocytosis X, cervical tumor, DIDMOAD syndrome, pituitary hemorrhage.
 
Case No. 292
  1. Pancytopenia.
  2. Felty's syndrome, folate deficiency causing megaloblastic anemia, methotraxate causing bone marrow depression.
  3. Splenomegaly (in Felty's syndrome).
  4. Bone marrow study.
Note: This patient has been suffering from rheumatoid arthritis and is on DMARD. So, there may be bone marrow depression by methotraxate. Also, prolonged intake of methotraxate may cause folate deficiency responsible for pancytopenia. It may be also due to Felty's syndorme, which is characterized by triad of rheumatoid arthritis with splenomegaly with neutropenia. Her pancytopenia may be due to hypersplenism. Neutropenia is due to spleen produced antibody, which can shorten neutrophil life span, while large granular lymphocyte (LGL) can attack bone marrow neutrophil precursor. Felty's syndorme occurs in long-standing seropositive, deforming but inactive arthritis, in <1% of cases. It is more in female, may be associated with anemia, thrombocytopenia and abnormal liver functions.
 
Case No. 293
  1. Serial blood cultures and transesophageal echocardiography.
  2. Intravenous broad spectrum antibiotic, after sending blood culture.
Note: This is a clear case of infective endocarditis, which has deve-loped after prosthetic valve replacement, indicating early prosthetic valve endocarditis. Commonest organism in early prosthetic valve endocarditis is coagulase negative staphylococcus epidermidis, which 474is a normal commensal of skin. Others are staphylococcus aureus and fungi. Organisms in late prosthetic valve endocarditis are similar to those causing infection on native valves.
Microorganisms usually reach the prosthesis by direct contamination during intraoperative period or via hematogenous spread several days or weeks after surgery. Patient commonly develops valve dehiscence and annular abscess. The risk of embolic phenomenon is also greater.
For diagnosis, 3 sets of blood culture should be taken at intervals of >1 hour within the first 24 hours.
Transesophageal echocardiography is more sensitive test (1 to 2 mm size vegetation can be detected), then transthoracic echocardiography to identify vegetation in prosthetic valve. Also, helpful to identify complications, like paravalvular abscess and fistula formation.
Treatment: High dose intravenous antibiotics (vancomycin and genta-micin) for six weeks. The need for surgical intervention is much higher than with native valve endocarditis. Indications for surgery in prosthetic valve endocarditis- (i) early endocarditis in first 2 months or less after surgery, (ii) murmur suggestive of valve dysfunction, (iii) moderate to severe heart failure, (iv) annular or aortic root abscess or new cardiac conduction abnormalities on ECG, (v) persistent fever for 10 or more days despite appropriate antibiotic therapy, (vi) Staphylococcus aureus or fungi cultured from the blood.
 
Case No. 294
  1. Streptococcus bovis.
  2. Marantic endocarditis.
  3. Colonoscopy and biopsy.
Note: This patient is suffering from endocarditis. In addition to the symptoms of endocarditls, he is also having altered bowel habit, which is highly suggestive of large bowel malignancy. Streptococcus bovis is a normal commensal of gastrointestinal tract. However, S. bovis bacteremia and endocarditis has a strong association with GI malignancy.
As the patient is probably suffering from carcinoma of large bowel, if blood culture is negative, there may be endocarditis associated with malignancy, called marantic endocarditis.
Treatment: I/V benzyl penicillin, or flucloxacillin and gentamicin for four weeks.475
 
Case No. 295
  1. Cavernous sinus thrombosis.
  2. CT scan of orbit, blood for C/S.
  3. IV broad spectrum antibiotic, heparin, steroid IV.
Note: This patient has infection in upper lip followed by headache, fever and proptosis, all are highly suggestive cavernous sinus thrombosis.
Facial veins drain into the sinus and the most common source of infection is due to squeezing of nasal furuncle without antibiotic cover.
Other sources of infection include otitis media, sinusitis and dental infections. Commonest organism is Staphylococcus aureus. Patient with cavernous sinus thrombosis presents with severe periorbital headache, which also affects areas innervated by the ophthalmic and maxillary branches of trigeminal nerve. Fever and periorbital edema usually develop later. Ocular swelling, chemosis, ophthalmoplegia and drowsiness are recognized complications. Ophthalmoplegia is due to compression of the third, fourth and sixth cranial nerves. Headache with ophthalmoplegia are highly suggestive of cavernous sinus thrombosis.
The diagnosis of cavernous sinus thrombosis is either with high resolution CT scan of the orbit with contrast or a gadolinium-enhanced MRI scan of the orbit.
Treatment: Intravenous flucloxacillin and heparin. Corticosteroid may be used, which may improve inflammation around the cranial nerves.
Causes of cavernous sinus thrombosis—infections of the face, ear and sinuses, hyereditary thrombophilic states, hyperviscocity states, oral contraceptive pill, Behcet's syndrome.
 
Case No. 296
  1. Pregnancy.
  2. Urine for pregnancy test.
  3. Repeat FT3 and FT4.
Note: In case of recently married girl with weight gain, pregnancy must be excluded. In pregnancy, thyroxine binding globulin level is increased. So, her total T4 and T3 are elevated, but TSH level is normal. In pregnancy, FT3 and FT4 should be done.
 
Case No. 297
  1. Acute tubular necrosis (ATN)
  2. Increased urinary sodium excretion, urine osmolality: plasma osmolality 1:1.1.
    476
Note: In ATN, urine to plasma ratio osmolality should be <1.1, urinary sodium excretion is typically >60 mmol and urinary urea excretion is <160 mmol/L. If this patient had a physiological oliguria, there would still be preservation of urine concentration, with low urinary sodium. ATN is caused by renal ischemia, may be precipitated by hypovolemia, septicemia, drugs, rhabdomyolosis. It is usually characterized by oliguric renal failure, often reversible. Ischemia usually affects medullary area, therefore concentrating ability of the urine is diminished and sodium excretion is inappropriately increased.
 
Case No. 298
  1. Organophosphorus poisoning.
  2. Contaminated clothings should be removed, gastric lavage, air way should be cleared, high flow oxygen, injection atropine I/V.
  3. ECG, blood gas analysis, serum electrolytes, serum amylase and serum glucose.
Note: In this patient, it is likely to be a case of organophosphorus poisoning. Organophosphorus insecticides (e.g. malathion and para-thion) are irreversible inhibitors of acetylcholinesterase, resulting in accumulation of acetylcholine at muscarinic and nicotinic synapses. Muscarinic effects include nausea, vomiting, abdominal pain, urinary and fecal incontinence, miosis. Increased pulmonary parasympathetic tone can result in wheezing due to bronchial hypersecretion and bronchospasm. In severe poisoning, bradycardia, hypotension, heart block and pulmonary edema may occur. Increased parasympathetic activity to salivary glands leads to hypersalivation. Nicotinic effects include twitching, fasciculations, muscle weakness, tachycardia, hyper-tension. CNS effects are headache, dizziness, confusion, drowsiness, fit and coma. Atropine 1.8 to 3 mg IV as a bolus dose, followed by double the dose every five minutes until atropinized (clear lung, dry tongue, normal heart rate, normal BP, pupil dilated). Antidote like pralidoxime or obidoxome may be given in selected case.
 
Case No. 299
  1. Statin induced polymyositis.
  2. Rhabdomyolysis.
  3. Statin should be stopped.
Note: Myalgia, myositis and myopathy are all recognized effects of HMG-CoA reductase inhibitors (statins). There is an increased 477incidence, when statins are co-administered with a fibrate or when given to patients on immunosuppressants. Unless therapy is withdrawn, rhabdomyolysis may ensue, often associated with acute renal failure, secondary to myoglobinuria.
 
Case No. 300
  1. Takayasu's arteritis.
  2. Aortography.
Note: The history, high ESR, positive MT, may suggest tuberculosis. However, absent or feeble pulses, bruit in neck and femoral artery with high ESR in a young woman should raise the diagnosis of Takayasu's arteritis. Diagnosis is a clinical one and can be confirmed with aortography. Symptoms are systemic features (such as sweating, weight loss, anorexia and myalgia) and vascular insufficiency in the limbs affected. Physical signs include absence of pulse, usually in upper limb (also called reverse coarctation).
Takayasu's arteritis is a chronic inflammatory, granulomatous panarteritis of unknown cause, involving commonly the aorta and its major branches, carotid, ulnar, brachial, radial and axillary. Occasionally, may involve pulmonary artery, rarely abdominal aorta, renal artery resulting in obstruction. Famale:Male = 8:1, more common in parts of Africa and Asia. Mean age of onset is 29 years, but it can occur from any age between infancy and middle age. It is very uncommon after middle age.
Inflammation may be multi-segmental, interspersed with normal segments. Pathological changes are panarteritis, intimal hyperplasia, thickening of media, thickening of adventitia, and later on, fibrosis.
Hypertension is present in 50% cases. The causes of hypertension are renal artery stenosis, acquired coarctation and reduced distensibility of the aortic arch. Complications are those of long-standing hypertension, aortic regurgitation and congestive cardiac failure. Inflammatory markers are usually raised and there may be anemia of chronic disease.
Treatment: Prednisolone 40 to 60 mg daily or 1 to 2 mg/kg. If refractory to steroid or difficult to taper steroid, methotrexate upto 25 mg weekly. Reconstructive vascular surgery in selected cases.
 
Case No. 301
  1. Serum B12 estimation (also folic acid).
  2. Bone marrow study.
    478
Note: This patient has anemia with pancytopenia and raised MCV. With this picture, likely diagnosis is megaloblastic anemia due to B12 or folic acid deficiency.
 
Case No. 302
  1. Take family history of hemochromatosis.
  2. Liver biopsy for iron staining and quantitative measurement of iron.
  3. DNA analysis to establish hereditary hemochromatosis.
Note: Hemochromatosis may present with generalized pigmentation only. Serum iron, ferritin all are very high in this case. TIBC also shows full saturation. Family history may be positive.
 
Case No. 303
  1. Whipple's disease.
  2. By demonstration of Tropheryma whippelii within the mucosa of a duodenal or jejunal biopsy via PAS staining.
Note: Whipple's disease is characterised by infiltration of small intesti-nal mucosa by ‘foamy’ macrophages which stain positive with periodic acid-schiff (PAS) reagent. The disease is a multisystem one and almost any organ can be affected, sometimes long before gastrointestinal involvement becomes apparent.
Electron microscopy reveals small Gram-positive bacilli (Tropheryma whipplei) within the macrophages. Villi are widened and flattened, densely packed macrophages occur in the lamina propria. These may obstruct lymphatic drainage, causing fat malabsorption.
Treatment: Cotrimoxazol or tetracycline is used.
 
Case No. 304
  1. Coeliac disease.
  2. Endoscopy and jejunal biopsy.
Note: Coeliac disease may be linked with type 1 DM. She has iron deficiency anemia, which supports the diagnosis. In some case of Coeliac disease, there may not be any GIT symptom.
 
Case No. 305
  1. Megaloblastic anemia with cardiac failure.
  2. Serum vitamin B12 and folate level.
    479
  3. Blood transfusion (this should be done very cautiously under diuretic cover).
  4. Pulmonary edema.
Note: This patient is severely anemic, which is the cause of heart failure. Correction of anemia with blood transfusion, preferably packed cell volume should be the first measure.
 
Case No. 306
  1. Methotrexate.
  2. Folic acid deficiency.
Note: The patient is taking methotrexate which may cause macrocytosis due to folate deficiency. Folinic acid should be given.
 
Case No. 307
  1. Amiodarone induced hypothyroidism.
Note: This is most likely a case of amiodarone induced hypothyroidism. Amiodarone which is used for atrial fibrillation, inhibits the peripheral conversion of T4 to T3.
 
Case No. 308
  1. Rhabdomyolysis due to trauma after fall.
Note: The patient has become unconscious in the street. He has renal impairment. There is high AST. These features are consistent with rhabdomyolysis due to trauma after fall.
 
Case No. 309
  1. Neuroleptic malignant syndrome.
Note: For details, see Case No. 022.
 
Case No. 310
  1. Hypercalcemia.
  2. PTH assay.
  3. Primary hyperparathyroidism.
  4. Parathytroid adenoma.
  5. Young patients age <50 yrs clear cut symptoms, documented complications such as peptic ulcer, renal stones, renal impairment or osteopenia.
    480
Note: In primary hyperparathyroidism, there is autonomous secretion of PTH usually by a single parathyroid adenoma (90%), multiple adenoma (4%), nodular hyperplasia (5%), carcinoma (1%). Most of the patients remain aysmptomatic or have vague symptoms. They should be reviewed every 6 to 12 months, with assessment of symptoms, renal function, serum calcium and bone mineral density.
 
Case No. 311
  1. Multiple Myeloma.
  2. Bone marrow study.
  3. Paraprotein deposition in renal tubule, hypercalcemia, infection, NSAIDs, amyloidosis, dehydration.
Note: Multiple myeloma is a malignant proliferation of plasma cells. In multiple myeloma, plasma cells produce immunoglobulin of a single heavy and light chain, a monoclonal protein called paraprotein. In some cases, only light chain is produced and this appears in urine as Bence Jones proteinuria.
 
Case No. 312
  1. Beta thalassemia major.
  2. Collapse of vertebrae.
  3. Radiotherapy to vertebral body.
Note: In beta thalassemia major, there is extramedullary hemopoiesis, enlargement of the spines of vertebra, which may cause compression of spinal cord, leading to paraplegia. In such case, radiotherapy may help to reduce neurological symptoms.
 
Case No. 313
  1. History of polyarthritis, skin rash, Raynauds phenomenon, oral ulcer, drug history, occupation.
  2. Butterfly rash, parotid swelling, alopecia, skin tightening.
Note: This history is suggestive of interstitial lung disease. It may be cryptogenic or secondary to other diseases such as - collagen disease (rheumatoid arthritis, scleroderma, dermatomyositis), sarcoidosis, extrinsic allergic alveolitis, drugs (busulphan, bleomycin, methotrexate, amiodarone, nitrofurantoin), asbestosis, silicosis, pneumoconiosis, long standing mitral valve disease, hemosiderosis.
High resolution CT scan is helpful in early diagnosis.481
 
Case No. 314
  1. Counseling is not done. Proper history of chest is not taken. Before surgery, proper investigation has not been done.
  2. CT scan of chest, isotope bone scan of whole body.
Note: Before total thyroidectomy, proper history, physical examination and investigation should be done. In any thyroid swelling, FNAC to exclude malignancy is essential. Also, serum thyroglobulin, other thyroid function tests and bone scan should be done to see any secondary.
 
Case No. 315
  1. Erythema induratum.
  2. Biopsy and histopathology.
Note: Erythema induratum is a form of panniculitis, usually found in the calf of legs of young female, also called Bazin's disease. It is characterized by chronic, recurrent, tender, subcutaneous, ulcerated nodule.
 
Case No. 316
  1. Oral methotrexate.
Note: Methotrexate is indicated in severe progressive psoriasis or diffuse exfoliative psoriasis or progressive arthropathy (arthritis mutilans). It is helpful both in psoriasis and arthropathy.
 
Case No. 317
  1. Eisenmenger's syndrome from reversal of patent ductus arteriosus (PDA).
Note: The lady is most likely suffering from patent ductus arteriosus (PDA) with reversal of shunt. This is called differential cyanosis and clubbing.
 
Case No. 318
  1. Pernicious anemia.
Note: She is suffering from pernicious anemia which is associated with other autoimmune disease like vitiligo in this case.482
 
Case No. 319
  1. Acrodermatitis from zinc deficiency.
Note: Acrodermatitis enteropathica is a rare inherited disorder characterized by zinc deficiency. It presents 4-6 weeks after weaning or earlier in bottle fed babies. Patients present with erythematous rashes or blisters around the perineum, mouth, hands and feet. It may be associated with photophobia, diarrhea and alopecia. Treatment is with lifelong oral zinc.
 
Case No. 320
  1. Cutaneous leishmaniasis.
  2. Biopsy of the skin lesion and staining for amastigotes.
Note: Cutaneous leishmaniasis is quite common in many Middle East countries. It is the most likely first diagnosis.
 
Case No. 321
  1. Dermatitis herpetiformis.
  2. Biopsy.
Note: Dermatitis herpetiformis presents with extremely itchy skin lesion and burning. Itching is paroxysmal and relapsing. Rarely oral mucosa may be involved. It may be associated with celiac disease, lymphoma and hypothyroidism. M:F is 2:1, 80% have HLA B8/DRw 3 and rare in childhood. It may be precipitated by taking iodide and gluten containing diet. Skin biopsy shows infiltration of neutrophils, eosinophils, fibrin at dermal papilla and subepidermal vesicle with neutrophil infiltration. Direct immunofluorescence in normal skin biopsy shows granular IgA deposition alone or with C3 at the dermo-epidermal junction with accentuation in dermal papillae (IgM and IgG are occasionally observed). Almost all patients have partial villous atrophy in jejunal biopsy, even without GIT symptoms.
 
Case No. 322
  1. Middle cerebral artery territory.
  2. Middle cerebral artery.
Note: Areas supplied by the middle cerebral artery include the most of the lateral surface of cerebral hemisphere (except for the superior part of the frontal and parietal lobes and the inferior part of the temporal lobe) and part of the internal capsule and basal ganglia. Its occlusion 483leads to paralysis and sensory loss of contralateral face and limbs and aphasia (when in dominant hemisphere). There may be homonymous hemianopia or quadrantanopia.
 
Case No. 323
  1. IV magnesium.
Note: Acute severe asthma is managed at hospital with high flow oxygen (40-60%), nebulized salbutamol 5 mg (or terbutaline 10 mg) with ipratropium bromide 0.5 mg 4 hourly and I/V hydrocortisone 200 mg 4 hourly for 24 hours followed by oral prednisolone 60 mg daily for 2 weeks. If there is no improvement, then magnesium sulphate 1.2-2 gm I/V over 20 minutes should be given. Other options are salbutamol 3-20 μg/min I/V or terbutaline 1.5-5.0 μg/min I/V. Ventilation should be considered, if PaCO2 is more than 7 kPa.
 
Case No. 324
  1. History of amenorrhea.
  2. Ultrasonogram of whole abdomen.
Note: Ultrasonogram of whole abdomen will show whether the patient is pregnant or not. It will also reveal any ascites, hepatic or renal abnormality or any intra-abdominal mass lesion.
 
Case No. 325 (see also Case No. 281)
  1. Drug induced SLE.
  2. Anti histone Ab.
Note: Drugs causing SLE are hydralazine, procainamide, carbamazepine, phenytoin, phenothiazine, INH, OCP, ACE inhibitor, penicillamine, methyldopa, minocycline, etc. Sex ration is equal in drug induced SLE. It commonly involves lung, but renal and neurological involvement are rare. ANA is usually positive, but anti-ds DNA is negative. Complements are normal. Antihistone antibody are positive in 95% of cases, this is characteristic but not specific. Drugs causing SLE like syndrome usually do not aggravate primary SLE. This is treated by withdrawal of drugs and short course of steroid, if necessary.
 
Case No. 326
  1. SIADH.
  2. Water restriction.
    484
Note: Treatment of SIADH involves restriction of fluid intake to 500 to 1000 mL/day and correction of underlying cause if possible. Plasma osmolarity, sodium and body weight should be measured frequently. If water restriction is not effective or poorly tolerated, then demeclocycline (600-1200 mg daily) may be given. In very severe cases, rarely, hypertonic saline (300 mmol/L slowly I/V) and frusemide may be given.
 
Case No. 327
  1. Night sweating, weight loss.
  2. Bone marrow study, CT chest and abdomen.
Note: Bad prognostic factors in non-Hodgkin's lymphoma are – age > 60 years, stage III or IV (advanced disease), high serum LDH, performance status (Eastern Cooperative Oncology Group ECOG 2 or more) and more than one extranodal site involvement.
 
Case No. 328
  1. Acromegaly.
  2. OGTT.
  3. Conduction problem. Visceral enlargement, IHD.
  4. Pegmisoment.
Note: Excess growth hormone production in adult (after epiphyseal fusion) leads to acromegaly. It is almost always due to a pituitary tumor, but rarely may be due to ectopic production of GHRH from pancreatic islet cell tumor, oat cell carcinoma of bronchus, medullary carcinoma of thyroid. Signs of active acromegaly include – progressive increase in size of the body, excessive sweating, increasing visual field defect, large skin tags (molluscum fibrosum), presence of glycosuria (DM), hypertension, progressive headache and enlarging thyroid.